abonnement Unibet Coolblue
  donderdag 23 december 2010 @ 18:00:19 #1
30719 keesjeislief
NextGenerationHippie
pi_90351969

Welkom in de officiële LaTeX-topic op Fok! Als je bezig bent een mooi document te zetten in LaTeX, maar je komt er niet uit, dan is dit de plek om je vraag te stellen. Je wordt echter het snelst en best geholpen als je een minimum example weet te maken. Probeer zoveel mogelijk packages die niets met het probleem te maken hebben uit je preamble te verwijderen, en probeer een zo klein mogelijk document over te houden door onnodige zaken eruit te halen. Vaak helpt dit sowieso al om te zien wat het probleem veroorzaakt (twee packages die niet met elkaar overweg kunnen) en daarbij geeft het duidelijkheid aan degenen die je vraag willen beantwoorden.

Als je gewoon wilt weten wat LaTeX nu eigenlijk is, of wat je nodig hebt om het te gebruiken, lees dan verder voor een overzicht van de benodigde software en cursussen.



Wat is LaTeX?
LaTeX (uitspraak: latech) is het meestgebruikte macropakket voor de opmaaktaal TeX (zie hieronder). De macro’s uit het pakket van LaTeX zorgen ervoor dat je op een eenvoudigere manier dan met TeX alleen (plain TeX genoemd) teksten kunt opmaken. LaTeX wordt meestal gebruikt om wetenschappelijke teksten en artikelen op te maken, maar dat is zeker niet de enige toepassing.
LaTeX is zoals blijkt het uit bovenstaande géén WYSIWYG-methode. Daarom wordt LaTeX vaak ook wel vergeleken met HTML, de code die je maakt beschrijft namelijk de opmaak van het document, pas nadat het gerenderd wordt zie je het eindresultaat – vaak als PDF.

Wat is TeX?
TeX is een opmaaktaal voor teksten en formules die als programmeertaal met uitbreidingsmogelijkheden te gebruiken is. Met TeX is het mogelijk om een ingewikkelde lay-out op een relatief eenvoudige manier te beschrijven. De eerste versie van TeX is verschenen in 1978, tegenwoordig is het in principe uitontwikkeld en worden er alleen nog fouten verholpen. De auteur, Donald Knuth, looft $327,68 uit voor elke bug die iemand vindt. Afgeleide projecten worden onder een andere naam voortgezet, zoals pdfTeX of XeTeX.

Wat is nou precies het voordeel van LaTeX?
LaTeX is gemaakt zodat de auteur zich tijdens het schrijven in feite zo min mogelijk moet bekommeren om de vorm. LaTeX automatiseert dan ook een heleboel, bijvoorbeeld kruisverwijzingen, koppen (het zorgt er bijvoorbeeld voor dat koppen nooit als laatste regel op een blad staan), positionering van afbeeldingen en woordafbreking. Ook kan men middels stijlbestanden relatief gemakkelijk een andere opmaak (bijvoorbeeld de huisstijl van de organisatie die het gaat publiceren) aan een document geven. Daarnaast is het grote voordeel t.o.v. bijvoorbeeld Microsoft Word dat het goed kan omgaan met zeer grote bestanden. In LaTeX kun je vrij simpel een bestand aanpassen dat honderden pagina’s bevat.
Voor andere, gespecialiseerde toepassingen, zijn ook extra pakketten verschenen, zoals het parallel zetten van teksten, het zetten van talen die van rechts naar links geschreven worden, het opmaken van kritische edities met meerdere voetnoten. Een zeer bekend voorbeeld is BibTeX dat automatisch uit een database de artikelen kan selecteren waarnaar verwezen wordt in een document, en deze volgens de gewenste criteria kan opmaken en sorteren.

Welke alternatieve macropakketten zijn er voor TeX?
Een alternatief macropakket is ConTeXt, ontworpen door de Nederlander Hans Hagen. Dit pakket probeert omvangrijker te zijn, en standaard meer opties te bieden dan LaTeX, zodat je minder extra packages nodig hebt. Er zijn echter ook minder packages in omloop die voor ConTeXt geschreven zijn.



Het lijkt me wel iets, wat heb ik allemaal nodig?
Je moet een editor hebben om je LaTeX-code in te kunnen schrijven, je hebt een TeX-engine nodig die deze code compileert, en als laatste heb je nog een viewer nodig die het eindproduct kan bekijken, en eventueel afdrukken. En als je er net mee begint heb je ook een beetje doorzettingsvermogen nodig. Voor de rest maakt het niet veel uit welk besturingssysteem je draait, want voor nagenoeg elke versie van Windows, elke BSD- of Linuxdistributie en Mac OS X is wel een pakket te vinden, maar zelfs als je nog BeOS of OS/2 draait is er wel een versie te vinden.

Welke editors zijn er zoal?
In principe voldoet elk programma waarin je gewone tekstbestanden kunt bewerken en kunt opslaan, zoals Notepad voor Windows, of TextEdit voor OS X. Er zijn echter speciale editors voor TeX, die sneltoetsen bieden om makkelijk speciale symbolen of omgevingen in te voeren, en die automatisch je TeX-engine kunnen aanroepen. Je hebt voor elk platform editors, sommige gratis en sommige alleen tegen een flink bedrag. Hier volgt een overzichtje van editors die een (beetje) extra functionaliteit hebben voor LaTeX:

Crossplatform
AUCTeX voor XEmacs en GNU Emacs. Dit is een pakket dat onder Emacs het editen van (La)TeX- (maar ook ConTeXt-) documenten vereenvoudigt. Het ondersteunt verder latex-preview, waardoor je bepaalde delen van je document (zoals formules) in een voorgerenderde vorm kunt bekijken terwijl je nog aan het bewerken bent. Omdat het met Emacs werkt krijg je verder ook de functionaliteit die je sowieso bij Emacs hebt. Emacs draait onder DOS, Windows, Linux, Solaris, de BSD’s, OS X, etc.
GNU TeXmacs. Een WYSIWYG editor voor TeX – niet LaTeX. De editor heeft ook een interface met algebra-programma’s (b.v. GNU Octave). Het is een heel eigen type editor, en niet goed vergelijkbaar met anderen.
LyX. Deze editor probeert je LaTeX te laten typen volgens het WYSIWYM paradigma. Dit geeft een soort combinatie tussen het typen van source, en het direct zien van een gerenderd resultaat. Je ziet in LyX iets wat het eindresultaat benaderd, dus b.v. voorgerenderde formules en geen \sum\_{k=1}\^{\infty}, maar je kunt het wel op die manier intypen. LyX is gratis en open source, het is beschikbaar voor Windows 2000/XP/Vista, Linux en Unixen en Mac OS X.
TeXmaker. Biedt syntax-highlighting, symbolenoverzicht, een structuuroverzicht en shortcuts om veelgebruikte omgevingen in te voeren. Het vereist wat meer handmatige configuratie onder Windows dan b.v. TeXnicCenter. Maakt gebruik van Qt dus het draait op Windows, Linux en OS X.
TeXworks. Een pakket geïntegreerd met viewer, dat gemodelleerd is naar het OS X programma TeXshop (zie hieronder). De handleiding is elders te vinden.
Vim-LaTeX: een pakket voor vim dat het bewerken van LaTeX-documenten probeert te vergemakkelijken, door bijvoorbeeld te integreren met de folding van Vim en het compilen vanuit vim. Standaard heeft Vim overigens ook al redelijke ondersteuning voor LaTeX, en b.v. spellingscontrole die weet wat TeX-commando’s zijn en deze niet controleert. Vim is beschikbaar voor Amiga, MS-DOS, Windows, Unix (incl. Linux), Mac (classic en OS X), maar ook voor je PDA.

Windows (Naast de bovengenoemde cross-platform editors.)
TeXnicCenter Gratis en Open Source. Richt zich vooral op beginners, door de meestgebruikte constructies in menu‘s te verstoppen. Biedt een overzicht van de structuur van het document dat je bewerkt en heeft spellingscontrole. Het is bedoeld als een geïntegreerd pakket dat verder weinig configuratie behoeft om te werken.
LEd (LaTeXEditor). Gratis, maar niet open source. Heeft een DVI-viewer, spellingscontrole, mogelijkheden symbolen in te voegen en werkt op Windows 95 t/m Vista. Werkt samen met MikTeX en TeX Live.
MeWa. Gratis, open source, en een afgeleide van TexnicCenter. Heeft ook uitgebreide spellingscontrole, de GUI is geïnspireerd op TeXnicCenter.
Scientific WorkPlace. Een grafische editor, waarbij je de sourcecode van je document niet te zien krijgt als je daar niet expliciet om vraagt. Het probeert mensen moeiteloos LaTeX te laten bewerken zonder dat ze het echt hoeven leren. Wordt op universiteiten nog wel eens aangetroffen, maar kost anders voor commercieel gebruik $845.
WinEdt, een editor met name voor (La)TeX. Deze is shareware, en biedt o.a. een overzicht van je document, en shortcuts om snel symbolen en omgevingen in te voeren.

Mac OS X (Naast de bovengenoemde cross-platform editors.)
TextMate. Deze editor richt zich niet speciaal op LaTeX, maar heeft wel hulpmiddelen. De editor is gratis te proberen voor 30 dagen, en kost daarna bijna ¤50,–.
TexShop. Vrij basale, gratis en open source editor voor OS X. Maakt het mogelijk om snel een document te compilen en te previewen, en biedt syntax highlighting. Het heeft een interface die goed integreert met OS X. Als cross-platform opvolger kan TeXworks beschouwd worden (zie hierboven).
iTeXMac een veel uitgebreidere TeX-editor voor OS X, alhoewel de ontwikkeling gestopt lijkt te zijn.

Linux
Onder Linux is het het gemakkelijkst om te zien welke editors je distributie meelevert. Als je die installeert, krijg je er waarschijnlijk ook een TeX-installatie bij. Van de cross-platform editors zul je de meesten ook wel in je Linux-distributie vinden.

AmyEdit Een opensource editor voor GTK, al een tijdje niet meer bijgewerkt. Het doel is lichtgewicht te zijn, en de editor heeft daarom vooral basisfeatures, zoals spellingscontrole en syntaxhighlighting.
Kile (gratis - Linux/KDE) Met wat moeite is deze editor ook wel onder Windows en Mac aan de praat te krijgen, maar hij is toch met name voor Linux geschreven. Het is een met TeXmaker vergelijkbare editor, en hij kent spellingscontrole, syntaxhighlighting, templates, documentoverzicht, etc.
Winefish. Een gratis editor voor Linux/KDE, voor gevorderde gebruikers.

Hoe kom ik aan een TeX-distributie?
Je bent het makkelijkst af door een hele TeX-distributie te downloaden. Dit omvat verschillende TeX-engines (b.v. pdfTeX om direct PDF-bestanden te maken), en ook een hele sloot aan extra packages en lettertypes: zaken die je misschien niet direct nodig hebt, maar allicht later van pas kunnen komen. Hier worden de relevante opties voor Linux, Windows en OS X genoemd, zoek je een distributie voor MS-DOS of Amiga of een Japanse distributie, kijk dan voor een vollediger overzicht onder TeXsystems in de TeX FAQ. Een distributie omvat (meestal) geen editor (proTeXt en MacTeX zijn uitzonderingen), die zul je dus apart moeten downloaden.

Windows
BaKoMa TeX Een sharewaredistributie van TeX. Deze richt zich met name op Acrobat en levert Type 1 lettertypen mee (dit zijn schaalbare lettertypen voor PDF). Ook heeft deze distributie een eigen editor.
MiKTeX. Dit is waarschijnlijk de meestgebruikte gratis Windows-distributie. Deze is niet zo groot qua omvang, omdat er standaard niet heel veel packages bijgeleverd worden, deze kun je echter via een interface downloaden als je ze nodig hebt. MikTeX draait niet op windowsversies ouder dan Windows 2000.
proTeXt. Een eveneens gratis distributie gebaseerd op MiKTeX, maar inclusief de editor TexNicCenter en de GhostScript-viewer voor Windows. Verder omvat het sowieso meer, want de hele download is maar liefst 540MB. Het is echter bedoeld om zonder problemen direct aan de slag te kunnen.
TeXLive. Een zeer omvangrijke, gratis, TeX-distributie, die oorspronkelijk voor Unix-systemen is ontworpen, wat ondanks de Windows-installer, onder Windows nog wel te merken valt. Beginners zijn beter af met proTeXt of MikTeX.

Unix en Linux
Ook hier is het het beste om te kijken wat je distributie aanbiedt. Let overigens op, als je distributie nog teTeX heeft, dan heb je waarschijnlijk een heel oude installatie, met moderne packages kun je dan problemen krijgen.

TeXLive. De de facto standaard TeX-installatie voor Linux en afgeleiden. Afhankelijk van je distributie kan het echter zijn dat je losse onderdelen kunt installeren. Je kunt ook een hele ISO downloaden, die loopt in de honderden MB’s. Voor OpenBSD zit texLive in de ports, voor NetBSD in de pkgsrc, voor Debian en Ubuntu is het gepackaged, in Gentoo kun je het emergen.

Mac OS X
Je kunt via Fink of Darwinports een TeX-distributie installeren, maar je kunt ook, en dat is het gemakkelijkst, de MacTeX distributie downloaden, die overigens wel vrij groot is (1.3GB). Optioneel zijn enkele hulpprogramma’s, zoals editors en een BibTeX managers te downloaden in de package MacTeXtras.

Wat heb ik nodig om gecompileerde documenten te bekijken?
Afhankelijk of je latex of pdflatex hebt gedraaid heb je een DVI-bestand of een PDF-bestand gegenereerd. (In beginsel kun je via tex4ht ook HTML-bestanden genereren, maar daar gaan we niet van uit). Het DVI-bestand wordt meestal geconverteerd naar PostScript of PDF, maar het kan ook direct bekeken worden. Sommige editors hebben standaard een DVI-viewer meegeleverd.

Windows
MikTeX levert de DVI-viewer al mee, deze heet ‘yap’, maar daar hoef je je doorgaans geen zorgen om te maken, omdat je editor deze waarschijnlijk al vindt. Met ‘yap’ kun je meestal in de DVI-file klikken (als je ergens een fout ziet) om naar de corresponderende plek in je source-code te gaan. Het hangt van je editor af hoe je dit moet instellen.

Voor PDF-bestanden zijn er ook een aantal mogelijkheden:
Adobe reader. Het bekendst, werkt altijd, maar ook vrij log. Een document herladen gaat niet heel rap.
Foxit. Een kleinere, snellere viewer. De basisversie is gratis.
GhostView. Het standaardprogramma om PostScript te bekijken, maar het opent ook PDF. De interface is niet al te appetijtelijk, maar het programma zelf is heel functioneel.
Sumatra PDF Kleiner, sneller, en ondersteunt het herladen door op ‘r’ te drukken.

Linux
DVI wordt bekeken met ‘xdvi’, DVI converteren naar PS gaat met dvips, naar PDF met dvipdf of dvipdfm. dvipdfm converteert rechtstreeks, dvipdf converteert via PS, en dan naar PDF. Voor KDE is er kdvi en vanaf KDE 4 ‘Okular’.

Voor PDF zijn er andermaal meer mogelijkheden.
Adobe reader. Het bekendst, maar is gesloten, niet voor elke Linux-distributie beschikbaar (zeker niet op alternatieve hardwareplatformen), en het is traag en log.
GhostView. In de meeste Linux-distributies ook aanwezig. Het wordt ook vaak onder de motorkap gebruikt in applicaties als eps2pdf, ps2pdf en b.v. ImageMagick om PostScript bestanden te renderen en te converteren.
xpdf. Sneller, maar rendert graphics iets minder mooi, heeft echter de heel handige mogelijkheid dat je door op ‘r’ te drukken het document kunt herladen en dat je direct de bladzijde weer voor je neus krijgt waar je was.

Mac OS X.
De Mac komt standaard met Preview.app; dat is een snelle PDF-viewer. De meeste workflow op Mac gebruikt dan ook PDF, en DVI minder. Als je wilt kun je onder X11 wel xdvi draaien. Preview opent ook PostScript bestanden door ze te converteren naar PDF. GhostScript is ook onder X11 aan de praat te krijgen, en via Fink of DarwinPorts te installeren.

Voor PDF zijn er nog wat alternatieven:
Adobe reader. Acrobat onder OS X is ook lang niet zo snel als Preview, zeker niet als het om zoeken in PDF-bestanden gaat.
Skim. Een open Source PDF viewer, die ook de mogelijkheid biedt om aantekeningen te maken bij PDF’s en notes toe te voegen. Let wel, deze worden niet in de PDF zelf opgeslagen.



Ik wil LaTeX leren, waar doe ik dat?
Je kunt online een heleboel materiaal vinden, of je kunt een boek ter hand nemen. Het is mijns inziens het gemakkelijkst om een online tutorial te volgen, zeker voor LaTeX leer je daar het meest van. Wil je echter meer van TeX zelf weten op den duur, dan kan het wel nuttig zijn om een boek aan te schaffen, of in je (universiteits)bibliotheek te kijken.

Boeken over LaTeX
LaTeX, a Document Preparation System by Leslie Lamport (second edition, Addison Wesley, 1994, ISBN 0-201-52983-1) Het standaard boek voor LaTeX, door de auteur van LaTeX geschreven. Zeker niet onmisbaar, maar vaak wel in universiteitsbibliotheken aanwezig.
The LaTeX Companion (Tools and Techniques for Computer Typesetting) by Frank Mittelbach, Michel Goossens, Johannes Braams, David Carlisle and Chris Rowley (second edition, Addison-Wesley, 2004, ISBN-10 0-201-36299-6, ISBN-13 978-0-201-36299-2) De eerste editie heeft een hond op de voorkant, de tweede een kompas. Omvat veel voorbeelden en beschrijvingen van extra packages. Dit kan wel nuttig zijn.

Online materiaal over LaTeX
Veel materiaal is in het Engels, maar er is ook een aantal Nederlandse handleidingen.

LaTeX handleiding door Piet van Oostrum (NL)
Een Introductie tot het Zetsysteem LaTeX door Gaspard Lequeux (NL)
LaTeX voor beginners door Wilfried Van Hirtum (NL)
Formatting information: a beginner’s introduction to typesetting with LaTeX door Peter Flynn (EN)
The Not So Short Introduction to LaTeX 2e door Tobias Oetiker (EN) De bekendste handleiding. Er is een Nederlandse vertaling van, maar deze is achterhaald. Als je deze helemaal doorneemt heb je een goed overzicht, ook van een aantal extra packages en mogelijkheden.

Er zijn ook enkele pagina’s gewijd aan de behoeften van een selectere groep gebruikers, of een specialistischer onderwerp:
LaTeX voor logici
LaTeX voor taalkundigen
LaTeX mathmode. Een uitgebreid overzicht van alle wiskundeomgevingen in LaTeX en AMS-LaTeX. Handig om een keer door te lezen.
Memoir manual. Memoir is een pakket dat bedoeld is om de opmaak van een LaTeX-document aan te passen. Het eerste deel van de handleiding is echter een inleiding over typografische regels en gewoonten. Dit leert je wat zaken waar je op moet letten om verzorgde teksten te maken.
Free Math Font Survey. Een overzicht van de beschikbare lettertypen voor LaTeX. Let wel, niet elk lettertype omvat alle symbolen.
Comprehensive LaTeX Symbol list. Een groot document met allerhande mogelijke symbolen die je ooit zou willen gebruiken in (La)TeX: wiskundig, natuurkundig, astronomisch, astrologisch, taalkundig, dingbats, etc.

Waar kan ik terecht met vragen?
Uiteraard in deze topic! Maar er zijn ook andere websites waar LaTeX-vragen worden beantwoord. Je hebt de zeer uitgebreide UK TeX FAQ, waar heel veel voorkomende problemen worden aangekaard (en eigenlijk alles wat in deze OP staat ook te vinden is). Verder heb je comp.text.tex, een drukbezochte news-group waar veel mensen te vinden zijn, inclusief veel auteurs van packages, en zeer, zeer ter zake kundigen die alle ins en outs van TeX kennen. Mits je een beknopte vraag stelt, in het Engels, met een duidelijk minimum example word je hier meestal goed geholpen.



Wat zijn de handigste packages?
Dat hangt er natuurlijk maar net vanaf wat je wilt. Er is echter wel een aantal packages te geven dat vaker dan gemiddeld gebruikt wordt. Hieronder wordt dan ook een aantal scenario’s besproken.

Welke package verandert Chapter in Hoofdstuk, etc?
Welke package geeft woordafbreking?
LaTeX kent geen ingebouwde spellingscontrole (dat kan je editor soms geven, zie hierboven), maar het kent wel een uitgebreid mechanisme voor woordafbreking. De package die dit regelt is ‘babel’. Je gebruikt Babel met ‘\usepackage[english,dutch]{babel}’. Dit geeft aan dat je document in twee talen geschreven wordt, Engels en Nederlands, waarbij Nederlands de standaard is, wat ook betekent dat koppen in het Nederlands worden gezet (b.v. Hoofdstuk, en niet Chapter).

In het document moet je handmatig aangeven wat Engels en Nederlands is. Als je hoofdtaal Nederlands is, dan kun je bijvoorbeeld \foreinglanguage{english}{Premature optimization is the root of all evil (or at least most of it) in programming.}

Babel regelt overigens nog meer, in het Frans is het gebruikelijk om kleine witruimtes rond de interpunctie te hebben Dus meer « zo » en niet zo vaak «zo». Babel kan hier ook rekening mee houden. Voor het Oud-Grieks zijn er ook mogelijkheden om polytonische teksten te typen, en zo heeft elke taal wel specifieke opties, welke je in de documentatie kunt vinden.

Omdat XeTeX geheel van Unicode, OpenType en UTF-8 invoer uitgaat, is Babel daar vaker een gesel dan een zegen. Daarom is de package Polyglossia ontwikkeld, dat voor XeLaTeX bedoeld is, deze package is nog in ontwikkeling. Als je echter een document in het Thaïs en Arabisch wilt schrijven is deze package wellicht handig, aangezien het ook automatisch van lettertype kan wijzigen als je van taal verandert.

Wat is het beste afbeeldingenformaat voor LaTeX?
Voor pdfLaTeX geldt dat je bestanden in PNG, JPG en PDF formaat kunt opnemen in je document, voor ‘gewoon’ LaTeX geldt dat bestanden in .EPS-formaat moeten zijn, eventueel moet je dat dus converteren.
In feite gelden voor LaTeX dezelfde regels als in het algemeen: voor foto’s kun je prima JPG gebruiken, dit levert een redelijk klein bestand op. Voor screenshots kun je beter PNG gebruiken, aangezien dit geen artefacten oplevert zoals JPG (denk aan smoezelige randjes). Voor schaalbare afbeeldingen, zoals grafieken of diagrammen is een vectorformaat het ideaalst: bij het printen blijft dit mooi.

Als je kunt kiezen in welk formaat je kunt exporteren, b.v. in Matlab, dan heeft iets als EPS de voorkeur. Afhankelijk of je LaTeX of pdfLaTeX gebruikt moet je de EPS converteren naar PDF. Sommige programma’s kunnen ook speciaal voor LaTeX exporteren, meestal betekent dit dat er code gegenereerd wordt die de tekst door LaTeX laat zetten. Dit heeft als voordeel dat het lettertype in de afbeelding hetzelfde is als in je document. Ook is schalen meestal makkelijker, omdat de tekst niet meeschaalt. Om bestanden te converteren kun je ImageMagick gebruiken, of GhostScript, dat onder Linux vaak voorzien is van de scripts ‘ps2pdf’ en aanverwanten (epstopdf). xpdf levert het programma ‘pdftops’.

Het programma epstopdf kan ook automatisch aangeroepen worden vanuit pdfLaTeX, maar alleen als je LaTeX met de optie -shell-escape draait, wat niet standaard is omdat het een (klein) veiligheidsrisico oplevert.

En hoe neem ik nu zo’n bestand daadwerkelijk op?
Daarvoor gebruik je de package ‘graphicx’. Dit kent een \includegraphics-commando waarmee je een bestand kunt opnemen. In het simpelste geval werkt het ongeveer zo:

1
2
3
4
\begin{figure}
  \includegraphics{myfig}
  \caption{Mijn figuur!}
\end{figure}

Als je je document nu compileert met ‘latex’ kijkt het over een document bestaat dat ‘myfig.eps’ heet, als je compileert met ‘pdflatex’ of er een document bestaat dat ‘myfig.pdf’, ‘myfig.jpg’ of ‘myfig.png’ heet. Het is dus niet nodig om de extensie op te geven! Je kunt nog extra opties opgeven om bijvoorbeeld de grootte van het plaatje in te stellen, het plaatje te schalen, of slechts een gedeelte van het plaatje weer te geven, die staan handleiding beschreven.

Welke afbeeldingenprogramma’s zijn speciaal geschikt voor LaTeX?
Een programma dat eenzelfde aanpak heeft voor het tekenen van plaatjes als LaTeX voor het zetten van teksten is MetaPost. Je beschrijft de tekening. De labels worden direct door TeX gezet, en de uitvoer is EPS. Voor LaTeX is dit tamelijk ideaal, en voor PDFLaTeX is het ook nog redelijk gemakkelijk. De problemen ontstaan zodra je andere lettertypes wilt gebruiken in MetaPost, daarnaast moet je ook een nieuwe taal leren (met soms een wat vreemde syntax).

Daarom is een ander programma, Asymptote ontwikkeld dat eenzelfde filosofie volgt: je beschrijft je tekening in een soort C-achtige taal. Een compiler geeft dan een PDF of EPS bestand. Er is ook een grafische interface in ontwikkling, xasy.

Een oudgediende op dit vlak is IPE, geschreven door Otfried Cheong, dat werkt m.b.v. een grafische interface, maar speciale ondersteuning heeft om labels m.b.v. LaTeX te zetten, en verder ook voor geometrische tekeningen kan ‘snappen’ naar snijpunten van lijnen.

Een ander programma dat goed met LaTeX overweg kan is xfig (niet voor Windows), dit is geheel grafisch, maar kan exporteren in LaTeX-formaten. Dit is met name geschikt als je diagrammen wilt maken. Voor grafieken is GNUPlot het aangewezen programma. Dit kent allerhande manieren om data en functies te plotten.

Matlab, Maple en Mathematica kunnen ook in EPS-formaat opslaan en kennen speciale LaTeX-functionaliteit. Ook de opensource wiskundige programma’s zoals Octave en R kunnen dit.

Is er ook iets om in LaTeX figuren te kunnen tekenen?
Van oudsher is het uitgebreidste tekenpakket voor LaTeX PSTricks. Dit maakt gebruik van PostScript (wat in feite een hele programmeertaal is) om afbeeldingen te maken. Als je je LaTeX document via DVI naar PostScript compileert werkt dit uitstekend. De ondersteuning is heel uitgebreid, het voordeel van het feit dat het in je LaTeX source wordt opgenomen is dat je ook gemakkelijker (als je dat zou willen) ‘over je pagina heen kunt tekenen’ en dat consistentie met tekst qua lettertype heel gemakkelijk is. Een nadeel is dat het niet (gemakkelijk) werkt met pdfLaTeX, omdat PDF slechts een deelverzameling van de PostScript-commando’s kent. PSTricks kent heel veel extra packages die het maken van afbeeldingen heel erg kunnen vereenvoudigen.

Een nieuwer alternatief is PGF met TikZ. TikZ is de ‘high level’ laag bovenop PGF. TikZ heeft als voordeel dat het zowel met pdfLaTeX als met LaTeX werkt (en zelfs met tex4ht, omdat het SVG kan produceren), en het qua syntax gewoon LaTeX-achtig is. De handleiding is zeer uitgebreid en er is op een andere plek een voorbeeldengallerij te vinden. Het nadeel aan TikZ is dat het heel traag kan zijn: het doet alle berekeningen in TeX, en dat is geen snelheidsmonster. Ook is het aantal packages minder groot dan voor PSTricks. Het wordt wel centraal ontwikkeld, wat de consistentie van de interface ten goede komt.

Als laatste zijn er nog pakketten zoals XYPic, maar deze raken onderhand allemaal verouderd en bovengenoemde pakketten hebben alle functionaliteit ook in zich.

Welke pakketten zijn nog meer handig voor figuren?
Het pakket subfig kun je gebruiken om afbeeldingen naast elkaar, of onder elkaar te zetten en te nummeren als ‘Fig. 1a’. De functionaliteit van dit pakket wordt ook bijvoorbeeld door Memoir geleverd, dus als je dat gebruikt is subfig niet nodig. ‘subfigure’ is een oudere implementatie van hetzelfde idee, je kunt beter subfig gebruiken.

Kan ik ook presentaties maken met LaTeX?
Ja zeker, er zijn zowel packages om PowerPoint-achtige slides te maken (maar dan een stuk verzorgder uiteraard), als om transparanten te maken. Het aangewezen pakket voor het eerste soort presentaties is ‘beamer’. Dit stelt je in staat om in een manier analoog aan hoe je normaal LaTeX-documenten maakt een presentatie te maken, maar dan met ‘\begin{slide}’, et cetera. Ook zijn er mogelijkheden om delen van een slide nog niet weer te geven in het begin, maar pas nadat je op spatie drukt. pdfLaTeX wordt gewoon gebruikt om de slide te renderen, dus de kwaliteit van de wiskunde is bijvoorbeeld excellent. De handleiding is zeer uitgebreid en bevat ook tutorials. Een voorbeeld is hier te vinden.

Een ouder, gelijksoortig pakket, is ‘prosper’, dit biedt echter minder functionaliteit, HA-prosper is hier een afgeleide van. Beamer biedt wel functionaliteit om prosper te emuleren.

PDF-presentaties worden gegeven door je PDF-applicatie in full-screen te zetten (voor Adobe is dit b.v. Ctrl+L of Appeltje+L). Het biedt als zodanig dus niet de functionaliteit van PowerPoint, en je wilt daarom misschien zelf een horloge bij de hand houden. Met Beamer is het wel mogelijk om in een dualscreen-setup op je laptop notes te hebben en op het projectiescherm niet. Dit werkt echter niet zo soepel als met PowerPoint of Keynote.

Voor transparanten is foiltex te gebruiken. Als je met een heel oude LaTeX-installatie opgescheept zit kun je desnoods de class ‘slides’ gebruiken. Beamer kan dit eventueel ook, daarnaast heeft Beamer faciliteiten om aan de hand van de slides ook handouts te genereren die meer tekst bevatten, of om een PDF te genereren (met behulp van pgfpages) die 4 of 8 slides per pagina heeft om uit te printen (eventueel kan zelfs de achtergrondkleur automatisch veranderd worden om inkt te sparen).

Met welke package maak ik mijn PDF klikbaar?
Met de package hyperref kun je (interne) links in je document klikbaar maken. Je kunt bijvoorbeeld in de inhoudsopgave of de index klikken om automatisch naar een pagina te gaan, je kunt kruisverwijzingen automatisch in hyperlinks omzetten en ook voetnoten. Als je al bij voorbaat hyperref gebruikt is het aan te raden om niet ‘zie Figuur \ref{fig:vb}’ te typen, maar om ‘zie \autoref{fig:vb}’ te typen, dit geeft hetzelfde resultaat in je PDF-bestand behalve dat in het laatste geval Figuur ook klikbaar is – anders kun je alleen op het getal klikken.

In de handleiding staat ook beschreven hoe je de presentatie van de links kunt aanpassen, standaard krijg je rode kadertjes om de links heen, dat is niet heel mooi.

Welke package geeft toegang tot extra PDF-functionaliteit zoals forms en de documentinfo?
Dit is ook hyperref. Hiermee kun je voor PDF de keywords en de auteur instellen, je kunt forms maken en je kunt instellen hoe je document geopend moet worden, b.v. single page, double page, of fullscreen. Iets wat je niet kunt instellen is de mogelijkheid om commentaar toe te voegen in je PDF, dit kan alleen met de volledige versie van Acrobat geactiveerd worden. Dan kun je wel in Adobe Reader commentaar toevoegen. Er is ook geen gratis software die dit kan activeren.

Welke package heb ik nodig voor een bibliografie?
Er zit een heel basale bibliografiefunctionaliteit standaard in LaTeX (deze staat b.v. uitgelegd in de ‘not so short introduction to LaTeX’), maar als je grotere documenten wilt maken is BibTeX het aangewezen programma. Dit is niet zozeer een package, als wel een los programma – maar wel eentje dat standaard meegeleverd wordt.

Een goede tutorial voor BibTeX is Tame The Beast – de documentatie van BibTeX zelf is niet héél duidelijk. Het idee is als volgt: je maakt een bestand waarin je alle artikelen en boeken die eventueel zou willen citeren opsomt, in een formaat als dit:

1
2
3
4
5
6
@book{companion, 
    author = "Goossens, Michel and Mittelbach, Franck and Samarin, Alexander", 
    title = "The {{\LaTeX}} {C}ompanion", 
    publisher = "Addison-Wesley", 
    year = 1993, 

Als je besluit dat je dit werk wilt citeren dan type je in je LaTeX-document \cite{companion}. De eerste keer dat je je document compileert mist LaTeX al deze verwijzingen. Dan echter draai je BibTeX, dat kijkt welke verwijzingen je allemaal hebt gebruikt, en deze haalt het uit bestand met artikelen. Dan maakt het de verwijzingen in de gewenste stijl op en sorteert de lijst eventueel. Als een bepaald tijdschrift of je school of universiteit op een speciale stijl staat, kun je BibTeX het werk laten doen. Je krijgt dan b.v. als auteurs M. Goossens, F. Mittelbach and A. Samarin, of M. Goossens et al. Verwijzingen in de tekst kunnen met nummers zijn, of b.v. GMA1993.

Een overzicht van de verschillende stijlen van BibTeX vind je hier.

Oorspronkelijk is BibTeX ontworpen voor Engelse literatuurlijsten bij technische artikelen. Ondertussen wordt het voor allerhande artikelen en talen gebruikt, het loopt dan ook op z’n achterste benen. De taal waarin de stijl-bestanden worden beschreven is ook niet heel toegankelijk. Als opvolger is BibLaTeX gemaakt, dat bijvoorbeeld op een eenvoudiger wijze de gebruiker de opmaak laat aanpassen.

Om je bibliografie-database te beheren zijn er ook vele programma’s in omloop. Sommige editors hebben hiervoor een hulpmiddel in handen, verder heb je voor OS X Bibdesk – dat standaard bij MacTeX komt, het commerciële Endnote dat ook in BibTeX kan exporteren, JabRef dat onder Java draait, RefDB dat ook andere formaten kan exporteren, of web-based systemen zoals Aigaion. Na een tijdje leg je een grote database aan en zijn zulke systemen onontbeerlijk!

Met welke package kan ik de marges veranderen?
Voor dit geldt: bezint, eer ge begint. De standaard layout in LaTeX is wel met enig overleg gekozen. Het is geen al te hippe layout, maar wel een functionele. De marges zijn op A4-papier heel breed, maar dat is ook omdat A4 heel breed is. Weinig boeken worden op A4-formaat uitgegeven, en als ze al zo groot zijn hebben ze vaak meerdere kolommen (kranten b.v. ook), dit omdat heel brede regels niet prettig lezen: als je oog naar de volgende regel gaat raak je gemakkelijker het spoor bijster en begin je een regel te hoog of te laag te lezen. Ook een zekere optische balans wordt nagestreefd. Voordat je de marges dus op 2cm zet, denk hier aan.

De marges en ruimte voor kop- en voetteksten aanpassen kan met geometry, of op een iets simpeler wijze vmargin.

Omvattender packages, zoals KOMA-script en memoir bieden deze functionaliteit ook.

Met welke package kan ik de opmaak van koppen veranderen?
Dit is misschien niet zo eenvoudig als je zou verwachten. Voor een aantal voorgekookte stijlen kun je fncychap gebruiken. Wil je nog iets anders, dan zul je zelf moeten programmeren (en voor paragrafen). Ook kun je weer een van de omvattender packages nemen, zoals KOMA-script en memoir, die hebben een makkelijker interface om de stijl aan te passen.

Met welke package kan ik kopteksten en voetteksten aanpassen?
Fancyhdr wordt hier het meest voor gebruikt, en is ook meestal aan te raden, tenzij je al een complexere class als memoir gebruikt die de functionaliteit ingebouwd heeft.

Met welke package kan ik andere lettertypen kiezen?
Het standaardlettertype van LaTeX is wel heel herkenbaar, maar op den duur wel een beetje saai. Het is ook niet echt voor z’n esthetische waarde ontworpen. Het biedt wel veel (zoals verschillende ontwerpgrootten, nette ligaturen, groot tekenrepetoir). Een goede start is de Free Math Font Survey.

De moeite van het bekijken waard zijn pxfonts (een Palatino-kloon), txfonts (Times-kloon) en Euler (een informeler Lettertype): voor presentaties zou je ook CMBright kunnen overwegen.

Wiskundige lettertypen zijn dungezaaid, dus voor wiskundige teksten ben je noodzakelijkerwijs op een klein aantal lettertypen aangewezen, omdat de meesten simpelweg de symbolen niet hebben. Er zijn wel commerciële lettertypen voor LaTeX, zoals Lucida, en als je Minion hebt gekocht kun je MnSymbol gebruiken. Wil je echter vooral prozatekst zonder bijzondere symbolen zetten, dan zijn er meer mogelijkheden als je een OpenType-font hebt.

OpenType-fonts kunnen geconverteerd worden naar een formaat dat LaTeX snapt, hiervoor kun je otfinst gebruiken. Een andere optie is XeTeX, dat direct OpenType-lettertypen kan gebruiken.

Met welke package kan ik een index maken?
Dit komt standaard met LaTeX mee, de makeidx package. Een flexibeler alternatief is xindy, dat beter om kan gaan met andere talen dan Engels. Voor afkortingen zijn ook specialistische packages zoals acronym voor afkortingen en glosstex voor een glossary.

Welke package is geschikt voor <...>?
Uiteraard kan niet elke package hier behandeld worden, er zijn nog genoeg packages die b.v. symbolenlijsten geven, mogelijkheden om tabellen te maken die over meerdere pagina’s gaan, mooiere tabellen te maken, parallelle teksten te zetten, kritische edities te zetten, en wat al niet. Er is een goede onderwerpsindex voor alle packages. Als je het daar niet vindt, dan kun je het altijd vragen!

Ik heb de package gevonden die ik nodig heb, maar hoe installeer ik die?
Als je MikTeX gebruikt, kijk dan eerst of je het in MikTeX niet in de package-manager kunt vinden. Dit zal meestal het geval zijn.

Voor Linux geldt dat je distributie vaak wel heel veel packages beschikbaar heeft in pakketten, het kan zijn dat je meer moet installeren, bijvoorbeeld omdat het in het pakket ‘texlive-humanities’ of ‘texlive-science’ zit (in Debian). Dit is het gemakkelijkst.

Zit het niet in de package-manager, of heeft die alleen een oude versie, dan kun je, als je het pakket maar voor één project nodig hebt, gewoon de gewenste .sty-file in dezelfde directory als je document zetten. Het kan zijn dat je package alleen als .ins en .dtx geleverd wordt, dan moet je het eerst zelf compileren: draai (pdf)latex <package>.ins en (pdf)latex <package.dtx> en je krijgt een .sty (of .cls) bestand en de documentatie.

Is het echter een groot pakket, of gaat het erom dat je een bestand wil patchen waar een bug in zit, of wil je de nieuwste versie, dan zul je het echt zelf moet installeren. Dit kan, je kunt een privé-directorystructuur erop nahouden. Als er bestanden zijn die zowel in jouw directory als in de systeemdirectory staan krijgt jouw bestand voorrang. Die directory wordt meestal aangeduid met $TEXMFHOME, en hoe je daar precies mee moet werken staat in de TUG FAQ.



Hier staan de vragen die al vaker voorbij gekomen zijn in de LaTeX-topics. Veel vragen zijn hierboven al beantwoord, als dat zo is wordt daarnaar verwezen.

Wat heb ik nodig om te kunnen LaTeXen?
Lees hierboven het stuk onder benodigdheden. Je kunt perfect uit te voeten met allemaal gratis software, of je nou Linux, Mac of Windows draait!

Is het nou handig?
Uiteindelijk wel. Het heeft een vrij steile leercurve, en voor eenvoudige documenten is het dan ook een stukje ingewikkelder dan Word – alhoewel dat voor een ervaren gebruiker niet meer geldt, maar uiteindelijk zijn langere documenten een stuk beter te behappen in LaTeX dan in Word. Het is echter geen volwaardig DTP-programma, zoals Quark of InDesign, en de opmaak voor een tijdschrift of glossy is dan ook echt moeilijker.

Kan ik LaTeX naar Word omzetten of omgekeerd?
Nee, niet direct. Je hebt wel rtf2latex2e en dat kan je aardig op weg helpen, maar het zal nooit perfect zijn. Ook is er latex2rtf. Dit laatste is echter geen TeX-engine, dus het kan al snel het spoor bijster raken. Je kunt nara tex4ht kijken, dat HTML als uitvoer biedt.

Hoe maak ik presentaties?
Gebruik beamer. Dat staat hierboven, onder packages, beschreven.

Hoe verander ik de lettertypes?
Dat staat ook hierboven uitgelegd, onder packages.

M’n papierformaat wordt niet correct doorgegeven!
Ook al doe je \documentclass[a5paper], dan nog kan het zijn dat Adobe denkt dat het A4 is. Door simpelweg \usepackage{geometry} te doen, los je dit meestal op, deze package schrijft namelijk de benodigde speciale commando’s naar de PDF (of DVI) die de viewer correct instrueren.

De opmaak zint me niet.
Hierboven staan packages, links en uitleg over het aanpassen van lettertypen, marges en koppen.

Mijn figuur staat niet waar ik het wil.
LaTeX kent zogenaamde ‘floats’, dit zijn bijvoorbeeld tabellen of figuren. Die kunnen wat wegdrijven van de plek waar jij ze opgeeft. Dit zorgt ervoor dat de pagina’s beter gevuld kunnen worden. Word doet dit niet, maar eigenlijk doet verder elke zetter of zetprogramma het wel. Kijk maar eens in professioneel gezette boeken. Figuren komen bovenaan een pagina te staan, of op een aparte pagina. Soms echter drijft een figuur een volgende paragraaf in, dat kan wel hinderlijk zijn (na een \clearpage, wat meestal bij een hoofdstuk gebeurt, worden figuren automatisch neergezet). Wil je dit niet, dan is de package placeins de oplossing. Hiermee kun je barrières definiëren waar floats niet voorbij mogen drijven. Wil je echt per se dat je figuur blijft staan op de plek waar jij het wilt, kijk dan in de TeX FAQ.

Hoe krijg ik het symbool voor …?
Kijk in de Comprehensive LaTeX Symbol list. Een andere goede optie is Detexify², waar je het symbool kunt tekenen en laten herkennen (let op: Vereist Firefox 3.5, Opera 9.6 of Safari 4, Internet Explorer werkt niet).

Mijn EPS plaatje werkt niet?
Je gebruikt waarschijnlijk pdfLaTeX, en dat kan geen EPS aan (EPS kan niet in PDF opgenomen worden), je moet het eerst naar PDF converteren, dat kan b.v. met epstopdf (zie ook hierboven).



TeX leer je door te doen. Van vrijwel alle packages kun je de source gewoon bekijken in je distributie. Als je zelf een .cls of .sty-bestand wilt schrijven, kijk dan eens in een .sty-bestand uit je distributie. Wil je iets aanpassen, kijk naar de definitie. Er zijn veel pagina’s met voorbeelden en examples. Lees je een manual, en denk je, hoe hebben ze dat voor elkaar gekregen? Je kunt meestal de source vinden.

Hier volgt een overzichtje van websites met voorbeelden:

TeX showcase. Voorbeelden wat met TeX allemaal mogelijk is.
TikZ example gallery
PSTricks examples
Tips voor wiskundige formules.
Een volledig Lineaire Algebraboek, inclusief source.
De source van de banners zoals die in deze topic gebruikt worden. (En de PDF).
Een voorbeeld LaTeX-scriptie die een goed idee geeft hoe je een hele scriptie met LaTeX schrijft. (En nog een linkverzameling.)

Overige relevante links
De Nederlandstalige TeX gebruikersgroep
LaTeX Homepage
Officiële website met TeX software, templates en meer
LaTeX-packages op onderwerp.
TeX FAQ
Google TeX Group

Vorige edities van deze topic.
LaTeX
LateX #2 TeXneuten beantwoorden TeXnische vragen
[LaTeX #3] Leer hier de TeXnische kneepjes van het vak.
[LaTeX #4] TeXnici helpen bij TeXnische problemen.
[LaTeX #5] TeXnische foefjes en trucjes.
[LaTeX #6] TeXnologen voor de zetTeXniek.

Je vindt de source van dit document op de Wiki, waar je deze zelf kunt aanpassen en van nuttige tips en links kunt voorzien.
heeft de hoop dat het allemaal stiekum toch nog goed komt...
Fotoboek
  donderdag 23 december 2010 @ 18:03:22 #2
30719 keesjeislief
NextGenerationHippie
pi_90352109
Ik had de vorige per ongeluk volgemaakt. :@.
heeft de hoop dat het allemaal stiekum toch nog goed komt...
Fotoboek
  donderdag 23 december 2010 @ 18:29:12 #3
296399 Outlined
Renaissance Man
pi_90353128
zie ik nu dat je ook muzieknotatie met LaTeX kan doen ?
Come on, who can, who can, can hear the bass drum.
pi_90529246
Is er een makkelijke manier om een url toe te voegen aan mijn bibliografie met BibDesk? Het lijkt me dat zoiets makkelijk moet kunnen, maar ik kan het niet vinden en blijkbaar Google ik ook niet goed genoeg. :?
Op dinsdag 23 augustus 2011 23:18 schreef problematiQue het volgende:
Mensen die zomaar claimen dat A beter is dan B moet je gewoon negeren. Internetruis.
  dinsdag 28 december 2010 @ 13:22:59 #5
30719 keesjeislief
NextGenerationHippie
pi_90534074
quote:
1s.gif Op donderdag 23 december 2010 18:29 schreef Outlined het volgende:
zie ik nu dat je ook muzieknotatie met LaTeX kan doen ?
Dat kan idd.

quote:
1s.gif Op dinsdag 28 december 2010 11:08 schreef Gebraden_Wombat het volgende:
Is er een makkelijke manier om een url toe te voegen aan mijn bibliografie met BibDesk? Het lijkt me dat zoiets makkelijk moet kunnen, maar ik kan het niet vinden en blijkbaar Google ik ook niet goed genoeg. :?
Als je googelt op "bibdesk add url to bib" vind je wel wat, zoals dit.
heeft de hoop dat het allemaal stiekum toch nog goed komt...
Fotoboek
pi_91051044
Hallo,

ik heb het gevoel dat dit vrij "basic" is maar toch vind ik het niet in handleidingen. Ik heb een stelling op het laatste van een pagina, het bewijs begint op het volgende... niet zo mooi...Is er een manier om een "blok" af te bakenen dat zeker op dezelfde pagina moet.

Zoiets als

\begin{pageblock}
bla bla

bla bla
\end{pageblock}

Want gewoon \newpage doen om de boel kunstmatig te manipuleren zou wel eens tot genante situaties kunnen leiden als je toch nog zaken verandert....
  zondag 9 januari 2011 @ 05:48:05 #7
30719 keesjeislief
NextGenerationHippie
pi_91056780
quote:
1s.gif Op zondag 9 januari 2011 00:24 schreef zuiderbuur het volgende:
Hallo,

ik heb het gevoel dat dit vrij "basic" is maar toch vind ik het niet in handleidingen. Ik heb een stelling op het laatste van een pagina, het bewijs begint op het volgende... niet zo mooi...Is er een manier om een "blok" af te bakenen dat zeker op dezelfde pagina moet.

Zoiets als

\begin{pageblock}
bla bla

bla bla
\end{pageblock}

Want gewoon \newpage doen om de boel kunstmatig te manipuleren zou wel eens tot genante situaties kunnen leiden als je toch nog zaken verandert....
Wat leesvoer: http://www.tex.ac.uk/cgi-bin/texfaq2html?label=nopagebrk. Ik moet overigens zeggen dat ik het eens ben met de onderliggende boodschap op die pagina, nl. dat het eigenlijk iets is dat 'niet past binnen de Latex-filosofie'. Het hebben van een aantal lege regels en een stelling die op een volgende pagina begint is (ook) lelijk. Persoonlijk heb ik me nooit gestoord aan dat soort afbrekingen, dat hoort er gewoon bij en er is m.i. niets mis mee...
heeft de hoop dat het allemaal stiekum toch nog goed komt...
Fotoboek
pi_91104448
quote:
1s.gif Op zondag 9 januari 2011 05:48 schreef keesjeislief het volgende:

[..]

Wat leesvoer: http://www.tex.ac.uk/cgi-bin/texfaq2html?label=nopagebrk. Ik moet overigens zeggen dat ik het eens ben met de onderliggende boodschap op die pagina, nl. dat het eigenlijk iets is dat 'niet past binnen de Latex-filosofie'. Het hebben van een aantal lege regels en een stelling die op een volgende pagina begint is (ook) lelijk. Persoonlijk heb ik me nooit gestoord aan dat soort afbrekingen, dat hoort er gewoon bij en er is m.i. niets mis mee...
Bedankt voor de link. Ik denk dat je gelijk hebt: misschien aanvaard ik het beter. Minipage vind ik wat overdreven, en \begin{samepage}...\end{samepage} werkt gewoon niet bij mij (het komt nog steeds op verschillende pagina's terecht).

Ik had trouwens nog een vraagje omtrent subscripten links van een letter.. dit heb je nodig bij de getwiste chevalleygroepen. Ik ben hier al eens geholpen voor dit probleem, maar nu zie ik nog steeds een probleem, als er nog iets voor het subscript komt, wordt die "2" wel erg ver verwijderd van de A:

1
2
3
4
5
6
7
8
\documentclass{article}
\begin{document}

\[^2A_{2 d -1}(q) = ^2\!A_{2 d -1}(q)\]

\[iets= ^2A_{2 d -1}(q) = ^2\!A_{2 d -1}(q)\]

\end{document}

Hoe kan ik dit wat mooier maken? Bedankt!
  maandag 10 januari 2011 @ 13:17:54 #9
30719 keesjeislief
NextGenerationHippie
pi_91111411
quote:
1s.gif Op maandag 10 januari 2011 09:30 schreef zuiderbuur het volgende:

[..]

Bedankt voor de link. Ik denk dat je gelijk hebt: misschien aanvaard ik het beter. Minipage vind ik wat overdreven, en \begin{samepage}...\end{samepage} werkt gewoon niet bij mij (het komt nog steeds op verschillende pagina's terecht).

Ik had trouwens nog een vraagje omtrent subscripten links van een letter.. dit heb je nodig bij de getwiste chevalleygroepen. Ik ben hier al eens geholpen voor dit probleem, maar nu zie ik nog steeds een probleem, als er nog iets voor het subscript komt, wordt die "2" wel erg ver verwijderd van de A:
[ code verwijderd ]

Hoe kan ik dit wat mooier maken? Bedankt!
Bedoel je zoiets?:

1
2
3
4
5
6
7
8
\documentclass{article}
\begin{document}

\[^2A_{2 d -1}(q) = ^2\!A_{2 d -1}(q)\]

\[iets= ^2\hspace{-0.2cm}A_{2 d -1}(q) = ^2\!A_{2 d -1}(q)\]

\end{document}
heeft de hoop dat het allemaal stiekum toch nog goed komt...
Fotoboek
pi_91149644
quote:
1s.gif Op maandag 10 januari 2011 13:17 schreef keesjeislief het volgende:

[..]

Bedoel je zoiets?:

[ code verwijderd ]

Dat ziet er keurig uit, dank je! Best eigenaardig dat het zo verandert als je er iets voor plaatst maar dit lost het op.

Ik heb trouwens nog een vraagje... ik schrijf een (lange) tekst met een driedelige Appendix, waarvan één deel in het Nederlands... De verandering van taal los op ik zo op:
1
2
3
4
5
6
7
8
9
10
\usepackage[dutch,english]{babel}

....
\appendix
\include{engelsetekst1}
\include{engelsetekst2}

\selectlanguage{dutch}
\include{nederlandsetekst1}
\selectlanguage{english}

En dat werkt (Figure wordt wel degelijk Figuur in dat stuk, de "hyphenatie" of woordsplitsing is aangepast,....) maar in de hoofding wordt "Appendix" opeens "Bijlage".... Dat vind ik nu eigenlijk een beetje overdreven, dat mag best "Appendix" blijven.... Is er een manier om deze verandering te vermijden?

Bedankt!

Edit : op de allerlaatste pagina van de Nederlandse tekst staat er trouwens niet "Bijlage" in de hoofding, maar "B"ylage" ... dat is uiteraard niet de bedoeling, het is zeer lelijk en ook mysterieus.

[ Bericht 7% gewijzigd door zuiderbuur op 11-01-2011 10:40:41 (extra probleem) ]
pi_91197220
quote:
14s.gif Op dinsdag 11 januari 2011 08:07 schreef zuiderbuur het volgende:

[..]

Dat ziet er keurig uit, dank je! Best eigenaardig dat het zo verandert als je er iets voor plaatst maar dit lost het op.

Ik heb trouwens nog een vraagje... ik schrijf een (lange) tekst met een driedelige Appendix, waarvan één deel in het Nederlands... De verandering van taal los op ik zo op:

[ code verwijderd ]

En dat werkt (Figure wordt wel degelijk Figuur in dat stuk, de "hyphenatie" of woordsplitsing is aangepast,....) maar in de hoofding wordt "Appendix" opeens "Bijlage".... Dat vind ik nu eigenlijk een beetje overdreven, dat mag best "Appendix" blijven.... Is er een manier om deze verandering te vermijden?

Bedankt!

Edit : op de allerlaatste pagina van de Nederlandse tekst staat er trouwens niet "Bijlage" in de hoofding, maar "B"ylage" ... dat is uiteraard niet de bedoeling, het is zeer lelijk en ook mysterieus.
Moet kunnen met dit en dit.
pi_91198620
quote:
1s.gif Op woensdag 12 januari 2011 04:07 schreef PorFavor het volgende:

[..]

Moet kunnen met dit en dit.
Bedankt, met

1
2
3
4
\addto\captionsdutch{%
  \renewcommand{\appendixname}%
    {Appendix}%
}
komt het inderdaad in orde!
pi_91662079
Ik heb opnieuw een vraagje, over TexShop ditmaal.....
Ik gebruik dus altijd dit programma, nu zie ik
bij File-> Page Setup... dat "US Letter" en niet A4 de standaard is bij mij.
Ik kan dat aanpassen per document, maar het is natuurlijk vrij vreemd dat dat zo is, en een beetje gevaarlijk... kan ik de "default" soms wijzigen? (vond het niet in help)

Verder dacht ik dat dit de oorzaak was van een ander probleem... ik heb een template van een flyer dat bij mij afgesneden wordt aan de rechterkant. Ik hoor van mensen die texshop niet gebruiken dat het met a4 te maken heeft... zij zetten om van dvi naar ps/pdf en geven daarbij in de command line de optie "-t a4" in.

Weet iemand hoe je dit dan aanpakt in TexShop?

Met veel dank!
  vrijdag 21 januari 2011 @ 15:53:08 #14
30719 keesjeislief
NextGenerationHippie
pi_91663392
quote:
1s.gif Op vrijdag 21 januari 2011 15:24 schreef zuiderbuur het volgende:
Ik heb opnieuw een vraagje, over TexShop ditmaal.....
Ik gebruik dus altijd dit programma, nu zie ik
bij File-> Page Setup... dat "US Letter" en niet A4 de standaard is bij mij.
Ik kan dat aanpassen per document, maar het is natuurlijk vrij vreemd dat dat zo is, en een beetje gevaarlijk... kan ik de "default" soms wijzigen? (vond het niet in help)

Verder dacht ik dat dit de oorzaak was van een ander probleem... ik heb een template van een flyer dat bij mij afgesneden wordt aan de rechterkant. Ik hoor van mensen die texshop niet gebruiken dat het met a4 te maken heeft... zij zetten om van dvi naar ps/pdf en geven daarbij in de command line de optie "-t a4" in.

Weet iemand hoe je dit dan aanpakt in TexShop?

Met veel dank!
Sorry maar ik ken TexShop niet.
heeft de hoop dat het allemaal stiekum toch nog goed komt...
Fotoboek
pi_91670349
quote:
1s.gif Op vrijdag 21 januari 2011 15:53 schreef keesjeislief het volgende:

[..]

Sorry maar ik ken TexShop niet.
TexShop is bij mijn weten "het" programma voor de Apple-gebruikers. Het is wat ik gewend ben, maar ik krijg wel vreemde dingen.

Ik wil een flyer maken (met \documentclass[a4paper,dutch]{article} bovenaan, en met "landscape" en "multicol")

Ik doe dan File->Page Setup (20.99 op 29.70 cm), en dan ziet pdf er "afgesneden" uit aan de rechterkant....
Ik zit nu toch al een tijdje te zoeken, en ik zie niet welke instelling de boosdoener is.
  maandag 24 januari 2011 @ 11:45:39 #16
46802 Visitor.Q
Neemt alles op!
pi_91791075
Ik gebruik nu XeTeX ism Linux Libertine -- een mooi font en er zitten tal van glyphs bij om bv een Romeinse II te maken uit 1 glyph. Ook biedt het de mogelijkheid tot numbers old-style. Dat vind ik best mooi, die doorhangende 3 enzo maar het ziet er soms wat gekkig uit:

• In (literatuur) referenties in de ISSN of DOI staan cijfers en letters door elkaar zonder spatie, dat hoort zo maar het ziet er gek uit. Plaatje onderaan spreekt (denk ik) voor zich. Ik kan van de hoofdletters in ISSN/DOI natuurlijk kleine letters maken maar zo krijg ik ze nu eenmaal geëxporteerd vanuit ScienceDirect en dan zou ik alles moeten gaan nalopen. Dat doe ik toch wel maar het kan natuurlijk niet de bedoeling zijn dat dit niet automatisch gebeurt :)
• Is het aan te raden om überhaupt old-style figures te gebruiken in wetenschappelijk werk (lees: mijn thesis)? Als ik in math-mode zit, krijg ik bijvoorbeeld new-style cijfers, wat me voor die modus ook geschikt lijkt (ik ga geen vergelijkingen e.d. in old-style zetten). Dat zou betekenen dat ik voornamelijk sectie- en paginanummering en jaartallen in old-style zou zetten.
• Is er iets te doen aan de haakjes die om de cijfers heen staan, ze lijken wat groot.

pi_92322397
Ik blijf deze error krijgen ! LaTeX Error: File `memoir.sty' not found.
Maar zelfs als ik de package memoir handmatig download is er nergens een memoir.sty?
"AAAAAHH ZENNE MOAT, WOARST VLEISCH"
pi_92596359
Vraagje...

Ik wil graag dat de description environment niet geheel wordt uitgelijnd naar links... wat je normaal hebt als je...

1
2
3
4
5
6
7
8
9
10
11
12
13
14
15
16
\documentclass[a4paper]{article}                    
\begin{document}

Lorem ipsum dolor sit amet, consectetur adipiscing elit. Aenean tristique justo
mollis magna accumsan pharetra. Praesent placerat pulvinar nisl, at consequat
turpis sodales eget.

\begin{description}
\item[aaa] meervoud van de letters van a
\item[bbb] meervoud van de letters van b
\end{description}

Sed feugiat tortor ac libero condimentum at iaculis nisi vestibulum. Fusce
dignissim, dolor non scelerisque eleifend, nunc libero scelerisque arcu, eu
vehicula mauris lacus a turpis.
\end{document}
gebruikt. Hoe zorg ik er dus voor dat de description environment dan m.b.t. de verticale positie dus, iets naar rechts wordt verschoven, 2 tabs bijvoorbeeld?

[ Bericht 10% gewijzigd door Dale. op 10-02-2011 18:22:50 ]
pi_92606306
quote:
1s.gif Op donderdag 10 februari 2011 17:38 schreef Dale. het volgende:
Vraagje...

Ik wil graag dat de description environment niet geheel wordt uitgelijnd naar links... wat je normaal hebt als je...

[ code verwijderd ]

gebruikt. Hoe zorg ik er dus voor dat de description environment dan m.b.t. de verticale positie dus, iets naar rechts wordt verschoven, 2 tabs bijvoorbeeld?

Zo kun je een list envoirment aanpassen:

1
2
3
4
5
6
7
8
9
10
\newenvironment{items}{%
\begin{list}{\textbullet}{%
\setlength{\topsep}{\onelineskip}
\setlength{\partopsep}{0pt}
\setlength{\parsep}{\parskip}
\setlength{\itemsep}{\parskip}
\setlength{\leftmargin}{0pt}
\setlength{\itemindent}{0pt}
}
}{\end{list}}
"AAAAAHH ZENNE MOAT, WOARST VLEISCH"
pi_92607452
quote:
1s.gif Op donderdag 10 februari 2011 20:45 schreef znarch het volgende:

[..]

Zo kun je een list envoirment aanpassen:
[ code verwijderd ]

1
2
3
4
Description    Resource    Path    Location    Type
Illegal unit of measure (pt inserted). \begin{items} (followed by: )    document.tex    /test    line 19    Texlipse Build Error
Missing number, treated as zero. \begin{items} (followed by: )    document.tex    /test    line 19    Texlipse Build Error
Undefined control sequence:  \begin{items} (followed by: )    document.tex    /test    line 19    Texlipse Build Error

Klopt iets niet me je code. Maar terzijde in die list kan ik niet \item[blabla] doen.
pi_92612549
Hij werkt hier wel. En je hoeft de mijne niet te gebruiken. Het ging om de parameters om de text uit te lijnen. En die zie je hier wel.
"AAAAAHH ZENNE MOAT, WOARST VLEISCH"
pi_92652085
quote:
1s.gif Op donderdag 10 februari 2011 22:08 schreef znarch het volgende:
Hij werkt hier wel. En je hoeft de mijne niet te gebruiken. Het ging om de parameters om de text uit te lijnen. En die zie je hier wel.
Mini example?
pi_92652326
In hoeverre is het mogelijk dingen uit mathematica in latex beamer te zetten? Wat ik met google vond ging over er movies van maken, maar ik heb een programmaatje met allemaal schuifjes (deze) die ik er graag in zou zetten.

Is dat uberhaupt mogelijk?

[ Bericht 5% gewijzigd door Hanneke12345 op 11-02-2011 23:16:55 ]
pi_92655804
quote:
1s.gif Op vrijdag 11 februari 2011 20:04 schreef Hanneke12345 het volgende:
In hoeverre is het mogelijk dingen uit mathematica in latex beamer te zetten? Wat ik met google vond ging over er movies van maken, maar ik heb een programmaatje met allemaal schuifjes ([url=http://demonstrations.wolfram.com/GaussianPrimes/]deze[/url) die ik er graag in zou zetten.

Is dat uberhaupt mogelijk?
Als je dat naar een film bestand, avi ofzo, kunt converteren dan kan het. Maar dat hele programme'tje met bij behorende functionaliteit (schuifbalkjes enzo) krijg je er nooit in.
pi_92657484
Jammer, maar had ik wel verwacht ;x Heb je nog tips hoe ik het 't makkelijkst als avi kan krijgen?
pi_92657684
quote:
1s.gif Op vrijdag 11 februari 2011 19:58 schreef Dale. het volgende:

[..]

Mini example?
Wat wil je exact zien? :)
"AAAAAHH ZENNE MOAT, WOARST VLEISCH"
pi_92658522
quote:
1s.gif Op vrijdag 11 februari 2011 21:57 schreef znarch het volgende:

[..]

Wat wil je exact zien? :)
De code die jij gebruikt waar die geen errors oplevert?

quote:
1s.gif Op vrijdag 11 februari 2011 21:53 schreef Hanneke12345 het volgende:
Jammer, maar had ik wel verwacht ;x Heb je nog tips hoe ik het 't makkelijkst als avi kan krijgen?
Nee sorry ben niet zo bekend met mathematica... maar ik zou hier en hier eens kijken.
pi_92659207
quote:
1s.gif Op vrijdag 11 februari 2011 22:13 schreef Dale. het volgende:

[..]

De code die jij gebruikt waar die geen errors oplevert?

[..]

Nee sorry ben niet zo bekend met mathematica... maar ik zou hier en hier eens kijken.
Ik ga nu stappen, zal morgen even voor je in elkaar draaien.
"AAAAAHH ZENNE MOAT, WOARST VLEISCH"
pi_92663820
Iemand een idee hoe je er voor kan zorgen dat de codes automatisch in kleur weergegeven worden? Ik vind het een stuk overzichtelijker, maar ik moet het nu elke keer als ik het opstart opnieuw instellen.
The biggest argument against democracy is a five minute discussion with the average voter.
pi_92678309
quote:
1s.gif Op vrijdag 11 februari 2011 19:58 schreef Dale. het volgende:

[..]

Mini example?
1
2
3
4
5
6
7
8
9
10
11
12
13
14
15
16
17
18
19
20
21
\documentclass{memoir}
\begin{document}

\newenvironment{items}{%
\begin{list}{\textbullet}{%
\setlength{\topsep}{\onelineskip}
\setlength{\partopsep}{0pt}
\setlength{\parsep}{\parskip}
\setlength{\itemsep}{\parskip}
\setlength{\leftmargin}{0pt}
\setlength{\itemindent}{0pt}
}
}{\end{list}}
text
\begin{items}
\item test1
\item test2
\end{items}
text

 \end{document}
"AAAAAHH ZENNE MOAT, WOARST VLEISCH"
pi_92681121
quote:
1s.gif Op zaterdag 12 februari 2011 14:18 schreef znarch het volgende:

[..]
[ code verwijderd ]

Thanks. Ligt dus blijkbaar aan dat jij memoir gebruikt als documentclass... als je article zou gebruiken zou je die errors ook krijgen denk ik. Verder lijkt het er niet echt op dat ik iets kan aanpassen aan de description environment want als ik nu dit gebruik verandert er helemaal niets.

1
2
3
4
5
6
7
8
9
10
11
12
13
14
15
16
17
18
19
20
21
22
23
24
25
26
27
28
29
30
31
32
33
\documentclass{memoir}
\begin{document}

\newenvironment{items}{%
\begin{description}{}{%
\setlength{\topsep}{2000pt}
\setlength{\partopsep}{200pt}
\setlength{\parsep}{2000pt}
\setlength{\itemsep}{200pt}
\setlength{\leftmargin}{200pt}
\setlength{\itemindent}{200pt}
}
}{\end{description}}
Lorem ipsum dolor sit amet, consectetur adipiscing elit. Pellentesque lacus
sapien, tincidunt nec sollicitudin ut, molestie in nisl. Vivamus id metus
purus, quis bibendum nunc. Proin condimentum, sem sit amet pharetra fringilla,
lectus ante varius lorem, in fermentum tortor felis eget purus. Donec cursus
pellentesque posuere.
\begin{items}
\item[Lorem ipsum dolor] \hfill \\
Lorem ipsum dolor sit amet, consectetur adipiscing elit.
Pellentesque lacus sapien, tincidunt nec sollicitudin ut, molestie in nisl. 
\item[Vivamus] Vivamus id metus
purus, quis bibendum nunc. Proin condimentum, sem sit amet pharetra fringilla,
lectus ante varius lorem, in fermentum tortor felis eget purus. Donec cursus
pellentesque posuere.
\end{items}
Lorem ipsum dolor sit amet, consectetur adipiscing elit. Pellentesque lacus
sapien, tincidunt nec sollicitudin ut, molestie in nisl. Vivamus id metus
purus, quis bibendum nunc. Proin condimentum, sem sit amet pharetra fringilla,
lectus ante varius lorem, in fermentum tortor felis eget purus. Donec cursus
pellentesque posuere.
 \end{document}
pi_92681131
quote:
1s.gif Op vrijdag 11 februari 2011 23:54 schreef M.rak het volgende:
Iemand een idee hoe je er voor kan zorgen dat de codes automatisch in kleur weergegeven worden? Ik vind het een stuk overzichtelijker, maar ik moet het nu elke keer als ik het opstart opnieuw instellen.
Ik snap niet helemaal wat je bedoelt maar het gaat? Maar ik denk dat je last hebt dat de latex code niet automatisch wordt gehighlight in je omgeving waar je het bewerkt, IDE? Zo ja... dan gewoon de juiste IDE gebruiken die .tex bestanden herkent en meteen de goeie syntax highlighting toepast?
pi_92683893
quote:
1s.gif Op zaterdag 12 februari 2011 15:48 schreef Dale. het volgende:

[..]

Ik snap niet helemaal wat je bedoelt maar het gaat? Maar ik denk dat je last hebt dat de latex code niet automatisch wordt gehighlight in je omgeving waar je het bewerkt, IDE? Zo ja... dan gewoon de juiste IDE gebruiken die .tex bestanden herkent en meteen de goeie syntax highlighting toepast?
Nou, zo zou ik het er het liefste uit hebben zien,

maar de tekst die hier rood en blauw is is bij mij iedere keer zwart, waardoor het dus wat onoverzichtelijk wordt. Hoe bedoel je precies, de juiste IDE?
The biggest argument against democracy is a five minute discussion with the average voter.
pi_92684872
quote:
1s.gif Op zaterdag 12 februari 2011 17:09 schreef M.rak het volgende:

[..]

Nou, zo zou ik het er het liefste uit hebben zien,
[ afbeelding ]
maar de tekst die hier rood en blauw is is bij mij iedere keer zwart, waardoor het dus wat onoverzichtelijk wordt. Hoe bedoel je precies, de juiste IDE?
IDE = Integrated Development Environment of te wel software-ontwikkelomgeving voor programmeurs. Maar eigenlijk is dat niet zo van betrekking op het punt. Wat jij gewoon zoekt is een goeie editor ;)

Maar TeXnicCenter zou ik eens proberen, http://www.texniccenter.org/. Of je kan natuurlijk ook gewoon notepad++, http://notepad-plus-plus.org/, gebruiken. Deze laatste heeft dan wel niet die zijbalk aan de linkerkant.

Of ander eclipse als je daarmee bekend bent, deze beschikt namelijk over een plugin texclipse.

Voor andere editors zou ik even kijken in de OP, onder 'benodigdheden' het puntje 'Welke editors zijn er zoal?'
pi_92686431
quote:
1s.gif Op zaterdag 12 februari 2011 15:48 schreef Dale. het volgende:

[..]

Thanks. Ligt dus blijkbaar aan dat jij memoir gebruikt als documentclass... als je article zou gebruiken zou je die errors ook krijgen denk ik. Verder lijkt het er niet echt op dat ik iets kan aanpassen aan de description environment want als ik nu dit gebruik verandert er helemaal niets.

[ code verwijderd ]

Volgens mij is alleen \onelineskip uit mijn voorbeeld memoir specifiek.
"AAAAAHH ZENNE MOAT, WOARST VLEISCH"
pi_92686435
Hallo,

ondanks heb ik een werk ingediend wat waarschijnlijk helemaal niet mogelijk zou zijn zonder de eerste-klas-hulp die altijd krijg in deze topics, dus eerst en vooral: _O_ :) ^O^

Verder wou ik eens polsen naar jullie mening over wiskundige presentaties, niet in Beamer maar in Keynote. Dat is de Apple-variant van PowerPoint. Dat is heel dynamisch (als je wil komt alles als een komeet of al draaiend in beeld) maar niet zo sympathiek als je zinnen vol met wiskundige symbolen staan.
Wie nieuwsgierig is kan hier een aantal tutorials bekijken.
Een eerste oplossing is het selecteren en dan copy-pasten uit een pdf-file (maar dan is de achtergrond soms niet compatibel!). Een andere mogelijkheid is het gebruik van LatexIt, een klein programma waarmee je snel een kort stukje latex-code kan omzetten in een pdf-je, dat je rechtstreeks kan slepen naar de file.

Hebben jullie positieve ervaringen met Latex in Keynote? Hoe pakken jullie dit aan?

Met dank!
pi_92688674
quote:
1s.gif Op zaterdag 12 februari 2011 18:21 schreef znarch het volgende:

[..]

Volgens mij is alleen \onelineskip uit mijn voorbeeld memoir specifiek.
Ja klopt, zie het nu ook. Maar iig kan niks veranderen helaas.
pi_92695608
quote:
5s.gif Op zaterdag 12 februari 2011 18:21 schreef zuiderbuur het volgende:
Hallo,

ondanks heb ik een werk ingediend wat waarschijnlijk helemaal niet mogelijk zou zijn zonder de eerste-klas-hulp die altijd krijg in deze topics, dus eerst en vooral: _O_ :) ^O^

Verder wou ik eens polsen naar jullie mening over wiskundige presentaties, niet in Beamer maar in Keynote. Dat is de Apple-variant van PowerPoint. Dat is heel dynamisch (als je wil komt alles als een komeet of al draaiend in beeld) maar niet zo sympathiek als je zinnen vol met wiskundige symbolen staan.
Wie nieuwsgierig is kan hier een aantal tutorials bekijken.
Een eerste oplossing is het selecteren en dan copy-pasten uit een pdf-file (maar dan is de achtergrond soms niet compatibel!). Een andere mogelijkheid is het gebruik van LatexIt, een klein programma waarmee je snel een kort stukje latex-code kan omzetten in een pdf-je, dat je rechtstreeks kan slepen naar de file.

Hebben jullie positieve ervaringen met Latex in Keynote? Hoe pakken jullie dit aan?

Met dank!
Ik geef mijn wiskundige voordrachten doorgaans met krijt, dus ik kan je helaas niet helpen.
pi_92696439
quote:
1s.gif Op zaterdag 12 februari 2011 19:29 schreef Dale. het volgende:

[..]

Ja klopt, zie het nu ook. Maar iig kan niks veranderen helaas.
Hoe bedoel je? Ik volg je even niet meer.
"AAAAAHH ZENNE MOAT, WOARST VLEISCH"
pi_92700790
quote:
1s.gif Op zaterdag 12 februari 2011 22:19 schreef znarch het volgende:

[..]

Hoe bedoel je? Ik volg je even niet meer.
Dat wanneer je "\onelineskip" verandert in 200pt bijvoorbeeld dat de errors weg zijn ;). Maar iig wanneer ik dus iets verander aan de parameters, verandert er niets aan de vormgeving.
pi_92720394
Ik wil in mijn file dat een een normale paragraaf geen witregel krijgt.

1
2
\setlength{\parindent}{1 ex}
\setlength{\parskip}{0 ex}

Als ik dat wel wil gebruik ik:

1\newcommand{\npar}{\par \hspace{5mm} \vspace{2.3ex plus 0.3ex minus 0.3ex}} 

Nu werkt mijn npar command wel. Maar als ik in mijn text een nieuwe paragraaf begin door een dubbele enter werkt de parindent niet. Iemand enig idee?
"AAAAAHH ZENNE MOAT, WOARST VLEISCH"
pi_92758082
Geen idee, als ik \setlength{\parindent}{1 ex} en \setlength{\parskip}{0 ex} er bij mij in plak, werkt het gewoon. Kan het misschien liggen aan je documentclass of confilcterende packages?
Kun je anders ff een mininal example posten?
pi_92758457
quote:
1s.gif Op zondag 13 februari 2011 15:57 schreef znarch het volgende:
Ik wil in mijn file dat een een normale paragraaf geen witregel krijgt.
[ code verwijderd ]

Als ik dat wel wil gebruik ik:
[ code verwijderd ]

Nu werkt mijn npar command wel. Maar als ik in mijn text een nieuwe paragraaf begin door een dubbele enter werkt de parindent niet. Iemand enig idee?
Minimum example bitte.
pi_92826347
Hallo,
ik gebruik onderstaande code om in meerdere kolommen te schrijven

1
2
3
4
5
6
7
8
9
10
11
12
13
14
    
\documentclass{article}
\usepackage{multicol}

\begin{document}
\begin{minipage}[t]{15cm}
    
        \begin{multicols}{2}
        \parskip\medskipamount
        
blablaa
    \end{multicols}\end{minipage}

\end{document}
Ik heb moeite om het helemaal te reconstrueren in een minimal example, maar het viel me op dat de lettergrootte veranderde naarmate ik die breedte van 15 cm liet variëren. Is dat normaal?
pi_92902303
Ik heb voor een project op de TU een tabel gemaakt, waarvan ik de 3e kolom het liefst zou willen line-wrappen, omdat de text anders buiten het scherm valt.
Ik heb een oplossing gevonden, maar ik vind deze niet zo netjes, is er een andere manier om de 3e kolom te laten wordwrappen?
Dit is wat ik momenteel gebruik:
1
2
3
4
5
6
7
\bgroup
  \catcode`\_=13%
  \def_{\textunderscore\hskip0pt}%
\begin{tabular}{|p{3.5cm}|p{3.5cm}|p{7.5cm}|}\hline
Header 1 & Header 2 & Hele lange tekst die niet past zonder wordwrapping oid\\ \hline
\end{tabular}
\egroup
das einzige erleuchtenden an religion ist ein brennende tempel bei nacht
pi_93031278
Hallo,

ik, als newbie latex gebruiker probeer een figuur in mijn document te kijgen (TexShop),
maar ik krijg de volgende foutmelding,

Missing & inserted.
<inserted text>
$
l.53 \includegraphics[width=6cm]{logo.jpg}

In de preamble heb ik het volgende staan m.b.t de afbeeldingen:
\usepackage{graphicx}
\graphicspath{{./Users/gebruiker/}}

\begin{document}

\begin{figure}
\includegraphics[width=6cm]{logo.jpg}
\end{figure}

\end{document}

Wie kan me helpen?
pi_93031467
quote:
1s.gif Op zaterdag 19 februari 2011 22:25 schreef klaaskootstra het volgende:
Hallo,

ik, als newbie latex gebruiker probeer een figuur in mijn document te kijgen (TexShop),
maar ik krijg de volgende foutmelding,

Missing & inserted.
<inserted text>
$
l.53 \includegraphics[width=6cm]{logo.jpg}

In de preamble heb ik het volgende staan m.b.t de afbeeldingen:
\usepackage{graphicx}
\graphicspath{{./Users/gebruiker/}}

\begin{document}

\begin{figure}
\includegraphics[width=6cm]{logo.jpg}
\end{figure}

\end{document}

Wie kan me helpen?
Is dat je hele document sectie? Het lijkt er namelijk eerder op dat je ergens anders iets fout hebt gedaan.
das einzige erleuchtenden an religion ist ein brennende tempel bei nacht
pi_93032297
Nee, het is een selectie vd grafische stukjes.
Wat ik eigenlijk wil, is het logo invoegen in de header. Maar voorlopig lukt het invoegen van een figuur zelfs nog niet.

Hele doc:

\documentclass[11pt,a4paper,oneside]{book}

\usepackage{a4wide} % Iets meer tekst op een bladzijde
\usepackage[english]{babel}

%afbeeldingen
\usepackage{graphicx}
\graphicspath{{./xxx/}}

%\usepackage[small,bf,hang]{caption}

%\setlength{\parindent}{0cm}
%\renewcommand{\baselinestretch}{1.2}

\usepackage{fancyhdr}
\pagestyle{fancy}
\fancyhf{}
\fancypagestyle{plain}
\fancyhead[R]{\bfseries Txxx}
%\fancyhead[L]{\includegraphics[width=3cm]{logo}}

\fancyfoot[R]{\thepage}
\fancyfoot[L]{CONFIDENTIAL}

\renewcommand{\headrulewidth}{0.4pt}
\renewcommand{\footrulewidth}{0pt}

\usepackage{amssymb}

% Standaard letterype naar Arial
\renewcommand{\rmdefault}{phv} % Arial
\renewcommand{\sfdefault}{phv} % Arial

\title{test}
\author{xxx}
\date{}

\begin{document}
\frontmatter

\maketitle

\tableofcontents

\mainmatter
\chapter{The beginning}
adfadsf

\includegraphics[width=6cm]{logo}

adsfadf
\newpage
bla bla

\chapter{The second}
in the meanwhile

\chapter{The end}
bye bye

\appendix

\backmatter
\end{document}
pi_93032587
quote:
1s.gif Op zaterdag 19 februari 2011 22:44 schreef klaaskootstra het volgende:
Nee, het is een selectie vd grafische stukjes.
Wat ik eigenlijk wil, is het logo invoegen in de header. Maar voorlopig lukt het invoegen van een figuur zelfs nog niet.

Hele doc:

\documentclass[11pt,a4paper,oneside]{book}

\usepackage{a4wide} % Iets meer tekst op een bladzijde
\usepackage[english]{babel}

%afbeeldingen
\usepackage{graphicx}
\graphicspath{{./xxx/}}

%\usepackage[small,bf,hang]{caption}

%\setlength{\parindent}{0cm}
%\renewcommand{\baselinestretch}{1.2}

\usepackage{fancyhdr}
\pagestyle{fancy}
\fancyhf{}
\fancypagestyle{plain}
\fancyhead[R]{\bfseries Txxx}
%\fancyhead[L]{\includegraphics[width=3cm]{logo}}

\fancyfoot[R]{\thepage}
\fancyfoot[L]{CONFIDENTIAL}

\renewcommand{\headrulewidth}{0.4pt}
\renewcommand{\footrulewidth}{0pt}

\usepackage{amssymb}

% Standaard letterype naar Arial
\renewcommand{\rmdefault}{phv} % Arial
\renewcommand{\sfdefault}{phv} % Arial

\title{test}
\author{xxx}
\date{}

\begin{document}
\frontmatter

\maketitle

\tableofcontents

\mainmatter
\chapter{The beginning}
adfadsf

\includegraphics[width=6cm]{logo}

adsfadf
\newpage
bla bla

\chapter{The second}
in the meanwhile

\chapter{The end}
bye bye

\appendix

\backmatter
\end{document}
Hmm, ik zie hier geen \begin{figure} en \end{figure}, probeer die eens toe te voegen.
Enne, het is handig om je LaTeX tekst in [ code] tags te zetten

Edit:
Heb net ff je tex document door TeXmaker gegooid, je moet het volgende pas na \begin{document} plaatsen:
1
2
3
4
5
6
7
\fancyhf{} 
\fancypagestyle{plain} 
\fancyhead[R]{\bfseries Txxx} 
%\fancyhead[L]{\includegraphics[width=3cm]{logo}}

\fancyfoot[R]{\thepage} 
\fancyfoot[L]{CONFIDENTIAL} 


[ Bericht 13% gewijzigd door AlchemistOfSorrows op 19-02-2011 22:54:59 ]
das einzige erleuchtenden an religion ist ein brennende tempel bei nacht
pi_93054600
helaas, ik krijg het nog steeds niet voor elkaar.
Zelfs in een heel simpel bestandje niet:

1
2
3
4
5
6
7
8
9
10
11
12
\documentclass[11pt,a4paper,oneside]{book}

\usepackage{graphicx}                
\graphicspath{{./Users/xxx/Documents/Afstuderen/Thesis/figuren algemeen/}}

\begin{document}

\begin{figure}
\includegraphics[width=6cm]{xxx} 
\end{figure}

\end{document}

en de log:
SPOILER
Om spoilers te kunnen lezen moet je zijn ingelogd. Je moet je daarvoor eerst gratis Registreren. Ook kun je spoilers niet lezen als je een ban hebt.
't gebeurt in Texshop op de mac met pdftex als compiler (zeg ik dat goed...)
pi_93057562
quote:
1s.gif Op zondag 20 februari 2011 14:01 schreef klaaskootstra het volgende:
helaas, ik krijg het nog steeds niet voor elkaar.
Zelfs in een heel simpel bestandje niet:

[ code verwijderd ]

en de log:
SPOILER
Om spoilers te kunnen lezen moet je zijn ingelogd. Je moet je daarvoor eerst gratis Registreren. Ook kun je spoilers niet lezen als je een ban hebt.
't gebeurt in Texshop op de mac met pdftex als compiler (zeg ik dat goed...)
Ah, watvoor plaatje gebruik je? Als je .png of .jpg gebruikt, moet je pdflatex draaien, niet pdftex.
das einzige erleuchtenden an religion ist ein brennende tempel bei nacht
pi_93092560
Ik gebruik een jpg plaatje. Maar ook wanneer ik dit plaatje als pdf probeer in te voegen krijg ik dezelfde foutmelding.

Hoe kan ik pdflatex ipv pdftex gebruiken als compiler?
  maandag 21 februari 2011 @ 10:22:07 #53
46802 Visitor.Q
Neemt alles op!
pi_93092992
Over het algemeen zou ik ook geen spaties en underscores in bestandsnamen/pad gebruiken, ik heb daar weleens wat mis mee zien gaan en het is makkelijk te omzeilen.
pi_93093174
Nou, ik 'm opgelost...

- er was inderdaad een probleem met de spatie in het pad
- voor mac gebruikers het pad zo specificeren
1\graphicspath{{/Users/xxx/Documents/Afstuderen/Thesis/figuren/}}

texshop compiler instellen op latex, maar dit is de standaard
pi_93328997
Ik heb een bizar probleem gezien bij een collega.
Zijn document is in de "book" stijl, en hij maakt gebruikt van een index en hyperref.
Als hij op pagina 1 een term introduceert, dan zal bij de index inderdaad "1" staan, maar als je erop klikt, kom je uit op een eerdere pagina (de inhoudsopgave of zo). Er is dus op een foute manier geteld.

Op mijn laptop gebeurt dat niet, maar wel bij die van nog een andere collega.

Al onze computers zijn echter Macbooks, en we gebruiken allemaal Texshop. Aan de updates kan het ook niet echt liggen, want de mijne is noch de oudste, noch de recentste.

Heeft iemand ook maar enig idee wat hier de boosdoener zou kunnen zijn?
pi_93338230
quote:
1s.gif Op vrijdag 25 februari 2011 22:44 schreef zuiderbuur het volgende:
Ik heb een bizar probleem gezien bij een collega.
Zijn document is in de "book" stijl, en hij maakt gebruikt van een index en hyperref.
Als hij op pagina 1 een term introduceert, dan zal bij de index inderdaad "1" staan, maar als je erop klikt, kom je uit op een eerdere pagina (de inhoudsopgave of zo). Er is dus op een foute manier geteld.

Op mijn laptop gebeurt dat niet, maar wel bij die van nog een andere collega.

Al onze computers zijn echter Macbooks, en we gebruiken allemaal Texshop. Aan de updates kan het ook niet echt liggen, want de mijne is noch de oudste, noch de recentste.

Heeft iemand ook maar enig idee wat hier de boosdoener zou kunnen zijn?
Probeer eens meerdere keren pdf(la)tex te draaien.
Het kan zijn dat de inhoudsopgave e.d. nog niet up-to-date zijn, en door meerdere keren het tex bestanden te compileren, wordt het gefixt.
das einzige erleuchtenden an religion ist ein brennende tempel bei nacht
pi_93344717
Ook wil het wel eens helpen alle .aux files enzo een x te verwijderen. Sommige latex editors hebben een of andere cleanup functie hiervoor.
"AAAAAHH ZENNE MOAT, WOARST VLEISCH"
pi_93347193
quote:
1s.gif Op zaterdag 26 februari 2011 02:54 schreef AlchemistOfSorrows het volgende:

[..]

Probeer eens meerdere keren pdf(la)tex te draaien.
Het kan zijn dat de inhoudsopgave e.d. nog niet up-to-date zijn, en door meerdere keren het tex bestanden te compileren, wordt het gefixt.
Dat en ook het verwijderen van de aux files is geprobeerd. We hebben werkelijk de hele handel met zo weinig mogelijk files in een aparte folder op zijn laptop en op de mijne geplaatst, en bij iedere test, hoe vaak we ook compileren, is de hyperlink incorrect bij hem niet, en correct bij mij. Kan dit aan iets anders liggen dan de versie van het programma Texshop? Het moet niet veel gekker worden.... :{
pi_93347549
quote:
1s.gif Op zaterdag 26 februari 2011 14:15 schreef zuiderbuur het volgende:

[..]

Dat en ook het verwijderen van de aux files is geprobeerd. We hebben werkelijk de hele handel met zo weinig mogelijk files in een aparte folder op zijn laptop en op de mijne geplaatst, en bij iedere test, hoe vaak we ook compileren, is de hyperlink incorrect bij hem niet, en correct bij mij. Kan dit aan iets anders liggen dan de versie van het programma Texshop? Het moet niet veel gekker worden.... :{
Ik heb ooit iets soortgelijks gehad met de memoir class.Weet alleen niet meer hoe ik het toen opgelost heb xD
"AAAAAHH ZENNE MOAT, WOARST VLEISCH"
pi_93381896
Is het mogelijk om in documentclass article de tabelnummering (en formulenummering) opnieuw te laten beginnen in de appendix? Zodat je eerste tabel in de appendix dus niet tabel 8 (doorgenummerd) wordt, maar tabel A.1. Ik weet dat het wel mogelijk is in class report, maar ik vraag me af of het ook mogelijk is in class article.
The biggest argument against democracy is a five minute discussion with the average voter.
  maandag 28 februari 2011 @ 21:04:19 #61
30719 keesjeislief
NextGenerationHippie
pi_93460879
Ik kan het quote-knopje niet meer vinden, en ik heb ook geen smileys meer?

Heb je \setcounter{equation}{0} geprobeerd aan het begin van de appendix? Dit reset in ieder geval de vergelijkingenteller, ik weet zo direct niet of het ook voor tabellen werkt.
heeft de hoop dat het allemaal stiekum toch nog goed komt...
Fotoboek
pi_93462115
quote:
1s.gif Op maandag 28 februari 2011 21:04 schreef keesjeislief het volgende:
Ik kan het quote-knopje niet meer vinden, en ik heb ook geen smileys meer?

Heb je \setcounter{equation}{0} geprobeerd aan het begin van de appendix? Dit reset in ieder geval de vergelijkingenteller, ik weet zo direct niet of het ook voor tabellen werkt.
Thanks! Het werkt ook voor tabellen. ^O^ Maar het is dus niet mogelijk dit automatisch te doen in class article?
The biggest argument against democracy is a five minute discussion with the average voter.
  maandag 28 februari 2011 @ 23:46:46 #63
30719 keesjeislief
NextGenerationHippie
pi_93470989
Heb je \numberwithin{equation}{section} geprobeerd? Als ik het me goed herinner produceert dat niet alleen nummering van de vorm section.eqnumber maar reset die ook automatisch aan het begin van elke section.
heeft de hoop dat het allemaal stiekum toch nog goed komt...
Fotoboek
pi_93477827
Hallo,

veronderstel dat ik op heel wat plaatsen in een document dezelfde figuur wil gebruiken.

Is er een manier om het pad naar die figuur in het begin te declareren, zodat ik bij een wijziging niet overal alles moet wijzigen?

Ik weet wel dat het mogelijk is om gewoon een stuk tekst te declareren:

1
2
3
4
5
6
\documentclass{article}
\newcommand{\tel}[1]{\newcommand{\ourtel}{{#1}}}
\tel{555}
\begin{document}
\ourtel is ons nummer.
\end{document}
maar hoe kan ik dit doen met een pad naar een bestand?
Bedankt!
pi_93498861
quote:
1s.gif Op maandag 28 februari 2011 23:46 schreef keesjeislief het volgende:
Heb je \numberwithin{equation}{section} geprobeerd? Als ik het me goed herinner produceert dat niet alleen nummering van de vorm section.eqnumber maar reset die ook automatisch aan het begin van elke section.
Werkt perfect, ook voor tabellen. ^O^
The biggest argument against democracy is a five minute discussion with the average voter.
  maandag 7 maart 2011 @ 15:34:33 #66
22818 V2
Deze ruimte is te huur!
pi_93772000
quote:
1s.gif Op dinsdag 1 maart 2011 08:02 schreef zuiderbuur het volgende:
Hallo,

veronderstel dat ik op heel wat plaatsen in een document dezelfde figuur wil gebruiken.

Is er een manier om het pad naar die figuur in het begin te declareren, zodat ik bij een wijziging niet overal alles moet wijzigen?
Kan je niet gewoon
1
2
3
\newcommand{\mooiplaatje}{\includegraphics{plaatje.pdf}}
...
\begin{figure}{tb} \mooiplaatje \caption{ZOMG wat een mooi plaatje} \end{figure}
doen?
As far as we know, our computer has never had an undetected error.
pi_94100013
Hallo,

ik sukkel nog steeds met figuren. Ik heb een png-afbeelding die echt in eps moet omgezet zijn.
Nu heb ik gehoord dat Texshop dit kan doen. Ik slaag er echter niet in om, eens de png geopend is met Texshop, op correcte wijze de figuur op te slaan als eps.

Weet iemand raad?
  dinsdag 15 maart 2011 @ 19:16:52 #68
22818 V2
Deze ruimte is te huur!
pi_94154458
PNG naar EPS kan je denk ik het beste doen met een image editor, zoals the Gimp. Zelf doe ik het meestal op de commandline met ImageMagick, maar da's niet de meest gebruikersvriendelijke manier. Overigens, hoezo moet het persé EPS worden? Als je pdflatex gebruikt dan kan je gewoon PNGs gebruiken in je document.
As far as we know, our computer has never had an undetected error.
pi_94183530
quote:
1s.gif Op dinsdag 15 maart 2011 19:16 schreef V2 het volgende:
PNG naar EPS kan je denk ik het beste doen met een image editor, zoals the Gimp. Zelf doe ik het meestal op de commandline met ImageMagick, maar da's niet de meest gebruikersvriendelijke manier. Overigens, hoezo moet het persé EPS worden? Als je pdflatex gebruikt dan kan je gewoon PNGs gebruiken in je document.
Bedankt voor je antwoord. Gimp kende ik niet, maar blijkbaar is het gratis beschikbaar voor Mac?

Ik heb het ondertussen gevonden: in TexShop kan je een Png openen, en dan in het menu :
Preview->"Save selection to file..." waar je dan vreemd genoeg eerst alles moet selecteren en dan kan opslaan als EPS.

Het moet echt een EPS-afbeelding zijn, ik kan niet "Pdftex" gebruiken, dit document gebruikt een template (met landscape) en zo dat blijkbaar alleen met "Tex and DVI" wil compileren.
pi_94240789
Ik vrees dat ik een nieuw probleempje heb....
ik wil een inhoudsopgave die voorafgegaan wordt door bijvoorbeeld een Preface en een Acknowledgements. Toch zou ik willen dat opgenomen worden in de Table of Contents.

Nu lukt dat wel, maar het paginanummer dat dan verschijnt is enkel "bijna" juist, het zit er eentje naast of zo.
Dit is een minimaal voorbeeld. Ik heb geen idee waarom hij dit doet?

1
2
3
4
5
6
7
8
9
10
 \documentclass[a4paper,11pt]{book}
\begin{document}

\addcontentsline{toc}{chapter}{Preface}
\include{PrefaceChapter}
\addcontentsline{toc}{chapter}{Acknowledgements}
\include{AckChapter}

\tableofcontents
\end{document}
pi_94240975
Wat gebeurt er als je \addcontentsline{toc}{chapter}{BLABLA} direct na \chapter{BLABLA} invoert in plaats van ervoor?
pi_94241954
quote:
5s.gif Op donderdag 17 maart 2011 15:37 schreef thabit het volgende:
Wat gebeurt er als je \addcontentsline{toc}{chapter}{BLABLA} direct na \chapter{BLABLA} invoert in plaats van ervoor?
Dan wordt het nog erger.
Hij geeft als paginanummers 4 en 6, terwijl hij 1 en 5 zou moeten tonen!
pi_94243704
quote:
1s.gif Op donderdag 17 maart 2011 15:58 schreef zuiderbuur het volgende:

[..]

Dan wordt het nog erger.
Hij geeft als paginanummers 4 en 6, terwijl hij 1 en 5 zou moeten tonen!
Niet na de \include, maar direct na \chapter, in de file die je includet dus. Of had je dat zo gedaan?
  donderdag 17 maart 2011 @ 17:22:39 #74
22818 V2
Deze ruimte is te huur!
pi_94245920
\addcontentsline moet direct na \chapter, anders kom je in de problemen als er een \clear(double)page in \chapter zit. Verder moet je latex minstens 2 keer runnen voordat de paginanummers kloppen.
As far as we know, our computer has never had an undetected error.
pi_94251507
quote:
1s.gif Op donderdag 17 maart 2011 16:32 schreef thabit het volgende:

[..]

Niet na de \include, maar direct na \chapter, in de file die je includet dus. Of had je dat zo gedaan?
Aahh.. je bedoelt in die files zelf! Dat werkt helemaal perfect blijkbaar! *O* _O_
pi_94292378
Ik wilde net eens kijken of het misschien makkelijk was om pdflatex in plaats van latex te gebruiken. Dit omdat ik dan mijn jpg's niet meer hoef te converteren naar eps. Het viel me op dat de tekst met pdflatex vaag/lelijk wordt. Screenshot ter illustratie:

http://img819.imageshack.us/i/testuz.png/

Tijdens conversie kreeg ik alleen wel: Warning: pdflatex (file pdftex.map): cannot open font map file. Gaat er bij mij iets mis, of ziet pdflatex er altijd wat minder mooi uit?
  vrijdag 18 maart 2011 @ 15:14:02 #77
148823 znarch
ondertitel
pi_94292601
Welk font gebruik je?
"AAAAAHH ZENNE MOAT, WOARST VLEISCH"
pi_94293020
Uhm... gewoon de standaard font denk ik? Dit is de tex file:

1
2
3
4
5
6
7
8
9
10
11
12
\documentclass[a4paper,10pt]{article}

\title{LateX vs PDFLatex}

\begin{document}
\maketitle

PDFLatex \\ 

Lorem ipsum dolor sit ..... bla bla ....

\end{document}
  vrijdag 18 maart 2011 @ 16:35:17 #79
22818 V2
Deze ruimte is te huur!
pi_94296122
quote:
1s.gif Op vrijdag 18 maart 2011 15:08 schreef Toryu het volgende:
Tijdens conversie kreeg ik alleen wel: Warning: pdflatex (file pdftex.map): cannot open font map file. Gaat er bij mij iets mis, of ziet pdflatex er altijd wat minder mooi uit?
Ik denk dat het probleem hier zit, door het niet kunnen vinden van de map-file zal hij wel bitmap-fonts gaan gebruiken. Wat zegt kpsewhich pdftex.map (ws niks, omdat hij 'm niet kan vinden) en helpt het om updmap en/of updmap-sys en/of mktexlsr of iets van die geest te runnen?
As far as we know, our computer has never had an undetected error.
pi_94433107
quote:
1s.gif Op donderdag 17 maart 2011 16:32 schreef thabit het volgende:

[..]

Niet na de \include, maar direct na \chapter, in de file die je includet dus. Of had je dat zo gedaan?
Ik heb hier nog een vraagje over.
Ik zie dat nu dat hij in mijn "Preface" die nog bij de "\frontmatter" staat, de hoofding verkeerd schrijft.
Ik zou willen dat er daar Romeinse cijfers, gevolgd door "Preface" komt te staan.
Ik gebruik deze code voor de frontmatter maar dat helpt niet, hij schrijft enkel een Romeins cijfer voor het paginanummer, geen "Preface".

1
2
3
4
\fancyhead[LE]{\mdseries \thepage~$\mid$}
\fancyhead[RE]{ \nouppercase{\leftmark}}
\fancyhead[RO]{\mdseries $\mid$ \thepage}
\fancyhead[LO]{\mdseries \nouppercase{\rightmark}  }

Mijn Preface is een file die begint met :

1\chapter*{Preface}\label{prefacechapter}\addcontentsline{toc}{chapter}{Preface}

Ik vermoed dat de boosdoener die \chapter* is. De "leftmark" en "rightmark" geraken hierdoor blijkbaar fout ingesteld.
Weet iemand raad?
pi_95369554
Ik gebruik voor een scriptie op een paar plekken de command \emph want ik heb ooit gehoord dat dat 'beter' is dan \textit, maar nu heb ik de boel in Garamond font gezet, en dan zie ik ineens niets meer speciaals aan de tekst die benadrukt (of voor mij gewoon schuingezet) moest worden. Hoe kan dat?
pi_95370308
Werkt het met \textit wel?
pi_95372099
Nope, ook niet!
Ik ga dit proberen \usepackage[urw-garamond]{mathdesign} maar packages downloaden duurt altyd wel even, dus oplossingen zijn welkom
pi_95372828
Doe eens een minimum example posten waar het misgaat.
pi_95375548
Hmmmm ik weet het niet zeker maar ik geloof dat "\emph" is voor wiskundige vergelijkingen dus gebruiken binnen \begin{equation}\end{equation}, $$, etc... environmenten, \textit gebruik je dus voor tekst.
  dinsdag 12 april 2011 @ 10:47:34 #86
22818 V2
Deze ruimte is te huur!
pi_95382190
Het verschil tussen \emph en \textit merk je pas als je gaat nesten: \emph in \emph geeft weer tekst die rechtop staat, bij \textit gebeurt dit niet. Verder is het geloof ik zo dat \emph op het einde automatisch italic correction toepast (\/), \textit doet dit volgens mij niet, maar dit weet ik niet 100% zeker.
As far as we know, our computer has never had an undetected error.
pi_95392735
1
2
3
4
5
6
7
8
9
10
11
12
13
14
15
16
17
18
19
20
21
22
23
24
25
\documentclass[12pt]{article}
\usepackage{amsmath}
\usepackage{amsfonts}
\usepackage{amssymb}
\usepackage{multirow}
\usepackage{url}
\usepackage{garamond} 
\usepackage{mathrsfs}
\usepackage{enumerate}

\makeatletter
\def\url@leostyle{%
  \@ifundefined{selectfont}{\def\UrlFont{\sf}}{\def\UrlFont{\small\ttfamily}}}
\makeatother
\urlstyle{leo}

\title{Bla}
\date{}
\usepackage[dutch]{babel}
\begin{document}
\maketitle
\section{Introductie}

Voor een imputatie $x$ en een coalitie $S$ in een spel $(N,v)$, is de \emph{excess} ...
\end{document}

Dit is de relevante code. Zonder Garamond doet ie wel netjes het woord excess schuingedrukt. Maar met Garamond dus niet...
Overigens werkte de '\usepackage[urw-garamond]{mathdesign}' oplossing niet, dus ik houd me aanbevolen voor beter advies.
  dinsdag 12 april 2011 @ 16:01:00 #88
22818 V2
Deze ruimte is te huur!
pi_95396893
Lastig om je hiermee te helpen, want ik heb URW Garamond niet, dus werkt mathdesign niet met garamond. Package garamond wat jij gebruikt heb ik ook niet, en zit ook niet in m'n package manager.

Edit: OK, met 'getnonfreefonts-sys garamond' blijk je onder Ubuntu heel simpel dat font te kunnen installeren, en daarmee werkt mathdesign prima. Ook \emph werkt hiermee gewoon.

[ Bericht 32% gewijzigd door V2 op 12-04-2011 16:06:46 ]
As far as we know, our computer has never had an undetected error.
pi_95501500
Hallo,

ik ben opnieuw aan het werken in Beamer, en ik gebruik

\useoutertheme{smoothbars}

Best overzichtelijk, alleen moet er daar een "institute" bij de auteur gegeven worden, dan krijg je helemaal linksonder op elke slide:

zuiderbuur (een instituut)

Maar ik wil helemaal geen instituut! Dat kan ik echter niet uitschakelen, er blijft zuiderbuur () staan als ik dat probeer.

Weet iemand raad?

Dank je!
pi_95518864
Ik moet een verslag schrijven, dus gebruik ik \documentclass{report}, alleen m'n hoofdstukken zijn niet zo enorm lang, dus vind ik \section te grote koppen geven. Is er een manier om \section koppen te geven die zo groot zijn als die van subsectie's (of \subsection de nummering van sectie's te geven)?
pi_95519499
quote:
1s.gif Op donderdag 14 april 2011 22:39 schreef Hanneke12345 het volgende:
Ik moet een verslag schrijven, dus gebruik ik \documentclass{report}, alleen m'n hoofdstukken zijn niet zo enorm lang, dus vind ik \section te grote koppen geven. Is er een manier om \section koppen te geven die zo groot zijn als die van subsectie's (of \subsection de nummering van sectie's te geven)?
Ik gebruik altijd dit wat wel een mooi effect geeft imo.

1
2
3
4
5
6
7
8
9
\usepackage{titlesec}
\usepackage{listings}
\lstset{
    breaklines     = true,
    numbers        = left,
    stepnumber     = 1,
}
\setlength{\parskip}{1ex}
\setlength{\parindent}{0mm}
pi_95616864
Ik zie geen verschil ;o
pi_95689314
Hallo,

zit met het volgende te knoeien...

Heb een symbolen lijst gemaakt in een apart bestand, volgens de volgende code:

1
2
3
4
5
6
7
\newglossaryentry{loa}{
    type=symbols,
    name={Length overall},        
    symbol={L\subscript{oa}},
    sort={loa},        
    description={Length overall}, 
    user1={\metre}}

In mijn hoofddocument heb ik een nieuwe glossary gemaakt voor de symbolen.
Deze wil op de volgende manier gepresenteerd hebben:

symbool, eenheid, description, paginanummer, waarbij het pagina nummer aansluit op de description, deze hoeft dus niet in een aparte kolom. Waarbij de lijst gesorteerd wordt op basis van de "sort"

Ik probeer het op de volgende manier voor elkaar te krijgen,

1
2
3
4
5
6
7
8
9
10
11
12
13
14
15
16
17
18
19
20
21
\usepackage[acronym,toc,sanitize={symbol=false}]{glossaries} % loading acronym option into table of contents, allowing math in descriptions, defining style
    \newglossary[slg]{symbols}{sym}{sbl}{List of Symbols} % creating a seperate list for symbols

\newglossarystyle{symbollist}{% 

\glossarystyle{long3col}% base this style on the list style 
\renewcommand{\glsgroupskip}{}% make nothing happen between groups 


% Set the table’s header: 
\renewcommand*{\glossaryheader}{% 
\bfseries Symbol & \bfseries Unit & \bfseries Description 
\\\endhead}% 

% Main (level 0) entries displayed in a row: 
\renewcommand*{\glossaryentryfield}[5]{% 
\glstarget{##1}{##4}% Name 
& \glsentryuseri{##1}% Units 
& ##3% Description 
\space ##5% Page list 
\\% end of row 

Echter, tot nu zonder succes. Heeft iemand hier een mooie oplossing voor?
pi_96595401

Met welk pakket maak ik het best zulke plaatjes?
xypic?
  dinsdag 10 mei 2011 @ 13:46:44 #95
46802 Visitor.Q
Neemt alles op!
pi_96595609
@TheLoneGunmen: Ik denk met TikZ
pi_96595907
Thanks, was ik al bang voor. Heb geen zin om weer iets nieuws te moeten leren... 't is een vrij simpele afbeelding dus hopelijk kan het eenvoudiger...
pi_96597376
Mwuah dit is niet met Tikz gemaakt geloof ik, zie de font van de cijfers. Denk dat dit gewoon is gemaakt met een vector editor zoals Inkscape.
pi_96598422
Ik deel de mening van Dale. Je zou evt. LaTeXDraw kunnen overwegen, die produceert PSTricks-code. Wat vervolgens weer een hoop gedoe kan opleveren afh. van je latexprocessor.
pi_96600759
Danku Q. I did it with tikz...


Code:
SPOILER
Om spoilers te kunnen lezen moet je zijn ingelogd. Je moet je daarvoor eerst gratis Registreren. Ook kun je spoilers niet lezen als je een ban hebt.


[ Bericht 49% gewijzigd door TheLoneGunmen op 10-05-2011 16:34:21 ]
pi_97552529
Ik ben bijna klaar met mijn proefschrift, maar loop nu tegen een groot probleem aan: tijdens het builden van de file (in Texnic Center, Latex -> PS en dan Acrobat Distiller om er een PDF van te maken) zet de Distiller:

%%[ Warning: AvantGarde-Demi not found, using Font Substitution. Font cannot be embedded. ]%%

Op zich ziet de PDF er prima uit, maara het is een probleem voor de drukker.

Via Google kwam ik op een ander output profile (Latex->PS->PDF), maar ook dat geeft problemen:

Trying to make PF font (...) at 7000 DPI
miktex-makepk: Specified BDPI doesn't match ljfour resolution (600)


en iets verder:

dvips.exe: Font (...) not found, characters will be left blank

In de style-file staat hetvolgende voor fonts:

%Fonts
\usepackage{pxfonts}
\usepackage{type1cm}
\usepackage{fixltx2e}

%Computer modern bright
\usepackage[T1]{fontenc}
\usepackage{cmbright}
\usepackage{eurosym}


Kan iemand me uit de brand helpen?
  woensdag 1 juni 2011 @ 19:42:40 #101
30719 keesjeislief
NextGenerationHippie
pi_97592911
quote:
0s.gif Op dinsdag 31 mei 2011 19:22 schreef KaatjeJ het volgende:
Ik ben bijna klaar met mijn proefschrift, maar loop nu tegen een groot probleem aan: tijdens het builden van de file (in Texnic Center, Latex -> PS en dan Acrobat Distiller om er een PDF van te maken) zet de Distiller:

%%[ Warning: AvantGarde-Demi not found, using Font Substitution. Font cannot be embedded. ]%%

Op zich ziet de PDF er prima uit, maara het is een probleem voor de drukker.

Via Google kwam ik op een ander output profile (Latex->PS->PDF), maar ook dat geeft problemen:

Trying to make PF font (...) at 7000 DPI
miktex-makepk: Specified BDPI doesn't match ljfour resolution (600)


en iets verder:

dvips.exe: Font (...) not found, characters will be left blank

In de style-file staat hetvolgende voor fonts:

%Fonts
\usepackage{pxfonts}
\usepackage{type1cm}
\usepackage{fixltx2e}

%Computer modern bright
\usepackage[T1]{fontenc}
\usepackage{cmbright}
\usepackage{eurosym}


Kan iemand me uit de brand helpen?
Je zou kunnen proberen om de cm-super fonts te installeren, of de lmodern fonts te gebruiken via \usepackage{lmodern}.
heeft de hoop dat het allemaal stiekum toch nog goed komt...
Fotoboek
  zondag 5 juni 2011 @ 17:07:42 #102
148823 znarch
ondertitel
pi_97743286
Als ik in mijn text iets tussen aanhalingstekens zet op deze manier: "blabla" plaatst hij geen spatie na het woord. Weet iemand hoe ik dit wel simpel voor elkaar krijg?
"AAAAAHH ZENNE MOAT, WOARST VLEISCH"
pi_97743448
quote:
0s.gif Op zondag 5 juni 2011 17:07 schreef znarch het volgende:
Als ik in mijn text iets tussen aanhalingstekens zet op deze manier: "blabla" plaatst hij geen spatie na het woord. Weet iemand hoe ik dit wel simpel voor elkaar krijg?
Probeer eens:
1``blabla''
  zondag 5 juni 2011 @ 17:14:01 #104
148823 znarch
ondertitel
pi_97743607
quote:
0s.gif Op zondag 5 juni 2011 17:10 schreef thabit het volgende:

[..]

Probeer eens:
[ code verwijderd ]

Thx, dat was hem! Weet je toevallig ook hoe je hetzelfde probleem oplost na gebruik van het \euro symbool? Je kunt het oplossen met een \phantom{} maar das zo slordig.
"AAAAAHH ZENNE MOAT, WOARST VLEISCH"
pi_97743744
quote:
0s.gif Op zondag 5 juni 2011 17:14 schreef znarch het volgende:

[..]

Thx, dat was hem! Weet je toevallig ook hoe je hetzelfde probleem oplost na gebruik van het \euro symbool? Je kunt het oplossen met een \phantom{} maar das zo slordig.
Je kunt altijd met backslash-spatie een spatie invoegen.
pi_97746391
quote:
0s.gif Op zondag 5 juni 2011 17:16 schreef thabit het volgende:

[..]

Je kunt altijd met backslash-spatie een spatie invoegen.
Ja, of met een ~ (tilde) Dat is een non-breakable space. Bijvoorbeeld: 's~ochtends
  zondag 5 juni 2011 @ 18:24:55 #107
148823 znarch
ondertitel
pi_97746842
Thx, ondanks dat ik al best lang met latex pruts had ik van beide opties nog nooit gehoord (of niet onthouden).
"AAAAAHH ZENNE MOAT, WOARST VLEISCH"
pi_97782203
Weet iemand van een brief style? Ik las dat de nederlandse latex user group een eigen letter style heeft gemaakt.
pi_97782466
Het is een standaard documentclass in latex. Zie hier voor uitleg en voorbeeld:
http://en.wikibooks.org/wiki/LaTeX/Letters

Of bedoeld je wat anders?
pi_97784612
quote:
0s.gif Op maandag 6 juni 2011 13:13 schreef Toryu het volgende:
Het is een standaard documentclass in latex. Zie hier voor uitleg en voorbeeld:
http://en.wikibooks.org/wiki/LaTeX/Letters

Of bedoeld je wat anders?
Ik weet dat er letter is maar standaard is dit in de Amerikaanse stijl. Ik heb inmiddels een pakket gevonden die een brief documentclass heeft, ntgclass. Aangepast aan de Nderlandse stijl. Alleen dit is lichtelijk gedateerd heb ik het idee.
pi_97786865
Had je ook niet een brief2 oid? Ik heb er weleens naar gekeken, maar vond 't allemaal maar niks.
pi_97838937
Een vraagje over de subfiles-package. Ik heb deze uitleg al gevonden, maar als ik dat volg kreeg ik steeds deze foutmelding bij het compileren van de mainfile:

1
2
3
4
5
6
7
8
! LaTeX Error: Can be used only in preamble.

See the LaTeX manual or LaTeX Companion for explanation.
Type  H <return>  for immediate help.
 ...                                              
                                                  
l.4 \usepackage
               {a4wide}

Als ik alleen een stukje tekst in de main file typ is er geen probleem, maar als ik een subfile wil invoegen:

1
2
3
4
5
\begin{document}

\subfile{HOOFDSTUK1.tex}

\end{document}

krijg ik dus bovenstaande foutmelding. Iemand enig idee waar het aan kan liggen?
The biggest argument against democracy is a five minute discussion with the average voter.
pi_97839474
quote:
2s.gif Op dinsdag 7 juni 2011 14:41 schreef M.rak het volgende:
Een vraagje over de subfiles-package. Ik heb deze uitleg al gevonden, maar als ik dat volg kreeg ik steeds deze foutmelding bij het compileren van de mainfile:

[ code verwijderd ]

Als ik alleen een stukje tekst in de main file typ is er geen probleem, maar als ik een subfile wil invoegen:

[ code verwijderd ]

krijg ik dus bovenstaande foutmelding. Iemand enig idee waar het aan kan liggen?
Waarom gebruik je niet \input{}?
pi_97839608
Hoe is HOOFDTUK1.tex opgebowd? Stel, je hebt main.tex als:

1
2
3
4
5
6
7
8
\documentclass{article} 
\usepackage{subfiles}

\begin{document}

\subfile{HOOFDSTUK1.tex}

\end{document}

Dan moet HOOFDSTUK1.tex als volgt zijn:
1
2
3
4
5
6
\documentclass[main.tex]{subfiles} 
\begin{document}

blabla...

\end{document}

Zie ook:
http://en.wikibooks.org/w(...)les#Subfiles_package

Anders kan het zijn dat het conflicteert met andere packages.
pi_97840214
quote:
7s.gif Op dinsdag 7 juni 2011 14:55 schreef Dale. het volgende:

[..]

Waarom gebruik je niet \input{}?
Omdat ik daarbij de losse files niet kan compileren ;).

quote:
0s.gif Op dinsdag 7 juni 2011 14:57 schreef Toryu het volgende:
Hoe is HOOFDTUK1.tex opgebowd? Stel, je hebt main.tex als:

[ code verwijderd ]

Dan moet HOOFDSTUK1.tex als volgt zijn:

[ code verwijderd ]

Zie ook:
http://en.wikibooks.org/w(...)les#Subfiles_package

Anders kan het zijn dat het conflicteert met andere packages.
Ik heb nu mijn file HOOFDSTUK1.tex als volgt, die kan ik ook gewoon compileren:

1
2
3
4
5
6
7
8
\documentclass[MAINFILE.tex]{subfiles}
\usepackage{subfiles}
\usepackage[dutch]{babel}
\begin{document}

tekst

\end{document}

En mijn file MAINFILE.tex zo:
1
2
3
4
5
6
7
8
\documentclass{article} 
\usepackage{subfiles}

\begin{document}

\subfile{HOOFDSTUK1.tex}

\end{document}

Als ik die mainfile echter wil compileren krijg ik de volgende foutmelding

1
2
3
4
5
6
See the LaTeX manual or LaTeX Companion for explanation.
Type  H <return>  for immediate help.
 ...                                              
                                                  
l.2 \usepackage
               {subfiles}

(de reden dat ik de package babel heb geladen is omdat hij 'm anders niet wil compileren.)
The biggest argument against democracy is a five minute discussion with the average voter.
pi_97844811
Je moet \usepackage{subfiles} uit HOOFDSTUK1.tex halen, dan doet ie het. Ik denk dat sowieso beter is om alle usepackages in mainfile te zetten. Je overige .tex-bestanden nemen die dan over.
pi_97845140
Bedankt, hij doet het nu inderdaad ^O^.
The biggest argument against democracy is a five minute discussion with the average voter.
pi_97953993
Ik heb nog een probleempje, met bovenstaande subfile-package. Ik heb twee appendices in mijn document staan, één met tabellen en een met Matlab-code. Deze staan in m'n mainfile als volgt onder elkaar:

1
2
3
4
5
6
%Appendix Tabellen
\subfile{AppendixTabellen.tex}
\newpage

%Code
\subfile{Code.tex}

Als ik echter het hele verslag compileer, loopt de code tussen de tabellen door. Iemand een idee hoe ik dit op kan lossen? Wat er dus eigenlijk moet gebeuren, is zorgen dat de code pas begint ná alle tabellen, op een nieuwe pagina.
The biggest argument against democracy is a five minute discussion with the average voter.
  donderdag 9 juni 2011 @ 22:07:22 #119
30719 keesjeislief
NextGenerationHippie
pi_97957808
quote:
0s.gif Op donderdag 9 juni 2011 21:03 schreef M.rak het volgende:
Ik heb nog een probleempje, met bovenstaande subfile-package. Ik heb twee appendices in mijn document staan, één met tabellen en een met Matlab-code. Deze staan in m'n mainfile als volgt onder elkaar:

[ code verwijderd ]

Als ik echter het hele verslag compileer, loopt de code tussen de tabellen door. Iemand een idee hoe ik dit op kan lossen? Wat er dus eigenlijk moet gebeuren, is zorgen dat de code pas begint ná alle tabellen, op een nieuwe pagina.
Je kunt

1
2
\clearpage
\pagebreak

gebruiken, de eerste zorgt ervoor dat alle 'floats' voor dat punt geplaatst worden.
heeft de hoop dat het allemaal stiekum toch nog goed komt...
Fotoboek
pi_97958598
quote:
0s.gif Op donderdag 9 juni 2011 22:07 schreef keesjeislief het volgende:

[..]

Je kunt

[ code verwijderd ]

gebruiken, de eerste zorgt ervoor dat alle 'floats' voor dat punt geplaatst worden.
Thanks ^O^.
The biggest argument against democracy is a five minute discussion with the average voter.
  zondag 3 juli 2011 @ 13:31:27 #121
148507 Schuifpui
Lone Star
pi_98978842
Blijkbaar is er ook een LaTeX topic, dat wist ik niet. Even m'n eerdere post hier plakken:

quote:
Voor m'n afstudeerverslag moet ik een aantal pdf plaatjes naar eps omzetten om ze in LaTeX te kunnen gebruiken. Ik heb al adobe acrobat 9 geprobeerd, maar de kwaliteit is niet optimaal. Ik heb bijvoorbeeld een plaatje met een achtergrond kleur, maar rond bepaalde objecten op het plaatje is de achtergrondkleur net wat anders. Er zijn ook van die sites waar je een pdf kan uploaden en je een eps terugkrijgt, maar die deden het helemaal niet. Ik wil m'n plaatjes zoveel mogelijk gewoon in vectorformat houden. Iemand een idee hoe ik dit het beste kan doen?
De pdf direct invoegen als plaatje werkt niet dan krijg ik de volgende melding:

quote:
cannot determine the size of the graphic in c:\... (no BoundingBox)
Iemand een idee? Ik heb de pdf overigens gecropped met adobe acrobat 9. Het was eerst een volledige A4 met een plaatje op ongeveer een halve pagina. Dat witte moet er uiteraard niet omheen blijven.
pi_99099886
Hallo

ik gebruik opnieuw "beamer" om slideshows in Latex te maken. Ik zou graag loskomen van klassieke thema's als Warsaw en CambridgeUS, en meer zelf voorkeuren ingeven.
Maar ik geraak een beetje in de war.

Ik heb nu gebruikt:
1
2
3
4
5
6
 \usetheme{CambridgeUS}
\useinnertheme{rectangles} 
\usecolortheme{seahorse}
\useoutertheme[subsection=false]{smoothbars}
\setbeamercolor{inner theme}{fg=blue}
\setbeamertemplate{blocks}[default]
maar mijn "blocks" zijn nu op geen enkele manier aangeduid. Stel dat ik zou willen dat het een doosje wordt, met een titel in een apart kleiner doosje.

\setbeamertemplate{blocks}[rounded]
lijkt daar iets aan te doen, maar wat zijn de andere opties? Of moet ik dit toch bij \useinnertheme zelf aanpassen?

Met veel dank!
  woensdag 6 juli 2011 @ 10:54:48 #123
46802 Visitor.Q
Neemt alles op!
pi_99103042
quote:
0s.gif Op zondag 3 juli 2011 13:31 schreef Schuifpui het volgende:
Blijkbaar is er ook een LaTeX topic, dat wist ik niet. Even m'n eerdere post hier plakken:

[..]

De pdf direct invoegen als plaatje werkt niet dan krijg ik de volgende melding:

[..]

Iemand een idee? Ik heb de pdf overigens gecropped met adobe acrobat 9. Het was eerst een volledige A4 met een plaatje op ongeveer een halve pagina. Dat witte moet er uiteraard niet omheen blijven.
Ik gebruik sinds kort XeLaTeX, die slikt allebei. Anders misschien pdf importeren in inkscape, en daar via document properties de bounding box goed zetten en saven als eps.
  woensdag 6 juli 2011 @ 10:59:03 #124
46802 Visitor.Q
Neemt alles op!
pi_99103198
quote:
11s.gif Op woensdag 6 juli 2011 08:12 schreef zuiderbuur het volgende:
Hallo

ik gebruik opnieuw "beamer" om slideshows in Latex te maken. Ik zou graag loskomen van klassieke thema's als Warsaw en CambridgeUS, en meer zelf voorkeuren ingeven.
Maar ik geraak een beetje in de war.

Ik heb nu gebruikt:
[ code verwijderd ]

maar mijn "blocks" zijn nu op geen enkele manier aangeduid. Stel dat ik zou willen dat het een doosje wordt, met een titel in een apart kleiner doosje.

\setbeamertemplate{blocks}[rounded]
lijkt daar iets aan te doen, maar wat zijn de andere opties? Of moet ik dit toch bij \useinnertheme zelf aanpassen?

Met veel dank!
Volgens mij staat dat inderdaad gedefinieerd in de style-file. Ik vond voor een poster (ook met beamer) een aardig voorbeeld waarbij de blocks worden herdefinieerd:
https://teamwork.jacobs-u(...)PandBiG/LaTeX+Poster

In de style file op die site zie je een 30-tal regels die gaan over de opbouw van een block - ziet er niet eenvoudig uit maar misschien kun je die als voorbeeld gebruiken.
pi_99106550
quote:
0s.gif Op woensdag 6 juli 2011 10:59 schreef Visitor.Q het volgende:

[..]

Volgens mij staat dat inderdaad gedefinieerd in de style-file. Ik vond voor een poster (ook met beamer) een aardig voorbeeld waarbij de blocks worden herdefinieerd:
https://teamwork.jacobs-u(...)PandBiG/LaTeX+Poster

In de style file op die site zie je een 30-tal regels die gaan over de opbouw van een block - ziet er niet eenvoudig uit maar misschien kun je die als voorbeeld gebruiken.
Dank je, maar ik denk dat dat misschien al iets te gevorderd is.
Ik wou niet in de sty-files gaan wijzigen, ik vroeg me af wat ik kon doen als ik gewoon die \setbeamertemplate{blocks} ten volle benutte?
  woensdag 6 juli 2011 @ 23:17:34 #126
30719 keesjeislief
NextGenerationHippie
pi_99134252
quote:
0s.gif Op zondag 3 juli 2011 13:31 schreef Schuifpui het volgende:
Blijkbaar is er ook een LaTeX topic, dat wist ik niet. Even m'n eerdere post hier plakken:

[..]

De pdf direct invoegen als plaatje werkt niet dan krijg ik de volgende melding:

[..]

Iemand een idee? Ik heb de pdf overigens gecropped met adobe acrobat 9. Het was eerst een volledige A4 met een plaatje op ongeveer een halve pagina. Dat witte moet er uiteraard niet omheen blijven.
Minimaal voorbeeld?

quote:
0s.gif Op woensdag 6 juli 2011 12:34 schreef zuiderbuur het volgende:

[..]

Dank je, maar ik denk dat dat misschien al iets te gevorderd is.
Ik wou niet in de sty-files gaan wijzigen, ik vroeg me af wat ik kon doen als ik gewoon die \setbeamertemplate{blocks} ten volle benutte?
Dat lijkt niet mogelijk, zie bijv. hier: http://tex.stackexchange.(...)med-blocks-in-beamer
heeft de hoop dat het allemaal stiekum toch nog goed komt...
Fotoboek
pi_99135609
quote:
0s.gif Op woensdag 6 juli 2011 23:17 schreef keesjeislief het volgende:

[..]

Minimaal voorbeeld?
In ieder geval een screenshot van het probleem met de eps:


Die lichte vlakken om de blokken (vooral duidelijk bij het blauwe) horen uiteraard niet. Dat building-up/hacking down principe maakt niet uit. Het gebeurt ook bij een leeg document met alleen het plaatje. Dit probleem lijkt puur in de EPS te zitten.

Als ik de pdf probeer, dan krijg ik die no bounding-box fout. Ik dacht eerst dat het kwam door het croppen van de pdf, maar nu net heb ik geprobeerd om andere pdf's in te voegen, maar die werken ook allemaal niet, die geven dezelfde foutmelding.
  donderdag 7 juli 2011 @ 08:51:48 #128
46802 Visitor.Q
Neemt alles op!
pi_99141626
quote:
0s.gif Op woensdag 6 juli 2011 23:47 schreef Schuifpui het volgende:

[..]

In ieder geval een screenshot van het probleem met de eps:

[ afbeelding ]
Die lichte vlakken om de blokken (vooral duidelijk bij het blauwe) horen uiteraard niet. Dat building-up/hacking down principe maakt niet uit. Het gebeurt ook bij een leeg document met alleen het plaatje. Dit probleem lijkt puur in de EPS te zitten.

Als ik de pdf probeer, dan krijg ik die no bounding-box fout. Ik dacht eerst dat het kwam door het croppen van de pdf, maar nu net heb ik geprobeerd om andere pdf's in te voegen, maar die werken ook allemaal niet, die geven dezelfde foutmelding.
Kun je de pdf/eps even ergens uploaden?
pi_99147967
quote:
0s.gif Op donderdag 7 juli 2011 08:51 schreef Visitor.Q het volgende:

[..]

Kun je de pdf/eps even ergens uploaden?
http://www.mediafire.com/?cw8mj1udakyn8a0

Ik heb de EPS en de originele PDF geupload. Dat is degene die nog niet gecropped is. Ze zitten in een .rar file.
  donderdag 7 juli 2011 @ 22:45:49 #130
30719 keesjeislief
NextGenerationHippie
pi_99175238
quote:
0s.gif Op donderdag 7 juli 2011 12:37 schreef Schuifpui het volgende:

[..]

http://www.mediafire.com/?cw8mj1udakyn8a0

Ik heb de EPS en de originele PDF geupload. Dat is degene die nog niet gecropped is. Ze zitten in een .rar file.
Bij mij werkt dit gewoon:

1
2
3
4
5
6
7
8
9
\documentclass[12pt, a4paper]{article}

\usepackage[pdftex]{graphicx}

\begin{document}

\includegraphics[width=\textwidth]{PDF.pdf}

\end{document}

Werkt dat niet bij jou?
heeft de hoop dat het allemaal stiekum toch nog goed komt...
Fotoboek
pi_99178737
Nee bij mij werkt dat niet. :{



Ik gebruik overigens TeXnicCenter met MikTeX 2.8.
  vrijdag 8 juli 2011 @ 00:37:58 #132
30719 keesjeislief
NextGenerationHippie
pi_99180980
quote:
0s.gif Op donderdag 7 juli 2011 23:47 schreef Schuifpui het volgende:
Nee bij mij werkt dat niet. :{

[ afbeelding ]

Ik gebruik overigens TeXnicCenter met MikTeX 2.8.
Compileer je wel met pdflatex?
heeft de hoop dat het allemaal stiekum toch nog goed komt...
Fotoboek
  vrijdag 8 juli 2011 @ 00:46:50 #133
148507 Schuifpui
Lone Star
pi_99181281
Eh.. Ik moet eerlijk zeggen dat ik al die instellingen niet heb gemaakt. De persoon die het voor me gedaan heeft, zit inmiddels ook niet meer in Nederland. Zit dit in build - define output profiles? Dat ziet er bij mij zo uit:

  vrijdag 8 juli 2011 @ 00:59:12 #134
30719 keesjeislief
NextGenerationHippie
pi_99181713
quote:
0s.gif Op vrijdag 8 juli 2011 00:46 schreef Schuifpui het volgende:
Eh.. Ik moet eerlijk zeggen dat ik al die instellingen niet heb gemaakt. De persoon die het voor me gedaan heeft, zit inmiddels ook niet meer in Nederland. Zit dit in build - define output profiles? Dat ziet er bij mij zo uit:

[ afbeelding ]
Daar zit inderdaad het probleem lijkt het. Latex geeft je een .eps file als output en kan niet met pdf-figuren omgaan, vandaar de foutmelding. Pdflatex produceert een .pdf file voor je en kan wel met pdf-figuren omgaan (maar weer niet met eps-figuren). Je kunt twee dingen doen:

1. Als je geen andere figuren in je document hebt, of die andere figuren zijn ook pdf, dan kun je je document compileren met pdflatex. Ik ken Texniccenter niet, maar het lijkt erop dat je of de optie "LaTeX => PDF" in het linkermenu moet nemen, of in "Make" het "Path to ..." moet aanpassen van "latex.exe" naar "pdflatex.exe" (die executable zou zich in dezelfde map als latex.exe moeten bevinden). Misschien moet je dan ook nog wat rommelen met wat er nu in "Command line ... compiler:" staat ingevuld, dat is wellicht specifiek voor latex.

2. Als er wel eps-figuren in je document voorkomen, dan moet je die of allemaal omzetten naar pdf en dan pdflatex gebruiken, of alle figuren omzetten naar eps en dan latex gebruiken. Voor zulke conversies zijn online progjes beschkbaar (maar de kwaliteit is niet altijd helemaal geweldig meen ik me te herinneren).
heeft de hoop dat het allemaal stiekum toch nog goed komt...
Fotoboek
  vrijdag 8 juli 2011 @ 01:04:25 #135
148507 Schuifpui
Lone Star
pi_99181889
Bedankt voor je uitleg. Ik heb nog heel veel andere figuren allemaal EPS. Een deel door de faculteit gegeven, voor de voorpagina e.d. dan moet ik toch een manier vinden om de paar pdf's om te zetten naar EPS zonder kwaliteitsverlies. Met adobe acrobat werkt het niet goed genoeg helaas, terwijl ik nou juist van dat programma wel zou verwachten dat het goed zou gaan.
  vrijdag 8 juli 2011 @ 01:44:30 #136
30719 keesjeislief
NextGenerationHippie
pi_99182877
quote:
0s.gif Op vrijdag 8 juli 2011 01:04 schreef Schuifpui het volgende:
Bedankt voor je uitleg. Ik heb nog heel veel andere figuren allemaal EPS. Een deel door de faculteit gegeven, voor de voorpagina e.d. dan moet ik toch een manier vinden om de paar pdf's om te zetten naar EPS zonder kwaliteitsverlies. Met adobe acrobat werkt het niet goed genoeg helaas, terwijl ik nou juist van dat programma wel zou verwachten dat het goed zou gaan.
Een andere optie is om epstopdf te gebruiken. Hiermee kun je toch pdflatex gebruiken, en worden je eps-figuren automatisch omgezet naar pdf-figuren (d.w.z. je hoeft niets met de hand te doen). Als de kwaliteit van de conversie pdf -> eps tegen blijft vallen kun je dit als alternatief proberen.
heeft de hoop dat het allemaal stiekum toch nog goed komt...
Fotoboek
  vrijdag 8 juli 2011 @ 08:44:10 #137
148507 Schuifpui
Lone Star
pi_99186975
Bedankt, ik zal volgende week eens proberen uit te zoeken wat de beste optie is.
  vrijdag 29 juli 2011 @ 14:48:42 #138
46802 Visitor.Q
Neemt alles op!
pi_100088781
Ik wil wat math fontjes in een figuur (PDF) zetten, en ik vind het het mooist om deze door LaTeX te laten renderen. Ik gebruik tikzpicture, en daarin een \includegraphics, en daarna zet ik met behulp van \pgfputat{\pgfxy(0.6,1.6)}{\pgfbox[left,base]{$x$}} een x op de plek waar die moet staan. Dat is wat gepriegel maar het werkt, alleen als ik van mijn draft naar origineel ga (A4 naar B5 papierformaat), verschuift de hele zooi en moet ik alles opnieuw gaan zitten verschuiven.
Kan ik de getallen die ik netjes finetune vermenigvuldigen met een afstand, zodat latex tijdens compile time de echte coordinaten uitrekent?

edit: gevonden! Je kunt dus gewoon * gebruiken. Ik heb nu een newcommand \xl=1 voor A4 paper, en \xl=0.838095238 (verhouding korte kant van B5 tov A4), en kan nu \pgfxy{0.6*\xl,1.6*\yl} gebruiken.

[ Bericht 6% gewijzigd door Visitor.Q op 29-07-2011 14:57:06 ]
pi_100124415
Fok heeft TeX support *O*
1
2
[tex]\lim_{x\to{}0}\ \frac{1}{x}\ =\ \infty[/]
[tex]\forall{}x\[-1\ \lt\ x\ \lt{}\ 2\ :\ x^{3} -  3x - 2\ \lt{}\ 0\][/]
Geeft
\lim_{x\to{}0}\ \frac{1}{x}\ =\ \infty
\forall{}x\[-1\ \lt\ x\ \lt{}\ 2\ :\ x^{3} -  3x - 2\ \lt{}\ 0\]
das einzige erleuchtenden an religion ist ein brennende tempel bei nacht
pi_100129457
x^2

edit: Nice :)

edit2:

\lim_{x\to 0}{\frac{e^x-1}{2x}} \overset{\left[\frac{0}{0}\right]}{\underset{\mathrm{H}}{=}}\lim_{x\to 0}{\frac{e^x}{2}}={\frac{1}{2}}
pi_100386971
Vraagje:

Ik heb veel formules met subscripts, die vaak terugkomen, bijvoorbeeld:

C_{L_{max}}

Nu valt m'n begeleider erover dat de max italic is, dat moet recht en in normale spacing. Ik weet dat het met \mathrm{} kan.

C_{L_{\mathrm{max}}}

Echter komt max heel vaak terug, dus m'n begeleider zei dat het handiger was om een nieuw command daarvoor aan te maken. Net als \sin e.d. dus. Ik snap alleen niet helemaal hoe dat moet. Iemand een idee? Als ik op internet dingen zoek, vind ik alleen \mathrm{}.
pi_100387334
In de preamble:
1\newcommand{\max}{\mathrm{max}}
of
1\DeclareMathOperator{\max}{max}
Die laatste vereist wel een of ander package.
pi_100387696
thanks *O*

Die eerste werkt. Behalve voor \max, want zo'n command bestaat blijkbaar al. :P
pi_100389242
Als een command al bestaat kun je \renewcommand gebruiken.
pi_100389562
Ah nog handiger, bedankt. :)
pi_101591619
Nog een probleem. -O-

Ik heb een sidewaystable, maar hij is nog te groot eigenlijk. Daarom moet hij een stukje naar links, of eigenlijk dus naar onderen op de pagina. Ik ben al 2 uur aan het kutten, maar kan het nog steeds niet vinden. Stomme begeleider die er persé nog een kolom bij moest hebben. ;(

Zie plaatje voor verduidelijking. De tabel gaat door de header heen, terwijl er onderin genoeg ruimte is.

pi_101595476
Wat is dit kut zeg. :{

Ik krijg het niet voor elkaar. Wel kan ik hem klein genoeg maken, zodat het past, met b{1.5cm} in de alignment settings van de tabel, maar dan wordt m'n tekst links uitgelijnd en dat kan ik dan niet meer aanpassen. In het plaatje hierboven staat het overigens ook fout, het moet rechts uitgelijnd worden.
  dinsdag 6 september 2011 @ 07:34:16 #148
30719 keesjeislief
NextGenerationHippie
pi_101622149
Om een gewone tabel ver genoeg naar links te kunnen plaatsen kun je definieren:

1
2
3
4
5
6
7
8
9
10
11
\newenvironment{narrow}[2]{%
  \begin{list}{}{%
    \setlength{\topsep}{0pt}%
    \setlength{\leftmargin}{#1}%
    \setlength{\rightmargin}{#2}%
    \setlength{\listparindent}{\parindent}%
    \setlength{\itemindent}{\parindent}%
    \setlength{\parsep}{\parskip}%
  }%
  \item[]
}{\end{list}}

en dan gebruiken als:

1
2
3
4
5
6
7
8
\begin{table}[h!]
\begin{narrow}{-2cm}{-2cm}
\begin{center}
........
\end{center}
\end{narrow}
  \caption{Limiting Distributions of Overshoot and Undershoots in the Cram\'er and Convolution Equivalent Cases}
\end{table}

Werkt dat in jouw geval ook?
heeft de hoop dat het allemaal stiekum toch nog goed komt...
Fotoboek
pi_101626027
Lijkt niet te werken, 100 errors. De eerste errors zijn: LateX error: Something's wrong, perhaps a missing \item

Ik heb dat eerste stuk in de preamble gezet en de tabel begint nu zo:

1
2
3
4
5
6
\begin{sidewaystable}[!htb]\footnotesize
    \centering
    \begin{tabular}{l l r r r r r r r r l} 
    \begin{narrow}{-2cm}{-2cm} 
    \begin{center}
--- etc
pi_101627118
Zijn deze alignment parameters te combineren?
1
2
\begin{tabular}{l l r r r r r r r r l}
\begin{tabular}{l l b{1.3cm} b{1.3cm} b{1.3cm} b{1.3cm} b{1.3cm} b{1.3cm} b{1.3cm} b{1.5cm} l}

Als ik dat zou hebben, dan moet het goed zijn. De breedte is vooral belangrijk voor de eerste row, de recht uitlijningen voor de rest.
  dinsdag 6 september 2011 @ 14:24:29 #151
30719 keesjeislief
NextGenerationHippie
pi_101631364
quote:
0s.gif Op dinsdag 6 september 2011 11:12 schreef Schuifpui het volgende:
Lijkt niet te werken, 100 errors. De eerste errors zijn: LateX error: Something's wrong, perhaps a missing \item

Ik heb dat eerste stuk in de preamble gezet en de tabel begint nu zo:
[ code verwijderd ]

Sorry, ik was niet duidelijk genoeg, dit is de volgorde die ik gebruikt heb (en die werkt):

1
2
3
4
5
6
7
8
9
10
11
12
13
14
\begin{table}[h!]
\begin{narrow}{-2cm}{-2cm}
\begin{center}
    \footnotesize{
    \begin{tabular}{| c | c | c | c |}
    \hline
    & Henk1              & Henk2                     & Henk3             \\
1  & Bla & Bla & Bla
    \end{tabular}
    }
\end{center}
\end{narrow}
  \caption{Limiting Distributions of Overshoot and Undershoots in the Cram\'er and Convolution Equivalent Cases}
\end{table}
heeft de hoop dat het allemaal stiekum toch nog goed komt...
Fotoboek
pi_101668483
Dan krijg ik geen errors, maar het doet helemaal niets. Waarschijnlijk kan dit niet met een sideways table. -O-

Ik denk dat ik toch dit moet doen, deze alignment parameters moet combineren. Ik hoop dat iemend me hiermee kan helpen. Die stomme tabel heeft me echt al veel te veel tijd gekost.
1
2
\begin{tabular}{l l r r r r r r r r l}
\begin{tabular}{l l b{1.3cm} b{1.3cm} b{1.3cm} b{1.3cm} b{1.3cm} b{1.3cm} b{1.3cm} b{1.5cm} l}
pi_101673525
Gelukt met alles tussen \begin{tabular} tot \end{tabular} tussen \scalebox{0.95}{ }.

Uiteraard inmiddels met het volgende probleem bezig: De tabellen op de plek krijgen waar ze moeten. :r
Schijt taal dat latex, hierna ga ik er nooit meer wat mee doen.

Zelfs m'n \newpage wordt gewoon keihard genegeerd. Waarom. |:( :r
pi_101673802
Gaat het om een float (figuur of tabel) die na \newpage geplaatst wordt? Gebruik dan \clearpage dan worden floats wel eerst geplaatst voordat de nieuwe pagina begint.

Met grote tabellen is latex inderdaad soms een beetje kut...
pi_101674051
Yes dat werkt. Thanks. *O*
pi_101693202
Ik ben net begonnen met LaTeX, maar ik loop wel tegen een aantal stompzinnige dingen aan waar jullie me hopelijk mee kunnen helpen.

Ik voel me er in ieder geval ontzettend dom door.

Ik wil iets betrekkelijk simpels:

Kopje, niet genummerd

[horizontale lijn]

nog een kopje, iets kleiner

bladiebla tekst

[horizontale lijn]

nog een kopje, zelfde uitstraling als hierboven

bladiebla tekst.

Nu zit het grootste probleem bij de enters. LaTeX heeft niet graag dat je zoiets doet geloof ik, getuige het geklooi met \hline, \hrule en \hrulefill. Mijn eerste aanpak was:

1
2
3
4
5
6
7
8
9
10
11
12
13
{\Large \textbf{Groot kopje}} \\[1cm]
\hrule
\paragraph{kleiner kopje} \\
\begin{tabbing}
blabla
\end{tabbing}

\hrule 

\paragraph{kleiner kopje} \\
\begin{tabular}{ l p{5cm} }
blabla
\end{tabular}

Maar dan vind hij het leuk om het tabelletje NAAST "kleiner kopje" te zetten. Voor de rest als ik de errors (Underfull \hbox (badness 10000)) negeer ziet het er wel okay uit, alhoewel het iets mooier zou zijn als de lijnen wellicht iets kleiner zijn.

Omdat het blijkbaar in het tweede geval onmogelijk is om een enter tussen mijn kopje te krijgen heb ik de \paragraph maar veranderd in \textbf{klein kopje} maar hierdoor staat deze tekst TEGEN de horizontale lijn aan. Uiteraard kan geen \\ daar wat aan doen. Op een of andere manier.

Hoe los ik dit op? Bestaat er in LaTeX niet zoiets als een harde enter, zonder bijdehand gedoe? Want hij negeert dus \\.

Edit: ook een \vspace kan mij niet meer redden.

[ Bericht 1% gewijzigd door Watertornado op 07-09-2011 22:45:57 ]
Beter onethisch dan oneetbaar
pi_101694137
Je kunt to gewoon op een nieuwe regel beginnen met \newline? Desnoods eerst newline dan \\ werkt dat niet?
"AAAAAHH ZENNE MOAT, WOARST VLEISCH"
pi_101694165
Voor koppen en subkoppen gebruik ik \section{] en subsection{}, eventueel nog subsubsection{]. Wel of geen nummering zit waarschijnlijk in je style bestand, maar daar weet ik niets vanaf.
pi_101694376
quote:
0s.gif Op woensdag 7 september 2011 22:57 schreef Schuifpui het volgende:
Voor koppen en subkoppen gebruik ik \section{] en subsection{}, eventueel nog subsubsection{]. Wel of geen nummering zit waarschijnlijk in je style bestand, maar daar weet ik niets vanaf.
Met section wordt het zo groot. En aangezien het maar een bestand is van een (of misschien twee) pagina's leek dit me makkelijker.

quote:
0s.gif Op woensdag 7 september 2011 22:57 schreef znarch het volgende:
Je kunt to gewoon op een nieuwe regel beginnen met \newline? Desnoods eerst newline dan \\ werkt dat niet?
Heb 't nu min-of-meer voor elkaar met het gebruik van \textbf en vspace{}. Maar ik vind 't nogsteeds nogal "vuil". En je moet ná de vspace nog een \\ doen om te voorkomen dat de tabel náást de "kop" (in \textbf) komt. Daar heeft hij wel echt een grote voorkeur voor.
Beter onethisch dan oneetbaar
pi_101713709
Vraagje mensen... Hoe maak je in latex zo'n artikel met 2 kolommen wat je wel vaker ziet?
pi_101714025
quote:
7s.gif Op donderdag 8 september 2011 16:40 schreef Dale. het volgende:
Vraagje mensen... Hoe maak je in latex zo'n artikel met 2 kolommen wat je wel vaker ziet?
http://texblog.wordpress.(...)icle-report-or-book/

Een seconde Googlen.
pi_101718989
Of je pakt het template van bv IEEE, dan ben je in een keer klaar:
http://www.ieee.org/confe(...)shing/templates.html

@Watertornado:

Paragraph gebruik je (geloof ik) als eerste wordt van een alinea. Dit is dus mooier/handiger met section, subsection en subsubsection. Als je er een sterretje bijzet voorkom je dat het een nummer krijgt.

1
2
3
4
5
6
7
8
9
10
11
12
13
14
15
16
17
18
19
20
21
\section*{Heel Groot Kopje}


\subsection*{Groot Kopje}
\hrulefill

% Of
\subsection*{Groot Kopje}
\hrule 
\vspace{1cm}   % naar keuze, mag ook negatief zijn geloof ik


\subsubsection*{Kleine Kop}

\begin{tabbing} % geen idee wat dit doet eigenlijk, nooit eerder gezien
blabla
\end{tabbing}


% voorbeeld paragraph
\paragraph{bla} blabla blabla blabla blabla blabla blabla blabla blabla blabla blabla blabla blabla~blabla~blabla~blabla~blabla blabla blabla
pi_101854262
quote:
0s.gif Op donderdag 8 september 2011 19:18 schreef Toryu het volgende:
@Watertornado:

Paragraph gebruik je (geloof ik) als eerste wordt van een alinea. Dit is dus mooier/handiger met section, subsection en subsubsection. Als je er een sterretje bijzet voorkom je dat het een nummer krijgt.
[ code verwijderd ]

Dit lijkt inderdaad veel beter te werken! Thanks!

Overigens is de {tabbing}-omgeving puur om met tabjes te werken. Door middel van \> kun je dan vooruit "tabben" nadat je met \= een "marker" hebt gegeven.

Zie een korte uitleg hier.
Beter onethisch dan oneetbaar
pi_101854706
Ahh zo. Ik begrijp alleen niet helemaal niet helemaal wanneer je dat zou willen gebruiken. Het mooie van LateX vind ik juist dat ik niet hoef te klooien met tabjes ;) Als ik een opsomming wil maken, gebuik ik enumerate of itemize.En om een heel stuk tekst in te laten springen, zou ik dus eerder kiezen voor
1
2
3
\begin{quote}
bla
\end{quote}
pi_101854879
quote:
0s.gif Op maandag 12 september 2011 13:37 schreef Toryu het volgende:
Ahh zo. Ik begrijp alleen niet helemaal niet helemaal wanneer je dat zou willen gebruiken. Het mooie van LateX vind ik juist dat ik niet hoef te klooien met tabjes ;) Als ik een opsomming wil maken, gebuik ik enumerate of itemize.En om een heel stuk tekst in te laten springen, zou ik dus eerder kiezen voor
[ code verwijderd ]

Ik heb het gebruikt voor
1
2
3
4
naam:           Toryu
adres:          LaTeXlaan 2.9
telefoonnummer: 099-1234567
...

etc gegevens. Dat kan uiteraard ook in een tabel. Maar dit is dan makkelijker. Jammer genoeg kun je geloof ik niet buiten je tabbing omgeving weer oplijnen met je vorige "markers". Maar daar is wel omheen te werken. :)
Beter onethisch dan oneetbaar
pi_101880967
Is er trouwens een manier hoe je bij een tabel de tekst in de laatste kolom automatisch kan wrappen tot aan de textwidth? Dus eigenlijk wil je een breedte van {p=\textwidth - breedte rest kolommen} aanmaken?

Edit: gelukt in de tabularx omgeving.

[ Bericht 10% gewijzigd door Watertornado op 14-09-2011 17:51:16 ]
Beter onethisch dan oneetbaar
pi_102947858
ik ben er ook weer, ben met afstudeerstage bezig en daar hoort ook een verslag bij. Bedrijf heeft blijkbaar een template, maar die moet ik nog ff op de kop tikken :D
Vaduz: Je snapt toch wel waarom sbientje het Franse woord bien in d'r naam heeft?
pi_102955601
Frappant dat een bedrijf dat heeft!
pi_103113483
Ja idd! Als voorblad gebruik ik nu altijd een Word-bestand dat ik naar pdf heb omgezet en dat als image...

Soit, ik had ook een klein vraagje; heeft iemand een idee wat ik kan doen als ik chemische moleculen wil invoegen? Momenteel red ik mij door afbeeldingen van wikipedia te halen maar echt simpel is het niet, en ook niet altijd even consistent qua typesetting...

Oja, chemdraw of hoe-het-de-industriestandaard is voor mij geen optie (heb 't niet zo vaak nodig dat ik er honderden euro's aan kan geven).
Voor de rest gebruik ik OSX als 't niet in LaTeX zelf kan :).
pi_103113898
quote:
0s.gif Op zaterdag 15 oktober 2011 15:08 schreef tony_clifton- het volgende:
Ja idd! Als voorblad gebruik ik nu altijd een Word-bestand dat ik naar pdf heb omgezet en dat als image...

Soit, ik had ook een klein vraagje; heeft iemand een idee wat ik kan doen als ik chemische moleculen wil invoegen? Momenteel red ik mij door afbeeldingen van wikipedia te halen maar echt simpel is het niet, en ook niet altijd even consistent qua typesetting...

Oja, chemdraw of hoe-het-de-industriestandaard is voor mij geen optie (heb 't niet zo vaak nodig dat ik er honderden euro's aan kan geven).
Voor de rest gebruik ik OSX als 't niet in LaTeX zelf kan :).
Er zijn natuurlijk nog meer programma's die chemische formules kunnen tekenen die gratis zijn, bijvoorbeeld ChemSketch, BKChem, XDrawChem (al is dit wat minder mooi) etc.

Dus er zijn zeker opties.
Beter onethisch dan oneetbaar
pi_103154611
Okay, thanks! Zal ze eens checken!
pi_103618737
Ik gebruik in mijn verslag eps plaatjes, en compileer daarom via Latex+dvi2pdf. Nu zou ik ook graag het todonotes package willen gebruiken, maar die werkt alleen met pdflatex. Maar daarmee zien mijn plaatjes er nogal gaar uit.

Is hier een oplossing voor, zodat ik zowel eps als todonotes kan gebruiken?
pi_103630923
Wat is er gaar aan je plaatjes? Met de juiste eps-naar-pdf omzetter zou je geen tot verwaarloosbaar verschil mogen zien, dacht ik...
pi_103652076
Hoe doe ik het volgende in LaTeX:



Bij voorbaat dank. :)
  vrijdag 28 oktober 2011 @ 00:04:17 #175
117598 Gebraden_Wombat
lekker bij rijst
pi_103652936
1\min_{i \leq k < j}
Op dinsdag 23 augustus 2011 23:18 schreef problematiQue het volgende:
Mensen die zomaar claimen dat A beter is dan B moet je gewoon negeren. Internetruis.
pi_103655469
quote:
0s.gif Op vrijdag 28 oktober 2011 00:04 schreef Gebraden_Wombat het volgende:

[ code verwijderd ]

Thanks, dus toch. Geeft 'n ander resultaat inline dan in 'n math block. :)
  vrijdag 28 oktober 2011 @ 07:30:30 #177
136730 PiRANiA
All thinking men are atheists.
pi_103658330
Vergeet de tex-tag op FOK! niet :)
\min_{i \leq k < j}
pi_103660553
quote:
10s.gif Op vrijdag 28 oktober 2011 07:30 schreef PiRANiA het volgende:
Vergeet de tex-tag op FOK! niet :)
\min_{i \leq k < j}
wist niet eens dat die er was :D
Vaduz: Je snapt toch wel waarom sbientje het Franse woord bien in d'r naam heeft?
pi_103662034
@vriendelijkevriend
Bedankt, het lijkt te werken. Ik heb het nu zo:
\usepackage[pdftex]{graphicx}
\usepackage{epstopdf}
\DeclareGraphicsExtensions{.eps}
  vrijdag 28 oktober 2011 @ 11:24:16 #180
117598 Gebraden_Wombat
lekker bij rijst
pi_103662400
quote:
10s.gif Op vrijdag 28 oktober 2011 01:05 schreef Diabox het volgende:

[..]

Thanks, dus toch. Geeft 'n ander resultaat inline dan in 'n math block. :)
Klopt, dat wist ik ook niet. Je kunt overwriten:
1\min\limits_{i \leq k < j}
Of je kunt de equation-variant veranderen naar de inline-variant:
1\min\nolimits_{i \leq k < j}

Zie ook http://en.wikibooks.org/w(...)#Advanced_formatting
Op dinsdag 23 augustus 2011 23:18 schreef problematiQue het volgende:
Mensen die zomaar claimen dat A beter is dan B moet je gewoon negeren. Internetruis.
  vrijdag 28 oktober 2011 @ 11:28:14 #181
136730 PiRANiA
All thinking men are atheists.
pi_103662539
Gebruik de tex-tags *O*.

1\min\limits_{i \leq k < j}
\min\limits_{i \leq k < j}

1\min\nolimits_{i \leq k < j}
\min\nolimits_{i \leq k < j}
pi_103750192
Hoe krijg ik het volgende in LaTeX?

Twee plaatjes naast elkaar (onderschrift lukt denk ik zelf wel)
Het is me wel eens gelukt om een plaatje in te voegen maar niet op een goede plek te krijgen, ook was de kwaliteit van het plaatje gewoon bagger.

Moet ik verder nog iets ''aanzetten'' in de bovenste gedeelte? (Ik gebruik het programma niet erg veel dus weet maar weinig termen, ik probeer zo veel mogelijk op internet te zoeken)



en het tweede (plaatje links en dan tekst aan de rechterkant)

pi_103750486
quote:
0s.gif Op zondag 30 oktober 2011 17:46 schreef stevin het volgende:
Hoe krijg ik het volgende in LaTeX?

Twee plaatjes naast elkaar (onderschrift lukt denk ik zelf wel)
Het is me wel eens gelukt om een plaatje in te voegen maar niet op een goede plek te krijgen, ook was de kwaliteit van het plaatje gewoon bagger.

Moet ik verder nog iets ''aanzetten'' in de bovenste gedeelte? (Ik gebruik het programma niet erg veel dus weet maar weinig termen, ik probeer zo veel mogelijk op internet te zoeken)

[ afbeelding ]

en het tweede (plaatje links en dan tekst aan de rechterkant)

[ afbeelding ]
de eerste heb ik zo gedaan:
1
2
3
4
5
6
7
\begin{figure}
  \centering
  \subfloat[\textit{N-gedopeerd silicium}]{\label{fig:ndope}\includegraphics[width=0.5\textwidth]{ndope}}                
  \subfloat[\textit{P-gedopeerd silicium}]{\label{fig:pdope}\includegraphics[width=0.5\textwidth]{pdope}}
    \caption[Gedopeerd silicium]{\textit{Schematische weergave van elektronen en gaten in gedopeerd silicium}~\cite{wikindope,wikipdope}}
  \label{fig:dope}
\end{figure}

ik denk met package subfig en/of float maar dat weet ik niet zeker. Code ziet er overigens lelijk uit vind ik, beetje onduidelijk. Dus wie weet kan het wel makkelijker. Ik heb dit iig gekopieerd van een wikibook ofzo :)
Vaduz: Je snapt toch wel waarom sbientje het Franse woord bien in d'r naam heeft?
pi_103754778
plaatje twee is me inmiddels gelukt, 1 nog niet.

Kort vraagje: ik krijg nu figuur 1.1 ipv gewoon figuur 1
(geen nummering na 1)

Hoe kan ik dat fixen?
(ik heb float, sidecap, wrapfig, graphicx nu aan staan, voorheen niet)
pi_103757596
quote:
0s.gif Op zondag 30 oktober 2011 19:47 schreef stevin het volgende:
plaatje twee is me inmiddels gelukt, 1 nog niet.

Kort vraagje: ik krijg nu figuur 1.1 ipv gewoon figuur 1
(geen nummering na 1)

Hoe kan ik dat fixen?
(ik heb float, sidecap, wrapfig, graphicx nu aan staan, voorheen niet)
Je gebruikt een report met chapters neem ik aan? :)
Beter onethisch dan oneetbaar
pi_103758092
Uhmm, ja ik denk het wel. Documentclass is idd report en ik gebruik hoofdstukken.
pi_103758984
quote:
0s.gif Op zondag 30 oktober 2011 20:41 schreef stevin het volgende:
Uhmm, ja ik denk het wel. Documentclass is idd report en ik gebruik hoofdstukken.
Als je article gebruikt en het indeelt in sections ben je je probleem kwijt.

Maar wellicht niet helemaal een netjes antwoord aangezien je misschien wel heel graag de indeling met chapters wil gebruiken.

Dan zul je de chngcntr-package moeten ophalen en zo de \counterwithout{figure} moeten gebruiken.
Beter onethisch dan oneetbaar
pi_103761955
Best gek dat hij dat nu doet. Mijn eerste plaatje had gewoon Figuur 1 als naam. Niet 1.1 o.i.d.
pi_103765364
quote:
0s.gif Op zondag 30 oktober 2011 17:56 schreef sbientje het volgende:

[..]

de eerste heb ik zo gedaan:
[ code verwijderd ]

ik denk met package subfig en/of float maar dat weet ik niet zeker. Code ziet er overigens lelijk uit vind ik, beetje onduidelijk. Dus wie weet kan het wel makkelijker. Ik heb dit iig gekopieerd van een wikibook ofzo :)
Dit is me ook gelukt, maar ik bedoel het eigenlijk anders.

Ik wil:

1
2
3
[plaatje 1]                   [plaatje 2]
Figuur 1                      Figuur 2
Hier bijschrift               Hier bijschrift

Hier gewone tekst van het rapport/paper.

Edit: nog een vraagje, als ik m in a5 papier zet (met geometry aan) komen enkele woorden buiten de buitenrand
Misschien een beetje vaag, maar ik bedoel, rechts is zeg maar een kantlijn, daar moet het woord worden afgebroken, meestal gebeurt dat maar soms ook niet.
Verder, kan je geen kleiner lettertype gebruiken dan 10pt? Heb 8 en 9 gebruikt maar deze werken niet (ik vind 10 net iets te groot voor a5). Misschien is er een scale optie?

[ Bericht 16% gewijzigd door stevin op 31-10-2011 22:19:09 ]
pi_103825229
veel mensen hier die ervaring hebben met het maken van presentaties in LaTeX? Lastig? Voordelen? Gebruik nu nog steeds powerpoint :)
Vaduz: Je snapt toch wel waarom sbientje het Franse woord bien in d'r naam heeft?
pi_103826338
quote:
0s.gif Op dinsdag 1 november 2011 11:44 schreef sbientje het volgende:
veel mensen hier die ervaring hebben met het maken van presentaties in LaTeX? Lastig? Voordelen? Gebruik nu nog steeds powerpoint :)
Voor presentaties gebruik ik meestal gewoon krijt. Voordelen zijn dat het tempo goed is, dat het publiek niet in keer *flits boem* een hoop informatie op zich af krijgt en dat die informatie vervolgens ook niet met 1 druk op de knop net zo snel weer weg is.

Als ik het al in LaTeX doe, dan probeer ik alles zo simpel mogelijk te houden. Geen ingewikkelde tierelantijntjes die de beamerclass allemaal heeft, geen andere kleuren dan zwart en wit, en ook gewoon een wat luchtigere inhoud.
pi_103826586
quote:
12s.gif Op dinsdag 1 november 2011 12:15 schreef thabit het volgende:

[..]

Voor presentaties gebruik ik meestal gewoon krijt. Voordelen zijn dat het tempo goed is, dat het publiek niet in keer *flits boem* een hoop informatie op zich af krijgt en dat die informatie vervolgens ook niet met 1 druk op de knop net zo snel weer weg is.

Als ik het al in LaTeX doe, dan probeer ik alles zo simpel mogelijk te houden. Geen ingewikkelde tierelantijntjes die de beamerclass allemaal heeft, geen andere kleuren dan zwart en wit, en ook gewoon een wat luchtigere inhoud.
tsja, mijn school verplicht het gebruik van een beamer en slides :') dus met krijt of whiteboard kom ik er niet.
Vaduz: Je snapt toch wel waarom sbientje het Franse woord bien in d'r naam heeft?
  dinsdag 1 november 2011 @ 12:44:17 #193
46802 Visitor.Q
Neemt alles op!
pi_103827195
quote:
0s.gif Op dinsdag 1 november 2011 11:44 schreef sbientje het volgende:
veel mensen hier die ervaring hebben met het maken van presentaties in LaTeX? Lastig? Voordelen? Gebruik nu nog steeds powerpoint :)
Ik gebruik het standaard voor presentaties en posters. Als je al redelijk wat in LaTeX hebt gedaan is beamer niet heel lastig en er zijn genoeg .tex voorbeelden waar je in no-time je eigen presentatie in kunt gieten. Zeker als je het bij plaatje-links-opsomming-rechts achtige zaken houdt werkt dat prima.

Het voordeel voor mij is dat ik equations, pdf/eps figuren etc van papers kan gebruiken. En ik vind het fijn om overlays te gebruiken (bv grafieken waarvan er steeds een extra dataset bij komt als ik klik), zo kun je individuele resultaten bespreken zonder dat ineens BAM de hele grafiek met 5 datasets in beeld staat. In ppt kan dat natuurlijk ook, maar dan verschilt de positie van het plaatje van slide tot slide en verspringt deze steeds met een pixel (dat boeit trouwens niemand behalve mijzelf). Ook vind ik de progress bar van beamer wel handig, hij zet dus afhankelijk van het thema automatisch de secties en slides ergens in beeld zodat je publiek weet waar je bent in je verhaal. Tenslotte zorgt het er vooral voor dat je niet van die uitpuilende slides krijgt met informatie in elk vrij hoekje.

Natuurlijk zijn er zat nadelen. Wat in ppt makkelijk gaat is plaatjes en tekstvakken over elkaar heen positioneren, en dat kan een enorm gepriegel zijn. Ook werk ik graag met hoge resolutie plaatjes en het duurt dan altijd even voordat de presentatie gecompiled is. Het werken met animaties en filmpjes is maar zo-zo (en bovendien alleen in acroreader op windows).
pi_103843832
Een simpel vraagje;

Hoe kan ik de witruimte tussen de tabeltitel en de tabel zelf iets vergroten? met \\ wordt 't direct een hele regel en da's te veel...

Alvast bedankt! :)

Wat 't nu is:



De code:

1
2
3
4
5
6
7
8
9
10
11
12
13
\begin{table}[ht]
\caption{Piekhoogten en stroomgevoeligheid naar $\mu$A voor Cd$^{2+}$}
\centering
\begin{tabular}{r c c c c}
$f_{verd}$     & [Cd$^{2+}$] ($\frac{mg}{ml}$) & $H$ (mm) & I-gevoeligheid ($\frac{A}{mm}$) & $H$ ($\mu$A)\\ [0.5 ex]
\hline
5x         & 0,1233            & 69                & $10\cdot 10^{-9}$        & 0,69    \\
6,67x    & 0,0925            & 190            & $5\cdot 10^{-9}$        & 0,55    \\
10x        & 0,0616            & 170            & $2\cdot 10^{-9}$        & 0,34    \\
20x        & 0,0308            & 169            & $1\cdot 10^{-9}$        & 0,17    \\
\end{tabular}
\label{tabel piekhoogten cadmium}
\end{table}


[ Bericht 92% gewijzigd door tony_clifton- op 01-11-2011 19:46:12 ]
pi_103845009
quote:
0s.gif Op dinsdag 1 november 2011 19:38 schreef tony_clifton- het volgende:
Een simpel vraagje;

Hoe kan ik de witruimte tussen de tabeltitel en de tabel zelf iets vergroten? met \\ wordt 't direct een hele regel en da's te veel...

Alvast bedankt! :)

Wat 't nu is:

[ afbeelding ]

De code:
[ code verwijderd ]

Werkt het niet gewoon met een \vspace{}, waarbij tussen de accolades een willekeurige grootte ingevuld kan worden?

Overigens houd je een aparte schrijfwijze qua eenheden erop na. De mu kun je gewoon romein laten. Net zoals de (\frac{mg}{ml}) waar ik eerder iets als \left(\mathrm{\frac{mg}{ml}}\right) van zou maken. En het is f_{\mathrm{verd}} i.p.v. f_{verd}. Voor de rest kom je er wel uit, maar dit zijn wat "overige" tips. :P

Edit: hm, in de tabel zelf gebruik je ook \\ [0.5 ex], werkt dat daar niet?

Edit2: zo 1-2-3 dit dus, niet nagekeken of het werkt maar goed.

1
2
3
4
5
6
7
8
9
10
11
12
13
\begin{table}[ht]
\caption{Piekhoogten en stroomgevoeligheid naar $\mathrm{\mu}$A voor Cd$^{2+}$ \vspace{0.5ex}}
\centering
\begin{tabular}{r c c c c}
$f_{\mathrm{verd}}$     & [Cd$^{2+}$] $\left(\frac{mg}{ml}\right)$ & $H$ (mm) & $I$-gevoeligheid $\left(\mathrm{\frac{A}{mm}}\right)$ & $H$ ($\mathrm{\mu}$A)\\ [0.5 ex]
\hline
5\times         & 0,1233            & 69             & $10\cdot 10^{-9}$       & 0,69    \\
6,67\times      & 0,0925            & 190            & $5\cdot 10^{-9}$        & 0,55    \\
10\times        & 0,0616            & 170            & $2\cdot 10^{-9}$        & 0,34    \\
20\times        & 0,0308            & 169            & $1\cdot 10^{-9}$        & 0,17    \\
\end{tabular}
\label{tabel piekhoogten cadmium}
\end{table}


[ Bericht 14% gewijzigd door Watertornado op 01-11-2011 20:09:53 ]
Beter onethisch dan oneetbaar
  dinsdag 1 november 2011 @ 20:19:41 #196
46802 Visitor.Q
Neemt alles op!
pi_103846154
Het wordt sowieso veel mooier door het pakket booktabs te gebruiken, dat geeft net wat meer ruimte tussen de regels. Ik zag net de tips van watertornado, dat moet je idd ook doen (deels meegenomen hieronder), \times moet dacht ik tussen haakjes.



1
2
3
4
5
6
7
8
9
10
11
12
13
14
15
\begin{table}[ht]
\caption{Piekhoogten en stroomgevoeligheid naar $\mu$A voor Cd$^{2+}$}
\centering
\begin{tabular}{rcccc}
\toprule
$f_\mathrm{verd}$     & [Cd$^{2+}$] ($\frac{\mathrm{mg}}{\mathrm{ml}}$) & $H$ (mm) & I-gevoeligheid ($\frac{\mathrm{A}}{\mathrm{mm}}$) & $H$ ($\mu$\mathrm{A})\\ [0.5 ex]
\midrule
$5\times$         & 0,1233            & 69                & $10\cdot 10^{-9}$        & 0,69    \\
$6,67\times$    & 0,0925            & 190            & $5\cdot 10^{-9}$        & 0,55    \\
$10\times$        & 0,0616            & 170            & $2\cdot 10^{-9}$        & 0,34    \\
$20\times$        & 0,0308            & 169            & $1\cdot 10^{-9}$        & 0,17    \\
\bottomrule
\end{tabular}
\label{tabel piekhoogten cadmium}
\end{table}

edit: je kunt ook pakket SIunits gebruiken, dan kun je eenheden schrijven als \kilo\gram\per\cubic\meter of \pascal\usk\second (waarbij usk voor een \cdot staat, of een spatie als je wil - het geeft je een consistent gebruik van eenheden).
pi_103934615
Okay thanks! 'T was mijn eerste tabel en ik had de eenheden zo gedaan om de tabel binnen de marges te houden. Liefst had ik mg.ml-1 gebruikt, maar in dat geval moest ik één rij in twee 'deelrijen' onderverdelen en dat kon ik niet. Like I said, first-timer :). Ik heb de codes wel gevonden, maar om 't foutloos te krijgen moet ik nog wat proberen :).


In ieder geval bedankt, dit zijn dingen die ik zeker ga onthouden :).

[ Bericht 18% gewijzigd door tony_clifton- op 03-11-2011 22:00:18 ]
pi_104040481
Als ik in mijn TeXnicCenter
1x=\frac{1+y}{1+2z^2}
invoer, krijg ik:
1+y\frac{}{1+2z^2} (maar dan zonder [?], ik weet niet hoe ik deze weg kan laten)
ipv
x=\frac{1+y}{1+2z^2}

Erg irritant, weet iemand waardoor dit kan komen?
pi_104041012
quote:
0s.gif Op zondag 6 november 2011 17:36 schreef kutkloon7 het volgende:
Als ik in mijn TeXnicCenter
[ code verwijderd ]

invoer, krijg ik:
1+y\frac{}{1+2z^2} (maar dan zonder [?], ik weet niet hoe ik deze weg kan laten)
ipv
x=\frac{1+y}{1+2z^2}

Erg irritant, weet iemand waardoor dit kan komen?
Geef eens een minimum example waarin dit voorkomt.
pi_104041081
quote:
0s.gif Op zondag 6 november 2011 17:51 schreef thabit het volgende:

[..]

Geef eens een minimum example waarin dit voorkomt.
In alles eigenlijk, dus bijvoorbeeld:
1
2
3
4
\documentclass[12pt]{article}
\begin{document}
\frac{asd}{asdq}
\end{document}
pi_104041353
Volgens mij (bedenk ik net) gebeurde het na het downloaden van één van deze packages:
1
2
3
4
\usepackage[latin1]{inputenc}
\usepackage{amsmath}
\usepackage{amsfonts
\usepackage{amssymb}
(toen ik een .tex bestand met oa dit erin probeerde te converteren met de commandline ging ie deze eerst downloaden en installeren, daarna merkte ik dat hij \frac niet goed meer pakte. Daarvoor heeft hij het zeker goed gedaan)
pi_104041379
quote:
0s.gif Op zondag 6 november 2011 17:52 schreef kutkloon7 het volgende:

[..]

In alles eigenlijk, dus bijvoorbeeld:
[ code verwijderd ]

In dit geval komt het doordat je niet in mathmode zit.
pi_104041427
quote:
0s.gif Op zondag 6 november 2011 18:01 schreef thabit het volgende:

[..]

In dit geval komt het doordat je niet in mathmode zit.
Ai, ik zie het al, n00b foutje :@, ik was gewoon de $'s vergeten. Ik was een beetje in de war omdat die code hier op het forum wel gewoon werkt.
pi_104119266
Vraagje... Hoe zet ik in een vierkant bij Tikz tekst? Ik heb nu:

1\draw[fill=blue,opacity=.5] (0,0) rectangle (2,2);
Nu wil ik dat hierin bijvoorbeeld test komt te staan.
pi_104519703
Hallo,

na vele beamerpresentaties wil ik voor het eerst een bibliografie op het einde tonen (zeer experimenteel)

Ik wou deze code gebruiken:

1
2
3
\begin{thebibliography}{}
\bibitem C.~Gauss \newblock{Best book ever} \newblock{Series of excellence}
\end{thebibliography}

quote:
. Gauss
Best book ever
Series of excellence
Dat kon toch niet de bedoeling zijn? Waar is de C. heen?

Bedankt!
pi_104519910
Wat gebeurt er als je "\bibitem{Naam} C.~Gauss" enz. doet?
pi_104520125
quote:
0s.gif Op vrijdag 18 november 2011 17:44 schreef vriendelijkevriend het volgende:
Wat gebeurt er als je "\bibitem{Naam} C.~Gauss" enz. doet?
Is al veel beter! *O*

Maar de logica ontgaat me eigenlijk een beetje... ik ging geen naam gebruiken (ik doe nu gewoon {}), wat kan ik met zo'n naam doen? Het is niet zoals in gewone tekst dat ik \ref{Naam} kan doen om ernaar te verwijzen?
pi_104520387
nee \cite moet je dan doen :)
Vaduz: Je snapt toch wel waarom sbientje het Franse woord bien in d'r naam heeft?
  zondag 20 november 2011 @ 04:23:02 #209
30719 keesjeislief
NextGenerationHippie
pi_104576345
Een andere reden is dat het commando anders niet goed gebruikt wordt.
heeft de hoop dat het allemaal stiekum toch nog goed komt...
Fotoboek
pi_104844083
Weet niet of het probleem geogebra, pdf of latex is (ik gok de eerste eigenlijk) maar toch;

Ik maak afbeeldingen in geogebra, en die exporteer ik dan als pdf. Het probleem is dat er dan teveel witruimte boven en onder m'n afbeelding komt, waardoor de afbeeldingen niet mooi in latex komen. Wat is de makkelijkste manier om dit op te lossen?



Edit, gevonden denk ik. De optie om geogebra tikzcode te laten maken!

[ Bericht 7% gewijzigd door Hanneke12345 op 26-11-2011 16:09:10 ]
pi_104968362
Beetje een stomme vraag, maar hoe kan ik (voor een sinterklaasgedicht :') ) een ander font kiezen?

ik heb zoiets:
1
2
3
4
5
6
7
8
\documentclass[12pt]{article}
\usepackage[T1]{fontenc}
\usepackage{vicent}
\begin{document}
\normalfont\cursiveshape
bla bla

\end{document}
Van:
http://www.tug.dk/FontCatalogue/vicentino/
En de sty:
http://www.ctan.org/tex-archive/fonts/vicentino/kilfiger

Maar na compilen krijg ik een pdf met het standaard latex font en de warning:

LaTeX Font /b/c12/cWarning:/b/c0/c Font shape `OT1//vc//m//it' undefined
(Font) using `OT1//cmr//m//n' instead on input line 10.
LaTeX Font /b/c12/cWarning:/b/c0/c Some font shapes were not available, defaults substituted.

Hoe verhelp ik dit?
pi_105130592
Is dit mogelijk in LaTeX?



tot nu toe krijg ik het volgende:


Dit is dus nog niet geheel wat ik wil. Het bovenstaande heb ik gedaan met de code:
SPOILER
Om spoilers te kunnen lezen moet je zijn ingelogd. Je moet je daarvoor eerst gratis Registreren. Ook kun je spoilers niet lezen als je een ban hebt.
Vraag 2
LaTeX breekt de woorden raar af. Bijvoorbeeld op het einde van een zin breekt hij als volgt af:
Bedri-
jfsonderdelen

pa-
per

volgens mij moet dit ook anders kunnen, dat hij het woord gewoon op de volgende regel zet en de zin dan iets breder maakt (net als Word)
Misschien iemand een idee hierover?

Ik ga eerst http://www.vtk.ugent.be/w(...)ndstalige_documenten proberen (is me niet gelukt... de download link werkte ook niet meer)

[ Bericht 3% gewijzigd door stevin op 03-12-2011 17:58:03 ]
pi_105132612
1\usepackage[none]{hyphenat}
Zorgde er dacht ik voor dat je woorden niet worden afgebruiken.
Not sure though, het ziet eruit alsof jij meer van LaTeX weet dan ik :P
pi_105133826
Haha, ik weet maar zéér weinig van LaTeX. Ik probeer zo veel mogelijk de info te googelen. Ik zal dat commando eens proberen, bedankt!

Het probleem met de plaatjes is groter, omdat het vaker voorkomt dat twee plaatjes naast elkaar moeten staan met een ander onderschrift...
pi_105139348
1\usepackage[dutch]{babel}
Je wil het afbreken juist niet uitzetten, da's nou net een van de vette mooie dingen van LaTeX.
pi_105139511
quote:
0s.gif Op zaterdag 3 december 2011 18:15 schreef vriendelijkevriend het volgende:

[ code verwijderd ]

Je wil het afbreken juist niet uitzetten, da's nou net een van de vette mooie dingen van LaTeX.
Ik had een keer een korte inleveropgave uitgewerkt in LaTeX waar één woord werd afgebroken, dus toen heb ik het voor het gemak maar even uitgezet. Verder geen idee hoe het anders moet ook :P.
pi_105139566
quote:
0s.gif Op zaterdag 3 december 2011 18:15 schreef vriendelijkevriend het volgende:

[ code verwijderd ]

Je wil het afbreken juist niet uitzetten, da's nou net een van de vette mooie dingen van LaTeX.
Hmm, die commando had ik er al in staan. Volgens mij heb je een NL woordenlijst nodig ofzo om in het Nederlands ook goede woordafbreking te krijgen.
pi_105415970
quote:
0s.gif Op zaterdag 3 december 2011 13:12 schreef stevin het volgende:
Is dit mogelijk in LaTeX?

[ afbeelding ]

tot nu toe krijg ik het volgende:
[ afbeelding ]

Dit is dus nog niet geheel wat ik wil. Het bovenstaande heb ik gedaan met de code:
SPOILER
Om spoilers te kunnen lezen moet je zijn ingelogd. Je moet je daarvoor eerst gratis Registreren. Ook kun je spoilers niet lezen als je een ban hebt.
Vraag 2
LaTeX breekt de woorden raar af. Bijvoorbeeld op het einde van een zin breekt hij als volgt af:
Bedri-
jfsonderdelen

pa-
per

volgens mij moet dit ook anders kunnen, dat hij het woord gewoon op de volgende regel zet en de zin dan iets breder maakt (net als Word)
Misschien iemand een idee hierover?

Ik ga eerst http://www.vtk.ugent.be/w(...)ndstalige_documenten proberen (is me niet gelukt... de download link werkte ook niet meer)
Heeft iemand hier een oplossing voor? Misschien is het mogelijk om te werken met 2 kolommen?
Alhoewel dit idee een oplossing zou bieden, hoe moet je dat in commando's weergeven?

Ter verduidelijking een plaatje wat ik bedoel met 1 kolom (tekst) / 2 kolommen (plaatjes) / 1 kolom (tekst)

pi_105502111
quote:
0s.gif Op zaterdag 10 december 2011 15:10 schreef stevin het volgende:

[..]

Heeft iemand hier een oplossing voor? Misschien is het mogelijk om te werken met 2 kolommen?
Alhoewel dit idee een oplossing zou bieden, hoe moet je dat in commando's weergeven?

Ter verduidelijking een plaatje wat ik bedoel met 1 kolom (tekst) / 2 kolommen (plaatjes) / 1 kolom (tekst)

[ afbeelding ]
Volgens mij is het wel redelijk gebruikelijk om het caption van een plaatje langer te maken dan 1 zin. Dat kun je makkelijk doen en dan krijg je dus zoiets:

http://imageshack.us/photo/my-images/24/latexcol.png/ (copy/paste deze link)

Wat betreft het afbreken van woorden, heb je wel het Nederlandse tekstpakket gebruikt? De herkenning van afbreekpunten is namelijk afhankelijk van de taal. (edit: ik zie nu dat je zoiets al gevonden had)

[ Bericht 1% gewijzigd door rijsttafel op 12-12-2011 18:40:41 ]
Fok!-reputatiemeter van Rijsttafel:
Slecht [-----X----------------------------------------------------------] Goed
pi_105502292
Ik heb ook een vraagje. Ik wil delen maken in mijn scriptie. Hiervoor wil ik geen bestaande chapters en subsecties gebruiken, maar iets anders, ongenummerd. Ter verduidelijking:

1. Introduction
Part A
2. Chapter title 2
3. Chapter title 3
Part B
4. Chapter title 4
5. Chapter title 5
Part C
6. Chapter title 6

Ik kan losse pagina's maken en de index later aanpassen, maar liefst wil ik dat de index ook automatisch gemaakt wordt. Iemand een idee voor een pakket oid?

[ Bericht 5% gewijzigd door rijsttafel op 12-12-2011 18:43:49 ]
Fok!-reputatiemeter van Rijsttafel:
Slecht [-----X----------------------------------------------------------] Goed
pi_105505776
quote:
2s.gif Op zaterdag 3 december 2011 18:21 schreef kutkloon7 het volgende:

[..]

Ik had een keer een korte inleveropgave uitgewerkt in LaTeX waar één woord werd afgebroken, dus toen heb ik het voor het gemak maar even uitgezet. Verder geen idee hoe het anders moet ook :P.
Aangeven waar het woord wel afgebroken moet worden mbv. \-?
pi_105506706
@rijsttafel:
\part{} gebruiken? Zie ook:
http://en.wikibooks.org/w(...)#Sectioning_Commands
pi_105508391
Ow, wist niet dat die bestond :P Dat zal vast gaan lukken! Thx.
Fok!-reputatiemeter van Rijsttafel:
Slecht [-----X----------------------------------------------------------] Goed
pi_105658187
quote:
7s.gif Op maandag 12 december 2011 18:31 schreef rijsttafel het volgende:

[..]

Volgens mij is het wel redelijk gebruikelijk om het caption van een plaatje langer te maken dan 1 zin. Dat kun je makkelijk doen en dan krijg je dus zoiets:

[ http://imageshack.us/photo/my-images/24/latexcol.png/ (copy/paste deze link) | http://img24.imageshack.us/img24/2002/latexcol.th.png (copy/paste deze link) ]

Wat betreft het afbreken van woorden, heb je wel het Nederlandse tekstpakket gebruikt? De herkenning van afbreekpunten is namelijk afhankelijk van de taal. (edit: ik zie nu dat je zoiets al gevonden had)
Ja dat is precies wat ik nodig heb! Hoe heb je die plaatjes naast elkaar gekregen met tekst er apart onder? ah.. ik denk dat ik al weet hoe je dat hebt gedaan! Ik ga het proberen.

Wat betreft woordafbreking: dat gaat nog niet helemaal goed bij mij. Volgens mij heb ik niet de goede woordenlijst / niet goed toegevoegd / niet geactiveerd. Weet iemand waar ik het NL'se tekstpakket kan downloaden + een instructie om het tekstpakket te activeren?

SPOILER
Om spoilers te kunnen lezen moet je zijn ingelogd. Je moet je daarvoor eerst gratis Registreren. Ook kun je spoilers niet lezen als je een ban hebt.
pi_105671438
Plaatjes naast elkaar kan met subfloats. Volgens mij in het package subfig. Succes!

Ik doe trouwens vaak meerdere zinnen als onderschrift van een figuur, maar bij subfigs zet ik het in het algemeen liever in de caption van het gehele plaatje aangezien het vaak om een vergelijking gaat tussen de twee figuren.

Anyway, zie zelf maar wat je doet. Ben blij dat ik kon helpen. Lukt het niet dan moet je het laten weten.

Daarnaast, aangaande het afbreken van woorden, kun je bij je package manager de "dutch hyphenation" installeren. Misschien helpt dat ook!
Fok!-reputatiemeter van Rijsttafel:
Slecht [-----X----------------------------------------------------------] Goed
pi_105687522
Ik heb liggen kloten en toen werkte het niet meer. Heb MikTex verwijderd en nu weer opnieuw geinstalleed (2.9). Nu krijg ik de melding:

1
2
3
Package babel Warning: No hyphenation patterns were loaded for
(babel)        the language ‘Dutch’
(babel)        I will use the patterns loaded for \language=0 instead.

NIET meer. Dat is dus fijn. Maar, nog steeds breekt hij de woorden niet goed af.

Een aantal voorbeelden:
Ef- fect
Belas- tingtarief
documen- tatie

enzovoort...
Zo ziet mijn body er atm uit:

SPOILER
Om spoilers te kunnen lezen moet je zijn ingelogd. Je moet je daarvoor eerst gratis Registreren. Ook kun je spoilers niet lezen als je een ban hebt.
Wat doe ik verkeerd? Ik heb ''Dutch hyphenation patterns'' (miktex-hyph-dutch) gedownload @ MiKTeX Package Manager.
Volgens mij, op een of andere manier, gebruikt LaTeX niet de Nederlandse woordafbreking (misschien is die niet aanwezig / onvolledig?). Wie kan helpen?

Edit: Is het anders niet mogelijk om het woord dat afgebroken zou moeten worden automatisch een regel te laten opschuiven (en dat LaTeX de afstand tussen de woorden iets groter maakt?)

Edit 2: Hmm. Ik heb nu deze commando's gebruikt:

1
2
\hyphenpenalty=9000
\tolerance=100000
Hierdoor wordt het ''tussenwit'' groter en gaat LaTeX dus minder afbreken. Het stoort minder als er maar 1 woord per pagina (een beetje raar) is afgebroken dan 10 woorden.

[ Bericht 5% gewijzigd door stevin op 17-12-2011 13:15:10 ]
pi_105690513
Wat is er mis met de afbreking in die woorden?

Maar inderdaad is 10 afgebroken woorden per pagina wel erg veel. Zoveel heb ik er misschien nieteens in m'n hele scriptie.
Fok!-reputatiemeter van Rijsttafel:
Slecht [-----X----------------------------------------------------------] Goed
pi_105691584
Wat er mis is met de afbreking? Belas-tingtarief zou eerder belasting-tarief moeten zijn. En Effect zou niet moeten worden afgebroken maar naar de volgende regel.

En hoe heb je maar 10 afgebroken woorden gekregen in je gehele scriptie?
(welke commando's / instellingen)
pi_105692675
10 in m'n hele scriptie is een wat optimistische schatting :P Maar sowieso niet 10 per pagina. Ik heb niets veranderd, maar ik werk in het Engels en heb sowieso veel plaatjes en formules, dus dat verklaart het misschien.
Fok!-reputatiemeter van Rijsttafel:
Slecht [-----X----------------------------------------------------------] Goed
pi_105693746
Ah jah... inderdaad. In het Engels zal het wel (beter) werken. Ik ga het gewoon zo proberen. Bedankt voor het meedenken!
pi_105694191
quote:
0s.gif Op zaterdag 17 december 2011 11:21 schreef stevin het volgende:
Ik heb liggen kloten en toen werkte het niet meer. Heb MikTex verwijderd en nu weer opnieuw geinstalleed (2.9). Nu krijg ik de melding:
[ code verwijderd ]

NIET meer. Dat is dus fijn. Maar, nog steeds breekt hij de woorden niet goed af.

Een aantal voorbeelden:
Ef- fect
Belas- tingtarief
documen- tatie

enzovoort...
Zo ziet mijn body er atm uit:

Ehm, de door Latex voorgestelde woordafbrekingen zijn wel juist hoor... Op elke plaats waar je in lettergrepen kan splitsen kan je een woord afbreken. Oké, belasting-tarief oogt misschien natuurlijker, maar belas-tingtarief is even juist. Niemand die je daar met recht en rede op zal afrekenen.

Bewijs:
http://woordenlijst.org/zoek/?q=belastingtarief&w=w
http://woordenlijst.org/zoek/?q=effect&w=w
http://woordenlijst.org/zoek/?q=documentatie&w=w
pi_105695503
quote:
0s.gif Op zaterdag 17 december 2011 15:40 schreef tony_clifton- het volgende:

[..]

Ehm, de door Latex voorgestelde woordafbrekingen zijn wel juist hoor... Op elke plaats waar je in lettergrepen kan splitsen kan je een woord afbreken. Oké, belasting-tarief oogt misschien natuurlijker, maar belas-tingtarief is even juist. Niemand die je daar met recht en rede op zal afrekenen.

Bewijs:
http://woordenlijst.org/zoek/?q=belastingtarief&w=w
http://woordenlijst.org/zoek/?q=effect&w=w
http://woordenlijst.org/zoek/?q=documentatie&w=w
Bedankt voor de opheldering. Ik zie dat de woordafbreking wel juist verloopt. Toch vind ik het soms wat raar uitzien, maar dat ligt misschien aan mij ;)
pi_105696318
Is normaal hoor in het begin, ik heb de eerste keer ook eens moeten kijken wat Latex nu eigenlijk aan't doen was met de woordafbrekingen, maar ik moet nog steeds een woord tegenkomen waarbij het fout loopt :).

Eigenlijk moet je ervan uit gaan dat Latex heilig is; je bekommert je enkel om de inhoud, Latex doet de rest. Klinkt misschien fanboy maar ik heb zelf ondervonden dat 't wel klopt eigenlijk :).
pi_105884051
Ik heb nog een aantal vragen:

Kun je voorkomen dat elke alinea inspringt? (hoe?)
(\nolinebreak[number] werkt wel... maar dan moet je dat bij elke alinea doen... dat is erg veel werk. Waarschijnlijk kan er in de preamble ook een commando worden geplaatst)

Ik heb 1 A4 vol met tekst en 5 woorden passen er net niet op en gaan op pagina 2. Het liefste zou ik die 5 woorden nog op dat andere A4'tje hebben, is dit mogelijk?

Hoe kan ik het paginaformaat bewerken?
Ik wil er een soort boekje van maken van 24cm x 16 cm
1\special{papersize=160mm,240mm}
werkte niet :<
A5 formaat vind ik te klein.

Kun je ook een zin dat onderaan de pagina staat naar de volgende pagina krijgen?
Kan met
1\clearpage


[ Bericht 11% gewijzigd door stevin op 21-12-2011 21:12:37 ]
pi_105900235
Alinea's niet inspringen:

\setlength{\parskip}{1.3ex plus 0.2ex minus 0.2ex}
\setlength{\parindent}{0pt}

of bewerken hoe je het zelf wilt.
Fok!-reputatiemeter van Rijsttafel:
Slecht [-----X----------------------------------------------------------] Goed
  donderdag 22 december 2011 @ 09:25:44 #236
22818 V2
Deze ruimte is te huur!
pi_105906704
Voor het paginaformaat: geometry package gebruiken.
As far as we know, our computer has never had an undetected error.
pi_105939690
Om te voorkomen dat enkele woorden naar een nieuwe pagina springen kun je
1
2
\widowpenalty=300
\clubpenalty=300
in de preamble zetten. Zie ook http://en.wikibooks.org/wiki/LaTeX/Page_Layout#Widows_and_orphans

Verder helpt het natuurlijk ook als je geen linebreaks forceert en hyphanation goed gaat. Eventueel kun je ook parskip wat kleiner zetten zoals in rijsttafel's post.
pi_105941558
quote:
7s.gif Op donderdag 22 december 2011 00:58 schreef rijsttafel het volgende:
Alinea's niet inspringen:

\setlength{\parskip}{1.3ex plus 0.2ex minus 0.2ex}
\setlength{\parindent}{0pt}

of bewerken hoe je het zelf wilt.
Thanks, werkt!

quote:
3s.gif Op donderdag 22 december 2011 09:25 schreef V2 het volgende:
Voor het paginaformaat: geometry package gebruiken.
Het is me gelukt (via \textheight = 590pt)

quote:
0s.gif Op donderdag 22 december 2011 22:34 schreef Toryu het volgende:
Om te voorkomen dat enkele woorden naar een nieuwe pagina springen kun je
[ code verwijderd ]

in de preamble zetten. Zie ook http://en.wikibooks.org/wiki/LaTeX/Page_Layout#Widows_and_orphans

Verder helpt het natuurlijk ook als je geen linebreaks forceert en hyphanation goed gaat. Eventueel kun je ook parskip wat kleiner zetten zoals in rijsttafel's post.
Thanks!
pi_106097525
[vraag 1] Ik zou graag géén witregel meer hebben tussen de titel van de subsubsections. Nu staat er een 'enter' tussen.

Voorbeeld:
1
2
3
Titel

Hier begint de tekst

Ik wil:
1
2
Titel
Hier begint de tekst

[vraag 2] Daarnaast zou ik de fontsize van de sections willen aanpassen.

Ik heb de package ''titlesec'' en hier een handleiding (http://www.ctex.org/documents/packages/layout/titlesec.pdf) maar ik kom niet er nog niet helemaal uit. Iemand een idee?

Edit:
[vraag 3] De footnotes beginnen nu telkens weer bij 1 wanneer ik een nieuw hoofdstuk begin. Hoe kan ik LaTeX ze laten doornummeren?

[ Bericht 7% gewijzigd door stevin op 28-12-2011 12:36:26 ]
pi_106284090
Vraag 3 heb ik inmiddels opgelost door ze maar handmatig te nummeren... gelukkig waren het niet al te veel voetnoten
(\footnote[number]{tekst})
pi_106351079
Vraagje mensen...

1
2
3
4
5
6
7
8
\tt{[true*.press\_start.!( \\
\qquad finished \\ 
|| get\_user\_selection(no\_selection) \\
|| sense\_coffeepowder(not\_ok) \\ 
|| sense\_sugar(not\_ok) \\ 
|| sense\_milk(not\_ok) \\ 
|| sense\_cup(not\_ok) \\
)*.press\_start]false}

Ik krijg bij finished geen spatie er tussen :(

Variaties als
1
2
3
4
\tt{[true*.press\_start.!(} \\
\qquad \tt{finished} \\ 
\tt{|| get\_user\_selection(no\_selection)} \\
...

1
2
3
4
\tt{[true*.press\_start.!( \\
\hspace{5cm} finished \\ 
|| get\_user\_selection(no\_selection) \\
...

werken allemaal niet :(
pi_106352001
Ben ik ook weer eens. Met een vraagje.

Ik heb dus een afstudeerverslag, gemaakt in LaTeX. Dat heb ik op oneside gezet want ik vind het nogal lelijk met die verspringende marges.Nu wordt het wel dubbelzijdig afgedrukt, maar wil ik de hoofdstukken wel op de rechterpagina laten beginnen. Hoe los ik dat op?

Heb al wat geprobeerd te googlen maar kom er niet aan uit -O-
Vaduz: Je snapt toch wel waarom sbientje het Franse woord bien in d'r naam heeft?
pi_106353082
quote:
0s.gif Op dinsdag 3 januari 2012 13:35 schreef sbientje het volgende:
Ben ik ook weer eens. Met een vraagje.

Ik heb dus een afstudeerverslag, gemaakt in LaTeX. Dat heb ik op oneside gezet want ik vind het nogal lelijk met die verspringende marges.Nu wordt het wel dubbelzijdig afgedrukt, maar wil ik de hoofdstukken wel op de rechterpagina laten beginnen. Hoe los ik dat op?

Heb al wat geprobeerd te googlen maar kom er niet aan uit -O-
Weet 't zo direct ook niet maar gaat 't nog wel mooi ogen wanneer je 't laat inbinden wanneer je de marges gelijk houdt?
pi_106355306
quote:
0s.gif Op dinsdag 3 januari 2012 13:35 schreef sbientje het volgende:
Ben ik ook weer eens. Met een vraagje.

Ik heb dus een afstudeerverslag, gemaakt in LaTeX. Dat heb ik op oneside gezet want ik vind het nogal lelijk met die verspringende marges.Nu wordt het wel dubbelzijdig afgedrukt, maar wil ik de hoofdstukken wel op de rechterpagina laten beginnen. Hoe los ik dat op?

Heb al wat geprobeerd te googlen maar kom er niet aan uit -O-
Gewoon \documentclass[a4paper,10pt,twoside]{report} ???, twoside toevoegen dus.
pi_106394909
quote:
0s.gif Op dinsdag 3 januari 2012 14:00 schreef tony_clifton- het volgende:

[..]

Weet 't zo direct ook niet maar gaat 't nog wel mooi ogen wanneer je 't laat inbinden wanneer je de marges gelijk houdt?
ik vind het juist heel lelijk met die rare marges. Heb ook altijd het idee dat ze precies verkeerdom zitten :')
quote:
7s.gif Op dinsdag 3 januari 2012 14:47 schreef Dale. het volgende:

[..]

Gewoon \documentclass[a4paper,10pt,twoside]{report} ???, twoside toevoegen dus.
ja maar dan zit ik dus met die marges. Begint ieder hoofdstuk dan op de rechterpagina trouwens? weer wat geleerd
Vaduz: Je snapt toch wel waarom sbientje het Franse woord bien in d'r naam heeft?
  woensdag 4 januari 2012 @ 14:32:20 #246
117598 Gebraden_Wombat
lekker bij rijst
pi_106395485
quote:
0s.gif Op woensdag 4 januari 2012 14:17 schreef sbientje het volgende:

[..]

ik vind het juist heel lelijk met die rare marges. Heb ook altijd het idee dat ze precies verkeerdom zitten :')

[..]
Pak eens een willekeurig studieboek uit je kast en kijk naar de marges, dan ziet het er opeens niet meer lelijk uit.
Op dinsdag 23 augustus 2011 23:18 schreef problematiQue het volgende:
Mensen die zomaar claimen dat A beter is dan B moet je gewoon negeren. Internetruis.
pi_106401733
quote:
0s.gif Op woensdag 4 januari 2012 14:32 schreef Gebraden_Wombat het volgende:

[..]

Pak eens een willekeurig studieboek uit je kast en kijk naar de marges, dan ziet het er opeens niet meer lelijk uit.
mijn studieboeken hebben geen ongelijke marges. En zeker niet de grotere marge aan de buitenkant..
Vaduz: Je snapt toch wel waarom sbientje het Franse woord bien in d'r naam heeft?
  woensdag 4 januari 2012 @ 18:02:27 #248
117598 Gebraden_Wombat
lekker bij rijst
pi_106404553
Ik heb nog wat meer studieboeken uit de kast getrokken en ze hebben het inderdaad niet allemaal. Sommige hebben vreemd genoeg de grote marge links op de linkerbladzijde en ook links op de rechterbladzijde.

Maar als workaround op je vraag: je kunt de marges aanpassen of als je dat per se niet wilt handmatig lege pagina's toevoegen indien nodig.
Op dinsdag 23 augustus 2011 23:18 schreef problematiQue het volgende:
Mensen die zomaar claimen dat A beter is dan B moet je gewoon negeren. Internetruis.
pi_107144823
Wanneer ik bijv. 4 bibitems citeer met:
1\cite{item1,item2,item3,item4}
Dan krijg je [1,2,3,4], hoe zorg ik dat LaTeX hier [1-4] van maakt?
pi_107172602
zo:
1\usepackage[numbers,sort&compress]{natbib}
bron
pi_107190222
^O^
pi_108324295
nog even terugkomend op deze post van mezelf over hoofdstukken op de rechterpagina beginnen. Wat ik zoek is dus eigenlijk een manier om alle hoofdstukken automatisch op de rechterpagina te krijgen.Twoside toevoegen doet dit niet, bij mij iig niet?

edit - volgens mij moet \cleardoublepage het doen, na ieder hoofdstuk? ik ga t eens proberen :)
edit - yep, werkt. voortaan beter googlen

[ Bericht 13% gewijzigd door sbientje op 23-02-2012 22:19:05 ]
Vaduz: Je snapt toch wel waarom sbientje het Franse woord bien in d'r naam heeft?
pi_108410815
Welke referentie manager gebruiken jullie hier? Ik heb op mijn werk goede verhalen van iemand gehoord over JabRef, maar misschien is er nog een handiger programma?
You don't need a weatherman to know which way the wind blows.
---------------------------------------------------------------------------------------------------------------------------------------------
last.fm Album top 100
  woensdag 7 maart 2012 @ 22:26:09 #254
46802 Visitor.Q
Neemt alles op!
pi_108843939
Het is grappig hoezeer je dingen die je dacht wel even op te schrijven ineens niet meer zeker weet als de deadline in beeld komt.

Vraag: hoe kan ik in de colofon van mijn boekje nou netjes opschrijven dat de tekst is gezet in (met?) LaTeX met (in?) Libertine?

En dat in het Engels; ik heb nu

Typesetting using \LaTeX\ in Linux Libertine.

Maar op de een of andere manier lijkt dat niet helemaal juist te lopen (with LaTeX?).
  woensdag 7 maart 2012 @ 22:27:11 #255
46802 Visitor.Q
Neemt alles op!
pi_108844009
quote:
0s.gif Op zondag 26 februari 2012 12:38 schreef Felagund het volgende:
Welke referentie manager gebruiken jullie hier? Ik heb op mijn werk goede verhalen van iemand gehoord over JabRef, maar misschien is er nog een handiger programma?
Ik gebruik JabRef maar ik zag laatst iemand bezig met papers voor MacOS, en dat zag er super relaxed uit. Er is ook een plugin voor Firefox waarmee je al browsend papers etc kunt opslaan, dat heb ik even gebrujikt maar dat was voor de tijd van automatisch syncende browsers en dat ging op allerlei computers dus uit de pas lopen...
pi_108849821
Ik ook Jabref. Heb ook een tijdje Mendelay geprobeerd, maar dat vond ik niet echt lekker werken.
pi_109838604
Heef iemand enig idee hoe je in Texmaker makeglossaries met xindy moet uitvoeren? Ik snap de commando's in je tekst wel maar dat makeglossaries-perl-script (?) doet mij hoofdpijn krijgen...

Momenteel ziet mijn snelle compilatie er zo uit;

Pdflatex
Bibtex
Pdflatex (2x)
View PDF

Dus ik zou hier graag ergens iets tussenkrijgen dat mijn woordenlijst/afkortingenlijst mee gecompileerd wordt; maar het lukt mij maar niet...

Alvast heel erg bedankt aan diegene die dat weet :).
pi_109841706
Bij Options > Configure Texmaker > Commands:
Het makeindex commando vervangen met makeglossaries. Als je dan makeindex uitvoert vanuit Texmaker, wordt makeglossaries aangeroepen. Met een beetje geluk werkt dat, maar ik heb het zelf niet geprobeerd.
pi_109863312
Nop, werkt niet. Anyway, heb een walkaround gevonden via de terminal. Beetje ongebruiksvriendelijk maar het werkt wel!

in de terminal de map naar het pad van de texfile veranderen en dan gewoon makeglossaries en de documentnaam...
pi_110100225
n00bvraag:
Ik heb de matrix
Df = \left( \begin{array}{cc} \frac{\partial f_1}{\partial t} & \frac{\partial f_1}{\partial v} \\ \frac{\partial f_2}{\partial t} & \frac{\partial f_2}{\partial v} \end{array} \right).
Deze wil ik iets ruimer maken, want de \partial's raken elkaar zowat in de pdf (hier valt het nog wel mee).
pi_110100642
quote:
2s.gif Op maandag 9 april 2012 17:10 schreef kutkloon7 het volgende:
n00bvraag:
Ik heb de matrix
Df = \left( \begin{array}{cc} \frac{\partial f_1}{\partial t} & \frac{\partial f_1}{\partial v} \\ \frac{\partial f_2}{\partial t} & \frac{\partial f_2}{\partial v} \end{array} \right).
Deze wil ik iets ruimer maken, want de \partial's raken elkaar zowat in de pdf (hier valt het nog wel mee).
Je kunt de \arraystretch verhogen met
1\renewcommand{\arraystretch}{1.2}
je kunt de waarde natuurlijk vergroten naar eigen smaak.

Je kunt ook klooien met \extrarowheight als je de array package hebt.
Beter onethisch dan oneetbaar
pi_112164116
Iemand enig idee welke font dit is in latex? En welke package ik nodig heb?

pi_112169838
Ziet behoorlijk als mathpazo uit.
pi_112529202
Hallo,

ik heb een vraagje over de bibliografie.
Ik doe dat bijna altijd met een aparte .bib file. Ik doe dan de tex-file zowel "bibtex" als "latex" (een aantal keer om ? weg te werken) Dat is gemakkelijk omdat op website referenties naar werken vaak in bibtex-formaat staan, zo dus:
1
2
3
4
5
6
7
8
9
10
@article{bartel2009,
    Author = {Bartel, David P. },
    Date = {2009/01/23},
    Journal = {Cell},
    Month = {01},
    Number = {2},
    Pages = {215--233},
    Title = {Micro{RNA}s: Target Recognition and Regulatory Functions},
    Volume = {136},
    Year = {2009}}
Maar soms moeten de referenties in de tex file zelf staan, en dat moet dan zo:
1
2
3
4
\bibitem[Bartel(2009)]{bartel2009}
David~P. Bartel.
\newblock Micro{RNA}s: Target recognition and regulatory functions.
\newblock \emph{Cell}, 136\penalty0 (2):\penalty0 215--233, 01 2009.
Hoe kan ik dit converteren?
http://texblog.org/2011/1(...)-bibtex-to-bibitems/
Volgens deze site moet ik eerst op de eerste manier het doen, vervolgens in de bbl-file gaan kijken, en dan weer mijn bib file weg doen en dat stukje uit de bblfile in de texfile plaatsen? Dat is toch vrij omslachtig? :'( :D
  woensdag 6 juni 2012 @ 18:33:26 #265
148823 znarch
ondertitel
pi_112530792
quote:
0s.gif Op woensdag 6 juni 2012 17:48 schreef zuiderbuur het volgende:
Hallo,

Wat ik sowieso even niet snap. Waarom "moet" hij in de tex file staan en niet gewoon in je .bib zoals je gewoonlijk doet?
"AAAAAHH ZENNE MOAT, WOARST VLEISCH"
pi_112531008
quote:
0s.gif Op woensdag 6 juni 2012 18:33 schreef znarch het volgende:

[..]

Wat ik sowieso even niet snap. Waarom "moet" hij in de tex file staan en niet gewoon in je .bib zoals je gewoonlijk doet?
In mijn geval nu omdat een mede-auteur er nu op die wijze al aan begonnen is. Maar algemener: sommige tijdschriften staan erop dat je hen één tex-file stuurt, zonder aparte bibliografie.
pi_112531171
quote:
0s.gif Op woensdag 6 juni 2012 18:33 schreef znarch het volgende:

[..]

Wat ik sowieso even niet snap. Waarom "moet" hij in de tex file staan en niet gewoon in je .bib zoals je gewoonlijk doet?
Ik doe het altijd op de manier beschreven :P ook omdat ik niet precies weet hoe .bib werkt :'( (maar ben wel lui genoeg om uren te spenderen als er iets niet goed is met de layout van hetgene waar ik aan werk in latex dé ironie :') _O- )
  woensdag 6 juni 2012 @ 18:58:37 #268
148823 znarch
ondertitel
pi_112531774
quote:
7s.gif Op woensdag 6 juni 2012 18:44 schreef Dale. het volgende:

[..]

Ik doe het altijd op de manier beschreven :P ook omdat ik niet precies weet hoe .bib werkt :'( (maar ben wel lui genoeg om uren te spenderen als er iets niet goed is met de layout van hetgene waar ik aan werk in latex dé ironie :') _O- )
Zodra je artikelen/thesis etc groter worden is het echt aan te raden aparte files te gebruiken. Voor mijn thesis had ik zelfs ieder hoofdstuk in een aparte tex file, een aparte style file, een aparte bibfile etc etc en 1 hoofdfile. Zo houdt je goed overzicht.
"AAAAAHH ZENNE MOAT, WOARST VLEISCH"
pi_112557868
Ik doe bij grotere documenten ook altijd meerdere .tex files en 1 of meerdere .bib-files.
Als je losse .bib's gebruikt kun je je referenties ook makkelijker managen met behulp van programma's als Bibtex of Mendelay.
pi_112563076
Ik gebruik dus ook liefst bib files, je maakt ook sneller fouten als je 100 referenties hebt die je allemaal zelf mag gaan zitten intikken.

Maar hoe nu converteren?? Ik heb het net gedaan: aparte tex gemaakt, referenties in bib geplaatst, tex gecompileerd en ervoor gezorgd dat er effectief referenties naar al die werken in staan, bbl file geopend (foutmelding gekregen dat hij dat niet kan openen :') ),.... dit moet toch sneller kunnen? |:(
pi_112601944
quote:
Dank je, dat is een handige site, maar tenzij ik het verkeerd begrijp, wordt daar uitgelegd hoe je met een bib file kan werken. Dat doe ik al, mijn probleem is net dat ik meestal referenties in het bibtex formaat als input krijg, en ik weet niet hoe ik dat kan converteren naar iets dat in de tex file zelf kan.
pi_112602080
pdflatex/bibtex runnen zoals normaal, dan in je bestand <paper>.bbl kijken; daar heeft bibtex alles naar \bibitems geconverteerd die je dan handmatig in je tex kan copy/pasten. Dit vergt dus wel wat handig management (in geval van wijziging van bibitems).
More oneness, less categories
Open hearts, no strategies
Decisions based upon faith and not fear
People who live right now and right here
pi_114437273
Hoe kan ik een mooi staartdeling in tex maken? Het beste resultaat dat ik tot nu toe heb was met
1
2
3
4
5
6
7
8
9
10
11
12
\begin{tabular}{l l r  l  r}
 8 & / &  1000 & $\backslash$ &  125 \\
&& 800 &  & 100 \\
\hline 
&& 200 & & \\
&& 160 &  & 20\\
\hline
&& 40 & & \\
&& 40 && 5 \\
\hline 
&& 0 & & 
\end{tabular}


Maar ik vind de lijnen zoals JvdC dat doet in Basisboek wiskunde (pag 7, korte lijntjes precies onder het getal, ipv een lange \hline) mooier. Iemand enig idee?
pi_114438328
edit: nvm, vraag verkeerd gelezen

[ Bericht 95% gewijzigd door thabit op 20-07-2012 00:01:49 ]
pi_114439004
Is niet helemaal wat we bedoelden geloof ik.. hoewel met een beetje aanpassen het vast ook wel werkt. Maar ik wilde m'n post net gaan editten, want zoals altijd met latex bleek de oplossing simpeler dan gedacht. Gewoon een \underline
pi_114649147
Ik wil gewoon mijn referenties in de goede format krijgen, maar het lukt niet...Wel mijn bibliografie, maar ik wil gewoon als ik iets citeer iets als [Author, 2007] of Author [2007] ofzo krijgen...
In die hele bibstyles.pdf van TexShop (ik gebruik een mac vandaar) staat niks wat ik wil. Hoe kan je dit zelf modificeren / waar zijn er meer styles te downloaden?
"Some guys they just give up living and start dying little by little piece by piece"
last.fm | Rate Your Music | MusicMeter | top 100 nummers | top 100 albums | top 50 2013 | top 100 jazz | Onze-blog: pat-sounds
pi_114806474
quote:
Dank, ga ik eens mee kijken morgen... Maar nu even uit mijn hoofd maakt mijn bibtex alleen maar error omtrent natbib...toch iets aan de hand met de compilatie ofzo...ik weet niet. Morgen vertel ik meer :P
"Some guys they just give up living and start dying little by little piece by piece"
last.fm | Rate Your Music | MusicMeter | top 100 nummers | top 100 albums | top 50 2013 | top 100 jazz | Onze-blog: pat-sounds
pi_114820634
Author [2007] is in LaTeX doorgaans Author~\cite{auth07}, bijvoorbeeld. Hetgeen te veranderen is wat tussen de [ en ] staat, en dat gebeurt met \bibliographystyle (bv. alpha, plain, unsrt, ..). Meestal is dat afdoende? Als je je eigen stijl ground-up wil maken, voer dan `latex makebst' in een terminal uit.
More oneness, less categories
Open hearts, no strategies
Decisions based upon faith and not fear
People who live right now and right here
pi_114821187
quote:
2s.gif Op zondag 29 juli 2012 21:29 schreef trancethrust het volgende:
Author [2007] is in LaTeX doorgaans Author~\cite{auth07}, bijvoorbeeld. Hetgeen te veranderen is wat tussen de [ en ] staat, en dat gebeurt met \bibliographystyle (bv. alpha, plain, unsrt, ..). Meestal is dat afdoende? Als je je eigen stijl ground-up wil maken, voer dan `latex makebst' in een terminal uit.
Ik zal de error die ik krijg morgen wel even posten, maar die zegt dus precies dat 'Author [year]' citations niet werken...en dat ie terugvalt naar plain numeric
"Some guys they just give up living and start dying little by little piece by piece"
last.fm | Rate Your Music | MusicMeter | top 100 nummers | top 100 albums | top 50 2013 | top 100 jazz | Onze-blog: pat-sounds
pi_114998093
Bovenstaande tips hielpen.

Nu heb ik wel iets anders. Ik heb geen zin om permanent op enter te blijven drukken als er errors zitten in de compilation van pdftex. Waarom zegt hij bijvoorbeeld dat mijn \renewcommand undefined is?? Terwijl die het gewoon doet als de pdf af is...

Dit is de command: \renewcommand{\Gty}{$\mathrm{Gt~y^{-1}}$}, wat is er mis mee? Hij zegt dat ik een begin-math symbols ben vergeten, maar dat is toch niet zo? :/
"Some guys they just give up living and start dying little by little piece by piece"
last.fm | Rate Your Music | MusicMeter | top 100 nummers | top 100 albums | top 50 2013 | top 100 jazz | Onze-blog: pat-sounds
pi_114998583
Ik raad ten sterkste aan je errors te repareren, zo lastig is het allemaal niet. Alle errors negeren is soms wel handig juist om de daadwerkelijke error te vinden; dit doe je met r <enter> (tenminste, in de terminal). Hij gaat dan in non-stop mode verder (zolang het kan).
LaTeX fouten zijn niet altijd even duidelijk, zoals je merkt; vaak is na het fixen van de eerste foutmelding al een heleboel ander gezeur verdwenen. In jouw geval lijkt de fout gewoon te zijn dat er voorheen geen commando \Gty bestond, dus moet je \newcommand gebruiken ipv renewcommand; er is immers geen voorgaande definitie om te vernieuwen.
More oneness, less categories
Open hearts, no strategies
Decisions based upon faith and not fear
People who live right now and right here
pi_115000147
quote:
3s.gif Op donderdag 2 augustus 2012 15:11 schreef trancethrust het volgende:
Ik raad ten sterkste aan je errors te repareren, zo lastig is het allemaal niet. Alle errors negeren is soms wel handig juist om de daadwerkelijke error te vinden; dit doe je met r <enter> (tenminste, in de terminal). Hij gaat dan in non-stop mode verder (zolang het kan).
LaTeX fouten zijn niet altijd even duidelijk, zoals je merkt; vaak is na het fixen van de eerste foutmelding al een heleboel ander gezeur verdwenen. In jouw geval lijkt de fout gewoon te zijn dat er voorheen geen commando \Gty bestond, dus moet je \newcommand gebruiken ipv renewcommand; er is immers geen voorgaande definitie om te vernieuwen.
_O-
duh....dank :P maar die andere error met de mathbegin die gaatniet weg :/ Ik negeer natuurlijk niet alle errors, maar soms zitten er gewoon van die onverklaarbare tussen die gewoon werken (of goed gefixed worden...) en daar wil je doorheen.

:edit ik heb het al opgelost. Ik moet natuurlijk geen mathrm en $s gebruiken tegelijk :P

[ Bericht 3% gewijzigd door Norrage op 02-08-2012 16:12:10 ]
"Some guys they just give up living and start dying little by little piece by piece"
last.fm | Rate Your Music | MusicMeter | top 100 nummers | top 100 albums | top 50 2013 | top 100 jazz | Onze-blog: pat-sounds
pi_115003057
Mwah, het lijkt er eerder op dat je je commando in mathmode aanroept, e.g. $x=\Gty+c$. Dan vult latex de boel in (met je oude definitie) en dan staat er opeens $x=$Gt~y^{-1}$+c$; ofwel, Gt~y^{-1} staat juist niet in mathmode.
Het weghalen van de $'s in de functiedefinitie is dan een juiste oplossing, maar niet vanwege wat je denkt. (\mathrm is zeker wel alleen geldig in mathmode, dus hij moet tussen dollars of iets dergelijks.)
More oneness, less categories
Open hearts, no strategies
Decisions based upon faith and not fear
People who live right now and right here
pi_115372181
Hallo

na jarenlang gebruik van Latex blijf ik sukkelen met het al dan niet beginnen van nieuwe pagina's.
Ik weet dat je nopagebreak en newpage en samepage hebt, maar die dingen lijken me niet te gehoorzamen.

Concreet heb ik nu een tekst waarin van een aantal theorems op het einde van een pagina staat: "Theorem 4.11: het volgende is equivalent:",
waarna de rest op de volgende pagina komt, wat er bijzonder lelijk uitziet.

Ik kan jammer genoeg geen simpele code vinden die net dit probleem simuleert.

Wat is volgens jullie de meest nette manier om dit op te lossen?
Bedankt!
pi_115372416
LaTeX maakt de opmaak naargelang penalty-formules (als ik zoveel % witruimte heb dan is dat 90 erg, als ik een thereom na 1 zin afbreek dan is dat 50 erg). Je kan latex hints given die die formules ietwat aanpassen; maar dat kan dus nog steeds overruled worden. Harde commando's zoals newpage werken wel altijd.

Wat m.i. het beste werkt als je iets echt niet vindt kunnen, is je tekst aanpassen. In jouw geval zou ik `het volgende is equivalent:' gewoon verwijderen en een =-teken toevoegen tussen je formules. Of als je per se de tekst wil houden, deze na de formules plaatsen.
More oneness, less categories
Open hearts, no strategies
Decisions based upon faith and not fear
People who live right now and right here
pi_115390405
quote:
Dank je allebei, maar voorlopig vrees ik dat het bij zoiets al newpage zal blijven. Die penalties by widows and orphans hadden geen enkele invloed.

Ik had verder nog een vraag: ik ben groot liefhebber van \usepackage{hyperref} maar ik heb nu een vervelend probleem. In de pdf klopt de referentie wel, maar als ik erop klik, kom ik zelfs op de verkeerde pagina terecht (op een absurde plaats, altijd dezelfde maar ik zie geen logica erachter)
Aux files trashen of zo... het helpt allemaal niet.
Wat kan hierachter zitten?
pi_115391462
Krijg je warnings over duplicate/missing labels oid?

Anders delen tekst en packages weg-commenten om de oorzaak te isoleren. PLaats anders een minimal working example hier.

edit:
soms wordt \phantomsection gebruikt om problemen met referenties naar secties op te lossen. Ik weet alleen niet precies wat het doet/hoe het werkt.
pi_115397736
quote:
0s.gif Op vrijdag 10 augustus 2012 19:29 schreef Toryu het volgende:
Krijg je warnings over duplicate/missing labels oid?

Anders delen tekst en packages weg-commenten om de oorzaak te isoleren. PLaats anders een minimal working example hier.

edit:
soms wordt \phantomsection gebruikt om problemen met referenties naar secties op te lossen. Ik weet alleen niet precies wat het doet/hoe het werkt.
Ik kreeg allerlei vreemde warnings, zoals :
quote:
l.376 \begin{eqnarray}
pdfTeX warning (ext4): destination with the same identifi
er (name{equation.4.4}) has been already used, duplicate ignored
Na wat moeite heb ik dan toch een minimal working example kunnen maken, en het was vrij leerzaam:

1
2
3
4
5
6
7
8
9
10
11
12
13
14
15
16
17
18
19
20
\documentclass[12pt,a4paper]{article}
 \usepackage{hyperref}
 \title{Probleem met foute hyperref}
\begin{document}
\maketitle

\begin{eqnarray}
a&=&blabla, \nonumber \\
c&=&blabla^2 \nonumber 
\end{eqnarray}

\begin{equation} \label{eq1}
a=b^2
\end{equation}
\begin{equation} \label{eq2}
c=d
\end{equation}

We bekijken (\ref{eq1}) and (\ref{eq2}).
\end{document}
Ik zie nu dat één van mijn voorgangers eqnarray met nonumber gebruikt heeft, en blijkbaar stuurt dat de hele boel in de war? (de eerste referentie brengt je foutief naar de eqnarray, de tweede doet het prima om de één of andere reden) :( :?
pi_115418333
Ik weet niet of het dit op gaat lossen, maar hier in elk geval wat tips:

- Gebruik align ipv eqnarray. eqnarray geeft soms de verkeerde spacing. Dus:
1
2
3
4
\begin{align}
a&=&blabla, \nonumber \\
c&=&blabla^2 \nonumber 
\end{align}
In plaats van \nonumber kun je align* gebruiken, maar zo zou het ook moeten werken.
Zie ook: http://en.wikibooks.org/w(...)s#align_and_align.2A
Ik geloof wel dat je hier de amsmath bij nodig hebt.

- Gebruik \eqref als je naar equation refereert, dus:
1We bekijken \eqref{eq1} and \eqref{eq2}.
pi_115418875
quote:
0s.gif Op zaterdag 11 augustus 2012 11:12 schreef Toryu het volgende:
Ik weet niet of het dit op gaat lossen, maar hier in elk geval wat tips:

- Gebruik align ipv eqnarray. eqnarray geeft soms de verkeerde spacing. Dus:
[ code verwijderd ]

In plaats van \nonumber kun je align* gebruiken, maar zo zou het ook moeten werken.
Zie ook: http://en.wikibooks.org/w(...)s#align_and_align.2A
Ik geloof wel dat je hier de amsmath bij nodig hebt.

- Gebruik \eqref als je naar equation refereert, dus:
[ code verwijderd ]

Ik heb het opgelost door \begin{eqnarray*}...\end{eqnarray*} te gebruiken. Nu werken alle referenties perfect, ik zou niet begrijpen waarom je nonumber overal zou plaatsen? Toch vreemd dat ie daar zo op reageerde.

Verder heb ik nog een ander probleem met de theorems. Ik wil dat de tekst niet cursief is. Deze code werkt, maar de regel "\usepackage{theorem}" is echt nodig.
1
2
3
4
5
6
7
8
9
10
\documentclass[12pt,a4paper]{article}
\usepackage{theorem}
\theorembodyfont{\rmfamily}
\newtheorem{theorem}{Theorem}
 \title{Probleem met theorembodyfont}
\begin{document}
\maketitle
\begin{theorem}test
\end{theorem}
\end{document}
Nu heb ik een grote file waarin "\usepackage{theorem}" blijkbaar botst met andere zaken. Maar is het niet vreemd dat voor iets vrij elementairs, je toch zo'n package moet inladen?
pi_116062687
Ik heb een nieuw probleem... ik moet een appendix aanmaken en dan nog wel binnen het elsevier-formaat.

Dus ik dacht aan zoiets:
quote:
Appendix : Verdere resultaten

Theorem A.1: blabla
Maar als ik dan
1
2
3
4
5
6
7
8
\documentclass[preprint,12pt]{elsarticle}
\newtheorem{theorem}{Theorem}[section]
\begin{document}
\appendix{titel}
\begin{theorem}
blabla
\end{theorem}
\end{document}
doe krijg ik
quote:
titel
Theorem Appendix .1. Een stelling
(de appendix wordt niet eens duidelijk aangekondigd?)
pi_116063955
\usepackage{appendix}?

oh, en: \appendix als single command, en losse appendices gewoon met \chapter{...}; \appendix geeft puur het begin aan van de appendix hoofdstukken. Je hebt ook een versie waar je \begin{appendices} en \end{appendices} kan gebruiken, maar dat lijkt me hier niet nuttig.
More oneness, less categories
Open hearts, no strategies
Decisions based upon faith and not fear
People who live right now and right here
pi_116468443
Weet iemand hoe je makkelijk het onderwerp (bv. Betreft: de schoenen van Elvis) aan een brief toevoegt? Ik gebruik \documentclass{letter}
pi_116468561
Ik dacht dat ik het helemaal gevonden had voor mijn appendix:
Het ziet er prima uit maar .... de referentie op het einde is een hyperlink die naar het verkeerde leidt!
Die setcounter is de boosdoener blijkbaar.
1
2
3
4
5
6
7
8
9
10
11
12
13
14
15
16
17
18
19
20
\documentclass[preprint,12pt]{article}
\usepackage{amssymb}
\usepackage{amsthm}
\newtheorem{theorem}{Theorem}[section]
\usepackage{hyperref}

\begin{document}

\section{Eerste sectie}
\begin{theorem} \label{stelling}Blabla\end{theorem}
\newpage

\section*{Appendix:Titel van mijn appendix}
\setcounter{theorem}{0 }
 \renewcommand{\thetheorem}{A.\arabic{theorem}}

 \begin{theorem} \label{appendixstelling}Blabla\end{theorem}
  Een verwijzing naar Theorem \ref{appendixstelling}
 
 \end{document}
pi_116474397
Niet gewoon zo?
1
2
3
4
\appendix
\section{Titel van mijn appendix}
 \begin{theorem} \label{appendixstelling}Blabla\end{theorem}
  Een verwijzing naar Theorem \ref{appendixstelling}
pi_116480346
quote:
0s.gif Op donderdag 6 september 2012 20:31 schreef Toryu het volgende:
Niet gewoon zo?
[ code verwijderd ]

Misschien, maar dan heb ik geen controle over de titel, dan krijg ik "A .... " en dat wil ik niet.

Ondertussen heb ik ontdekt dat je het met een klein trucje zo in orde kan krijgen:
SPOILER
Om spoilers te kunnen lezen moet je zijn ingelogd. Je moet je daarvoor eerst gratis Registreren. Ook kun je spoilers niet lezen als je een ban hebt.
pi_116483487
quote:
0s.gif Op donderdag 6 september 2012 18:40 schreef Toryu het volgende:
Weet iemand hoe je makkelijk het onderwerp (bv. Betreft: de schoenen van Elvis) aan een brief toevoegt? Ik gebruik \documentclass{letter}
Ik doe:
1
2
\begin{letter}{Adres\\ Postbus 1234\\ 2600 AA Delft\\%
[3\parskip]Betreft: Onderwerp}
Maar da's een hack.

[ Bericht 0% gewijzigd door vriendelijkevriend op 06-09-2012 23:17:55 ]
abonnement Unibet Coolblue
Forum Opties
Forumhop:
Hop naar:
(afkorting, bv 'KLB')